deja review behavioral science

241

Upload: melody-smith

Post on 07-Aug-2015

182 views

Category:

Health & Medicine


1 download

TRANSCRIPT

Page 1: Deja review   behavioral science
Page 2: Deja review   behavioral science

DEJA REVIEWTM

Behavioral Science

Page 3: Deja review   behavioral science

NOTICE

Medicine is an ever-changing science. As new research andclinical experience broaden our knowledge, changes in treat-ment and drug therapy are required. The authors and thepublisher of this work have checked with sources believed tobe reliable in their efforts to provide information that is com-plete and generally in accord with the standards accepted atthe time of publication. However, in view of the possibility ofhuman error or changes in medical sciences, neither theauthors nor the publisher nor any other party who has beeninvolved in the preparation or publication of this work war-rants that the information contained herein is in every respectaccurate or complete, and they disclaim all responsibility forany errors or omissions or for the results obtained from use ofthe information contained in this work. Readers are encour-aged to confirm the information contained herein with othersources. For example and in particular, readers are advised tocheck the product information sheet included in the packageof each drug they plan to administer to be certain that theinformation contained in this work is accurate and thatchanges have not been made in the recommended dose or inthe contraindications for administration. This recommenda-tion is of particular importance in connection with new orinfrequently used drugs.

Page 4: Deja review   behavioral science

Gene R. Quinn, MD, MSResident Physician

Department of MedicineUniversity of California, San Francisco

San Francisco, CaliforniaUniversity of Washington School of Medicine

Seattle, WashingtonClass of 2009

New York Chicago San Francisco Lisbon London Madrid Mexico CityMilan New Delhi San Juan Seoul Singapore Sydney Toronto

DEJA REVIEWTM

Behavioral ScienceSecond Edition

Page 5: Deja review   behavioral science

Copyright © 2010, 2007 by The McGraw-Hill Companies, Inc. All rights reserved. Except as permitted under the United States Copyright Act of 1976, no part of this publication may be reproduced or distrib-uted in any form or by any means, or stored in a database or retrieval system, without the prior written permission of the publisher.

ISBN: 978-0-07-163293-5

MHID: 0-07-163293-X

The material in this eBook also appears in the print version of this title: ISBN: 978-0-07-162728-3,

MHID: 0-07-162728-6.

All trademarks are trademarks of their respective owners. Rather than put a trademark symbol after every occurrence of a trademarked name, we use names in an editorial fashion only, and to the benefi t of the trademark owner, with no intention of infringement of the trademark. Where such designations appear in this book, they have been printed with initial caps.

McGraw-Hill eBooks are available at special quantity discounts to use as premiums and sales promotions, or for use in corporate training programs. To contact a representative please e-mail us at [email protected].

TERMS OF USE

This is a copyrighted work and The McGraw-Hill Companies, Inc. (“McGrawHill”) and its licensors reserve all rights in and to the work. Use of this work is subject to these terms. Except as permitted under the Copyright Act of 1976 and the right to store and retrieve one copy of the work, you may not decompile, disassemble, reverse engineer, reproduce, modify, create derivative works based upon, transmit, distribute, disseminate, sell, publish or sublicense the work or any part of it without McGraw-Hill’s prior consent. You may use the work for your own noncommercial and personal use; any other use of the work is strictly prohibited. Your right to use the work may be terminated if you fail to comply with these terms.

THE WORK IS PROVIDED “AS IS.” McGRAW-HILL AND ITS LICENSORS MAKE NO GUAR-ANTEES OR WARRANTIES AS TO THE ACCURACY, ADEQUACY OR COMPLETENESS OF OR RESULTS TO BE OBTAINED FROM USING THE WORK, INCLUDING ANY INFORMATION THAT CAN BE ACCESSED THROUGH THE WORK VIA HYPERLINK OR OTHERWISE, AND EXPRESS-LY DISCLAIM ANY WARRANTY, EXPRESS OR IMPLIED, INCLUDING BUT NOT LIMITED TO IMPLIED WARRANTIES OF MERCHANTABILITY OR FITNESS FOR A PARTICULAR PURPOSE. McGraw-Hill and its licensors do not warrant or guarantee that the functions contained in the work will meet your requirements or that its operation will be uninterrupted or error free. Neither McGraw-Hill nor its licensors shall be liable to you or anyone else for any inaccuracy, error or omission, regardless of cause, in the work or for any damages resulting therefrom. McGraw-Hill has no responsibility for the content of any information accessed through the work. Under no circumstances shall McGraw-Hill and/or its licensors be liable for any indirect, incidental, special, punitive, consequential or similar damages that result from the use of or inability to use the work, even if any of them has been advised of the possibility of such damages. This limitation of liability shall apply to any claim or cause whatsoever whether such claim or cause arises in contract, tort or otherwise.

Page 6: Deja review   behavioral science

To my family

Page 7: Deja review   behavioral science

This page intentionally left blank

Page 8: Deja review   behavioral science

Faculty Reviewers/Student Reviewers xiPreface xiii

SECTION I HUMAN DEVELOPMENT AND PSYCHOLOGY 1

Chapter 1 EARLY STAGES OF LIFE: INFANCY TO CHILDHOOD 3Theories of Development / 3Infant Morbidity and Mortality / 5Neonatal Reflexes / 5Developmental Milestones / 6Attachment / 11Early Adolescence (11 to 14 Years) / 12Clinical Vignettes / 13

Chapter 2 MIDDLE STAGES OF LIFE: ADOLESCENCE TO ADULTHOOD 15Early Adulthood / 15Middle Adulthood / 17Clinical Vignettes / 17

Chapter 3 LATE STAGES OF LIFE: AGING, DEATH, AND BEREAVEMENT 19Aging / 19Dying, Death, and Bereavement / 21Clinical Vignettes / 23

Chapter 4 PSYCHOANALYTIC THEORY 25Clinical Vignettes / 33

Chapter 5 LEARNING THEORY 35Associative Learning vs Nonassociative Learning / 35Clinical Vignettes / 42

Chapter 6 SLEEP SCIENCE AND DISORDERS 43Normal Sleep / 43Abnormal Sleep / 45Other Sleep Changes / 47Clinical Vignettes / 48

Contents

Page 9: Deja review   behavioral science

viii Contents

Chapter 7 SEXUALITY 49Sexual Development / 49Sexual Development and Physiologic Abnormalities / 50Hormones and Their Influence on Behavior / 51Sexual Response Cycle / 51Sexual Dysfunction / 52Paraphilias / 54Influence of Medical Conditions on Sexuality / 54Effects of Drugs and Neurotransmitters on Sexuality / 55Clinical Vignettes / 56

Chapter 8 ABUSE AND AGGRESSION 57Children and Elder Abuse and Neglect / 57Domestic Partner Abuse / 60Sexual Violence / 61Aggression / 62Clinical Vignettes / 63

Chapter 9 SUICIDE 65Suicide / 65Clinical Vignettes / 68

Chapter 10 GENETIC BASIS OF BEHAVIOR 69Genetic Studies / 69Psychiatric Disorders Genetics / 69Neuropsychiatric Disorder Genetics / 71Alcoholism Genetics / 72Clinical Vignettes / 72

Chapter 11 NEUROCHEMISTRY IN BEHAVIORAL SCIENCES 75Neuroanatomy / 75Brain Lesions / 76Neurotransmitters / 77Amines / 78Neuropeptides / 81Clinical Vignettes / 82

SECTION II PSYCHIATRIC DISORDERS AND TREATMENT 83

Chapter 12 PSYCHOTIC DISORDERS 85Introduction / 85Disorders / 85Clinical Vignettes / 92

Page 10: Deja review   behavioral science

Chapter 13 MOOD DISORDERS 93Clinical Vignettes / 101

Chapter 14 ANXIETY DISORDERS 103Panic Disorder / 104Obsessive-Compulsive Disorder / 105Social Phobia / 106Posttraumatic Stress Disorder / 107Specific Phobia / 108Adjustment Disorder with Anxiety / 109Clinical Vignettes / 110

Chapter 15 COGNITIVE DISORDERS 111Delirium / 111Dementia / 112Amnestic Syndromes / 115Clinical Vignettes / 115

Chapter 16 SOMATOFORM DISORDERS 117Somatization Disorder / 118Conversion Disorder / 119Hypochondriasis / 120Body Dysmorphic Disorder / 121Factitious Disorder / 122Malingering / 123Clinical Vignettes / 123

Chapter 17 PERSONALITY DISORDERS 125Cluster A: The Mad / 126Cluster B: The Bad / 127Cluster C: The Sad / 130Clinical Vignettes / 133

Chapter 18 DISSOCIATIVE DISORDERS 135Clinical Vignettes / 136

Chapter 19 SUBSTANCE ABUSE DISORDERS 139Clinical Vignettes / 151

Chapter 20 EATING DISORDERS 153Clinical Vignettes / 156

Contents ix

Page 11: Deja review   behavioral science

Chapter 21 CHILD PSYCHIATRY 159Pervasive Development Disorders / 159Disruptive Behavior Disorders / 161Attention-Deficit Hyperactivity Disorder / 162Other Neuropsychiatric Disorders of Childhood / 162Clinical Vignettes / 164

Chapter 22 PSYCHOPHARMACOLOGY 165Clinical Vignettes / 174

SECTION III ETHICS, HEALTH CARE, AND STATISTICS 175

Chapter 23 CLINICAL PRACTICE AND DIFFICULT SITUATIONS 177Clinical Vignettes / 180

Chapter 24 MEDICAL ETHICS AND LEGAL ISSUES 181Clinical Vignettes / 188

Chapter 25 HEALTH CARE IN THE UNITED STATES 191Health-Care Insurance / 191Health-Care Costs / 193Health-Care Delivery Systems / 194Health Status and Determinants / 195Clinical Vignettes / 196

Chapter 26 EPIDEMIOLOGY AND RESEARCH DESIGN 199Epidemiology / 199Research Study Designs / 201Testing / 203Measures of Association / 205Clinical Vignettes / 206

Chapter 27 BIOSTATISTICS 209Statistical Distribution / 209Statistical Hypothesis and Error Types / 212Statistical Tests / 214Clinical Vignettes / 215

Index 217

x Contents

Page 12: Deja review   behavioral science

Faculty Reviewers

Debra L Klamen, MD, MPHE Associate Dean, Education and CurriculumProfessor and Chair, Department of Medical

EducationSouthern Illinois University School of

MedicineSpringfield, Illinois

Jessica BuryMayo Medical SchoolMD/MPH CandidateClass of 2010

Sarah FabianoSUNY Upstate Medical UniversityClass of 2010

Daniel MarcoviciSackler School of MedicineTel Aviv UniversityClass of 2011

Student Reviewers

Page 13: Deja review   behavioral science

This page intentionally left blank

Page 14: Deja review   behavioral science

Preface

Behavioral science is an extremely high-yield, yet often overlooked, subject on theUSMLE Step 1. Understanding important behavioral science concepts and being ableto apply them to test questions can significantly increase your score. Deja Review:Behavioral Sciences is the perfect format to study behavioral science material; it helpsyou rapidly review material you know as well as fill-in gaps in your knowledge.In this second edition of Deja Review: Behavioral Sciences I have highlighted high-yieldconcepts such as psychiatric disorders and developmental milestones while still keepingthe content comprehensive enough to use during your preclinical course and clerkship.Epidemiology and biostatistics are often tested on the USMLE—this new edition hasfully revised and expanded sections on these subjects that will review and teach youcore concepts needed to work through epidemiology questions on the USMLE. With alittle bit of work and Deja Review: Behavioral Sciences, you’ll be well-prepared for themultitude of behavioral science questions on the USMLE.

ORGANIZATION

All concepts are presented in a question and answer “flashcard” format that covers keyfacts on commonly tested topics in behavioral medicine. Behavioral science is oftenconceptual, and questions are designed to review and teach these concepts instead ofsimply memorizing material.

The question and answer format has several important advantages:• It provides a rapid, straightforward way for you to assess your strengths and

weaknesses.• It serves as a quick, last-minute review of high-yield facts.• It allows you to efficiently review and commit to memory a large body of

information.

At the end of each chapter, you will find clinical vignettes that expose you to the typeof questions classically tested on the USMLE Step 1. These board-style questions putthe basic science into a clinical context, allowing you apply the facts you have justreviewed in a clinical scenario and make the diagnosis.

HOW TO USE THIS BOOK

This text was assembled with the intent to represent the core topics tested on courseexaminations and USMLE Step 1. Remember, this text is not intended to replace com-prehensive textbooks, course packs, or lectures. However, it may serve as a supplement

Page 15: Deja review   behavioral science

to your studies during your coursework and Step 1 preparation. You may use the bookto quiz yourself or classmates on topics covered in recent lectures and clinical case dis-cussions. A bookmark is included so that you can easily cover up the answers as youwork through each chapter. The compact, condensed design of the book is conduciveto studying on the go, especially during any downtime throughout your busy day.

However you choose to study, I hope you find this resource helpful throughout yourpreparation. Good luck and good studying!

Gene R. Quinn

xiv Preface

Page 16: Deja review   behavioral science

S E C T I O N I

Human Developmentand Psychology

Page 17: Deja review   behavioral science

This page intentionally left blank

Page 18: Deja review   behavioral science

C H A P T E R 1

Early Stages of Life:Infancy to Childhood

3

THEORIES OF DEVELOPMENT

Describe Erik Erikson’s theories of Critical periods at which achievementdevelopment. of social goals need to be achieved,

otherwise they won’t be achieved.

Describe Sigmund Freud’s theories of Organized by parts of the body from development. which pleasure is derived at each age of

development. Each stage is part of thedevelopment into the sexual maturity ofadulthood, characterized by egoformation and the ability to delaygratification.

Describe Jean Piaget’s theories of Learning capabilities of the child at development. various ages during development;

children must move through four stagesof development. There is a specific setof skills that must be mastered at eachstage of development beforeprogression to the other stages.

Erik Erikson’s Theory of Development

Which stage of development is Trust vs Mistrust: birth to 18 monthscharacterized by an infant establishing faith in their caregiver?

Which stage of development is Autonomy vs Shame and Doubt:characterized by a child learning age 18 months to 3 yearsphysical skills such as walking and learning to use the bathroom?

Which stage of development is Initiative vs Guilt: age 3 to 6 yearscharacterized by a child becoming assertive in their learning?

Page 19: Deja review   behavioral science

Which stage of development is Industry vs Inferiority: age 6 to 12 yearscharacterized by a child acquiring new skills at a rapid rate?

Which stage of development is Identity vs Role Confusion: age 12 to characterized by a teen who achieves a 18 yearssense of identity in politics, sex roles, or occupation?

Which stage of development is Intimacy vs Isolation: age 19 to 40 yearscharacterized by an adult determining whether or not they want to establish an intimate relationship with another individual?

Which stage of development is Generativity vs Stagnation: age 40 tocharacterized by an adult finding ways 65 yearsto support and encourage the next generation?

Which stage of development is Ego Integrity vs Despair: age 65 yearscharacterized by an adult reflecting to deathon their experiences to derivemeaning from their life?

Sigmund Freud’s Theory of Development

Which stage of development is Oral phase present from birth to characterized by focus on receiving age 1 yearpleasure through food consumption or sucking on pacifiers?

Which stage of development is Anal phase present from age 1 to characterized by focus on receiving 3 yearspleasure through potty training?

Which stage of development is Phallic phase present from age 3 to characterized by focus on identifying 6 yearswith adult role models and the oedipal complex?

Which stage of development is Latency phase present from age 6 to characterized by focus on expanding 12 yearssocial interactions?

Which stage of development is Genital phase present from age 13 to characterized by focus on establishing adulthooda family?

Jean Piaget’s Theory of Development

Which stage of development is Sensorimotor period from birth to characterized by infants and toddlers age 2 yearsfocusing on their eyes, ears, hands, and other senses?

4 Deja Review: Behavioral Science

Page 20: Deja review   behavioral science

Which stage of development is Preoperational thought present fromcharacterized by children acquiring age 2 to 7 yearsrepresentational skills in the area of mental imagery and language?

Which stage of development is Concrete operational present from characterized by children being more age 7 to 12 yearslogical, flexible, and organized than in early childhood?

Which stage of development is Formal operational from age 12 to characterized by being able to think adulthoodlogically, theoretically, and abstractly?

INFANT MORBIDITY AND MORTALITY

Define premature birth. Less than 34 weeks gestation or birthweight less than 2500 g

What are the potential outcomes of Increased infant mortalitybeing a premature infant? Delayed physical and social development

Emotional and behavioral problemsDyslexiaChild abuse

In the United States, what percentage 6% for white women and 13% for of births is premature? African American women (an average

of 7.2 per 1000 live births)

What are the common risk factors Low socioeconomic statusassociated with premature births? Teenage pregnancy

Poor maternal nutrition

NEONATAL REFLEXES

What are the six important neonatal 1. Mororeflexes? 2. Palmar grasp

3. Rooting4. Stepping5. Asymmetric tonic neck6. Parachute

Which neonatal reflexes are present Moro at birth? Palmar grasp

RootingStepping

Early Stages of Life: Infancy to Childhood 5

Page 21: Deja review   behavioral science

What is the Moro/”startle” reflex? Head extension causes extremityextension followed by flexion.

What is the palmar grasp? If a finger is placed in an infant’s palm,it is grasped.

What is the rooting reflex? If an object is placed around an infant’smouth, the infant will pursue it.

What is the stepping reflex? When held upright and leaningforward, an infant will make walkingmotions with their legs.

At what age do the Moro, 4 to 6 monthspalmar, rooting, and steppingreflexes disappear?

What is the asymmetric tonic neck? While supine, turning of the headcauses ipsilateral extremity extensionand contralateral flexion.

At what age does asymmetric tonic Present at 2 weeks and disappearsneck appear and then disappear? by 6 months

What is the parachute reflex? While sitting and tilted to one side, aninfant extends the ipsilateral arm tosupport the body.

DEVELOPMENTAL MILESTONES

What are the key categories of Gross motordevelopment? Fine/visual motor

LanguageSocial

What are the developmental milestones Gross motor: when prone lifts head at 1 month of age? slightly

Fine/visual: with eyes tracks objects to midline; tight graspLanguage: startles to soundSocial: fixes on face

What are the developmental milestones Gross motor: steadily holds head up;at 2 to 3 months of age? when prone lifts chest up

Fine/visual: hands open at restLanguage: smiles responsively; coosSocial: recognizes parents; reaches for familiar objects or people

6 Deja Review: Behavioral Science

Page 22: Deja review   behavioral science

What are the developmental milestones Gross motor: rolls front to back and backat 4 to 5 months of age? to front; sits well supported

Fine/visual: grasps with both handsLanguage: orients to voiceSocial: laughs; enjoys observingenvironment

What are the developmental milestones Gross motor: sits well unsupported; sitsat 6 months of age? upright

Fine/visual: transfers hand to hand;reaches with either handLanguage: babbles

Social: recognizes strangers and hasstranger anxiety

What are the developmental milestones Gross motor: crawls, pulls to standat 9 months of age? Fine/visual: uses pincer grasp; finger

feedsLanguage: says “dada/mama”;understands “no”Social: waves bye-bye; playspat-a-cake

What are the developmental milestones Gross motor: walks aloneat 12 months of age? Fine/visual: throws, releases objects

Language: one to eight words otherthan “dada/mama”; one-stepcommandsSocial: imitates actions; comes whencalled; cooperates with dressing

What are the developmental milestones Gross motor: walks backward; creeps at 15 months of age? upstairs

Fine/visual: builds two-block towers;scribbles; uses a cupLanguage: uses four to eight wordsSocial: throws temper tantrums

What are the developmental milestones Gross motor: runs; kicks a ballat 18 months of age? Fine/visual: feeds self with utensils

Language: points to body partswhen asked; names commonobjectsSocial: plays around but not with otherchildren; start of toilet training

Early Stages of Life: Infancy to Childhood 7

Page 23: Deja review   behavioral science

What are the developmental milestones Gross motor: squats and recoversat 21 months of age? Fine/visual: builds five-block towers

Language: two-word combinationsSocial: toilet training

What are the developmental milestones Gross motor: walks well up and down at 24 months of age? stairs; jumps

Fine/visual: removes clothing; copies alineLanguage: 50-word vocabulary;stranger understands half ofspeechSocial: follows two-step commands;engages in parallel play

What are the developmental milestones Gross motor: throws ball over handat 30 months of age? Fine/visual: removes clothes; copies

linesLanguage: appropriate pronoun useSocial: knows first and last names

What are the developmental milestones Gross motor: pedals tricycle; goes at 3 years of age? up and down stairs with alternating

feetFine/visual: draws a circle; eats withutensilsLanguage: three-word sentences;uses plurals and past tense;stranger understands three-fourthsof speechSocial: group play; shares toys

What are the developmental milestones Gross motor: hops and skipsat 4 years of age? Fine/visual: catches ball; dresses alone;

copies a cross

Language: knows colors; counts to 10Social: imaginative play

What are the developmental milestones Gross motor: hops and skipsat 5 years of age? Fine/visual: ties shoes

Language: prints first nameSocial: plays cooperative games;understands rules and abides by them

8 Deja Review: Behavioral Science

Page 24: Deja review   behavioral science

Early Stages of Life: Infancy to Childhood 9

Table 1.1 Developmental Milestones

Gross Fine/Visual Motor Motor Language Social

1 month oldWhen With eyes Startles to Fixes on face

prone lifts tracks soundhead objects to slightly midline;

tight grasp

2 to 3 months oldSteadily holds Hands open Smiles Recognizes

head up; at rest responsively; parents; when prone coos reaches for lifts chest familiar up objects or

people

4 to 5 months oldRolls front to Grasps Orients to Laughs;

back and with both voice enjoys back to front; hands observing sits well environmentsupported

6 months oldSits well Transfers Babbles Recognizes

unsupported; hand to strangers sits upright hand; and has

reaches with stranger either hand anxiety

9 months oldCrawls, Uses pincer Says Waves

pulls to grasp; “dada/mama”; bye-bye; stand finger understands plays

feeds “no” pat-a-cake

12 months old Walks alone Throws, One to eight Imitates

releases words other actions;objects than “dada/ comes when

mama”; called;one-step cooperates commands with

dressing

(Continued)

Page 25: Deja review   behavioral science

10 Deja Review: Behavioral Science

Table 1.1 Developmental Milestones (Continued )

Gross Fine/Visual Motor Motor Language Social

15 months oldWalks Builds two- Uses four to Throws

backward; block eight words temper creeps towers; tantrums upstairs scribbles;

uses a cup

18 months oldRuns; kicks a Feeds self Points to body Plays around

ball with parts when but not withutensils asked; names other

common children;objects start of

toilettraining

21 months oldSquats Builds Two-word Toilet

and five-block combinations trainingrecovers towers

24 months oldWalks well Removes 50-word Follows two-

up and clothing; vocabulary; step down stairs; copies a stranger commands; jumps line understands engages in

half of speech parallelplay

30 months oldThrows ball Removes Appropriate Knows first

over hand clothes; pronoun use and last copies nameslines

3 years oldPedals Draws a Three-word Group play;

tricycle; circle; eats sentences; shares toysgoes up with uses pluralsand down utensils and past stairs with tense; alternating stranger feet understands

three-fourths of speech

Page 26: Deja review   behavioral science

Early Stages of Life: Infancy to Childhood 11

Table 1.1 Developmental Milestones (Continued )

Gross Fine/Visual Motor Motor Language Social

4 years oldHops and Catches Knows colors; Imaginative

skips ball; counts to 10 playdresses alone;copies a cross

5 years oldHops and Ties shoes Prints first Plays

skips name cooperative games;understandsrules and abides by them

ATTACHMENT

What is anaclitic depression? Physical, psychological, and socialproblems caused by prolonged removalof parental nurturing during ages 6 to12 months. This is reversed if nurturingis reintroduced.

What occurs without proper Failure to thrivemothering or attachment?

What occurs during failure Developmental retardationto thrive? Poor health and growth

High death rates, even with adequatephysical care

Toddlers who are hospitalized Separation from parents or care are most likely to fear what? providers more than bodily harm or

pain

Page 27: Deja review   behavioral science

In what age group is elective 7 to 11 yearssurgery best tolerated?

By age 3, how does separation from Separation from parents has no parents or care providers affect long-term negative effects on children. children? In fact, children at age 3 are able to

spend significant portions of the daywith other adults.

How do toddlers understand death? It is an incomplete understanding of themeaning of death and the child mayexpect a friend, relative, or pet to comeback to life.

At what age do children begin to At the age of 8 yearsunderstand the concept of death?

In what age group do children 7 to 11 yearsbegin to form relationships withother adults?

At what point should adopted At the earliest age possible, when they children be told that they are able to understand languageare adopted?

EARLY ADOLESCENCE (11 TO 14 YEARS)

How is the start of puberty marked? In girls: the onset of menstruation,beginning at age 11 to 12 yearsIn boys: the first ejaculation, occurring atage 13 to 14 years

How is the development through By the Tanner stagespuberty measured?

How many Tanner stages are there? There are five Tanner stages.

What are the three categories of 1. Male genitaliameasurement? 2. Female breasts

3. Pubic hair

12 Deja Review: Behavioral Science

Page 28: Deja review   behavioral science

CLINICAL VIGNETTES

A worried mother brings her 6-month-old son to your office for a checkup. Shestates that his highest achieved milestones are: he can recognize strangers, wavebye-bye, use a pincer-grasp, smile responsively, coo, crawl, and pull to stand. Inwhich developmental areas is he behind?

He is delayed in his language skills. By 6 months he should be babbling, and heshould have oriented to voice before that. His language skills are that of a 2-3-month-old. His gross motor, fine motor, and social skills are all at a 9-month level.

Early Stages of Life: Infancy to Childhood 13

Table 1.2 Tanner Stages of Development

Male Genitalia Female Breasts Pubic Hair

Stage IChildhood-sized Preadolescent breasts Absent.

penis, testes, and with elevation of the scrotum papilla only

Stage IIEnlargement of the Breast buds with Pubic hair is

testes and scrotum elevation of breast sparse and and papilla straight with

downy hair on labia/penis base.

Stage IIIPenis enlargement Breast and areola Pubic hair is

enlargement with curled, darker, ingle contour and coarse.

Stage IVScrotal skin Areola and papilla Pubic hair is

darkening and projection with adult- type hairrugations are separate contour limited to thepresent. (secondary mound). genital area.

Stage VAdult-sized and Mature breasts Pubic hair is

-shaped penis, adult-quantity testes, and scrotum and -pattern and

spreads to the thighs.

Page 29: Deja review   behavioral science

Your medical school classmate is a braggart; he loves to tell everyone howadvanced his 12-month-old daughter is. He claims she can walk backward, uses acup, builds a two-block tower, uses six different words, and throws the cutesttemper tantrums you’ve ever seen. How far advanced is she in developmentalmilestones?

She is at a 15-month level in all areas—fine motor, gross motor, language, andsocial.

A 35-year-old-male patient comes to your office worried that he never finishedgoing through puberty. His pubic hair is sparse, but it is dark and curled. His penishas enlarged, but there is no scrotal darkening or rogations. At what Tanner stageis he?

Tanner stage III

14 Deja Review: Behavioral Science

Page 30: Deja review   behavioral science

C H A P T E R 2

Middle Stages of Life:Adolescence toAdulthood

15

EARLY ADULTHOOD

What age range constitutes early 20 to 40 yearsadulthood?

What are the primary characteristics Role in society is defined.of this stage of life? Physical development peaks.

Sense of independence.

Which Erikson stage is prevalent Intimacy vs Isolationduring early adulthood?

Which life events often occur during Marriagethis stage? Having children

Occupation

What percentage of Americans is 60% to 70%married by age 30?

What are postpartum blues? Mild mood swings that may switchquite rapidly and occur after delivery—usually within a few days.

What is the percentage of women who 40% to 80% (very common andsuffer from postpartum blues? considered normal)

How long do the symptoms of They usually resolve within 2 weeks.postpartum blue last?

Page 31: Deja review   behavioral science

What can a physician do to assist Support and reassurancea patient with postpartum blues? Advise to get help with child care

Watch for development of postpartumdepression

What is the percentage of women who 5% to 25% (making it important tosuffer from postpartum depression? screen for!)

What are the diagnostic criteria for The same as a major depressive episode,postpartum major depression? but occurring after delivery.

Note: The Diagnostic and StatisticalManual of Mental Disorders, fourth edition(DSM-IV-TR) does not have a separatediagnosis for postpartum depression—only a “postpartum onset” modifier ofmajor depression.

What complications for the mother/ Failure to form bond with the babychild relationship may result from Thoughts of failure as a motherpostpartum depression?

Feeling unable to take care of babyInterference with child developmentSuicide of mother or homicide of infant

When do the symptoms of postpartum Within 4 weeks after delivery (DSM-IV-TRmajor depression occur? criteria for “postpartum onset”), though

some say up to 12 months after

What can a physician do to assist a Psychosocial therapypatient with postpartum major Antidepressant medicationdepression?

May require hospitalization if severe

What percentage of women suffer from 0.1% to 0.2%postpartum psychosis?

What are characteristics of postpartum Delusionspsychosis? Hallucinations

Mother may harm infant

When do the symptoms of postpartum 2 to 3 weeks after deliverypsychosis occur?

What can a physician do to assist a Antipsychotic medicationpatient with postpartum psychosis? Hospitalization

Note: This is a psychiatric emergency!

Which postpartum emotional reaction Postpartum bluesis considered normal?

16 Deja Review: Behavioral Science

Page 32: Deja review   behavioral science

MIDDLE ADULTHOOD

What age range constitutes middle 40 to 65 yearsadulthood?

What are the primary characteristics of Powerthis stage of life? Authority

Which Erikson stage is prevalent during Generativity vs Stagnationmiddle adulthood?

What term describes the decrease in Climacteriumphysiologic function that occursin midlife?

What physiologic functions decrease Endurancein men? Muscle strength

Sexual performance

What major reproductive change occurs Menopausein women?

What are the characteristics of Lack of menstrual cyclesmenopause? Hot flashes

Sexual Dysfunction/Vaginal Dryness

Osteoporosis (long-term)

What medical intervention has been Short-term estrogen replacementused to treat acute menopausal therapy (long-term not recommended)symptoms?

How long should contraceptive 1 yearmeasures continue after thelast menstrual period inmenopause?

Middle Stages of Life: Adolescence to Adulthood 17

Page 33: Deja review   behavioral science

CLINICAL VIGNETTES

A woman who delivered a healthy baby boy 4 days ago comes into your clinic. Shestates that she has been having mood swings—sometimes laughing, sometimescrying. She feels it is impacting her care of her baby.

What is the most likely diagnosis?

Postpartum blues

What should your treatment be?

You should screen her for postpartum depression and stay vigilant for thisdiagnosis—but reassure her that her symptoms are normal and will likely resolvewithin a couple of weeks. Also, recommend that she have someone help her withchild care. Be sure to distinguish this from postpartum depression—a commondisorder with more severe symptoms that should be treated more aggressively.

A 58-year-old woman complains of less interest in sex over the last few years. Shesays it is a combination of less desire and pain with intercourse. What is the mostlikely cause of the patient’s symptoms?

Postmenopausal women have a significant decrease in estrogen, leading to vaginalatrophy, dryness, and sexual dysfunction. Topical vaginal estrogen preparationsmay help, as can lubricants.

A 32-year-old married woman develops hallucination and delusions 2 weeks afterthe delivery of a healthy baby boy. She informs the physician that she has hadthoughts about harming the infant. The physician decides to hospitalize themother immediately and place her on antipsychotic medication. What postpartumreaction has the woman experienced?

Postpartum psychosis

18 Deja Review: Behavioral Science

Page 34: Deja review   behavioral science

C H A P T E R 3

Late Stages of Life:Aging, Death, andBereavement

19

AGING

What percentage of the US population 15%will be 65 years and older by theyear 2020?

Which is the fastest growing age group 85 years and olderin the United States today?

What is the average life expectancy in 77 yearsthe United States?

What factors within a population can Gender (females tend to live longer thanhave an effect on differences in life males; an average difference of 7 years)expectancy? Race (whites tend to live longer than

blacks)White female (80 years) > black female(74 years) = white male (74 years) >black male (66 years)

How are the life expectancies changing Males and African Americans are livingin regard to different ethnic groups? longer, thus the gap between them and

the longest living (white female) isdecreasing.

What physical changes are associated Sensory: impaired vision and hearingwith aging? Visceral: decreased pulmonary, renal,

and gastrointestinal functionExtremities: increased fat deposits,osteoporosis, and decreased musclemass and strength

Page 35: Deja review   behavioral science

What changes in the brain usually Less cerebral blood flowaccompany aging? Decreased brain weight

Enlarged ventricles and sulciIncreased amount of senile plaques andneurofibrillary tangles (also found inolder brains from patients who did notsuffer from Alzheimer disease [AD])

Psychological

Does an individual’s level of No. Slight decreases in cognitive abilityintelligence change throughout life? are usually compensated for by life

experience and wisdom—so called“crystallized intelligence.”

Which Erikson stage of development Ego Integrity vs Despair. During thisis characteristic of the last stage of life? time there is an evaluation of the choices

made in one’s life and a reconciliationof those choices with one’s values. Apositive evaluation leads to contentment,whereas a negative evaluation leads todiscontent.

Psychopathology

What is the most common psychiatric Major depressionillness of the elderly?

What factors can lead to depression in Loss of spouse, friends, and/or familythe elderly? Loss of prestige and income

Decline of healthRetirement

What common disease process involving Dementiacognitive decline may mimic depressionin the elderly?

What three methods can be used to 1. Pharmacotherapysuccessfully treat depression? 2. Psychotherapy

3. Electroconvulsive therapy (this isstill the most effective treatment fordepression)

What changes in sleep patterns occur Decreased quality of sleep with lessin the elderly? time in deep sleep and more time in

light sleepDecreased rapid eye movement (REM)sleep

20 Deja Review: Behavioral Science

Page 36: Deja review   behavioral science

What substance-use disorders are Alcohol-related disorderscommon in the elderly population butoften unidentified?

Do psychiatric drugs produce the same No. There are differences in drugeffects in the elderly as they do in bioavailability, metabolism, andyoung adults? response, and the elderly have a greater

susceptibility to side effects. Therefore,when you prescribe, you should startlow and go slow.

Longevity

What factors are associated with Family history of longevitylongevity? Continuation of occupational and

physical activityHigher educationSocial support system including marriage(only men benefit from marriage)

DYING, DEATH, AND BEREAVEMENT

Which author describes five stages of Elizabeth Kubler Rossdeath and dying, which may or maynot occur in consecutive order?

At what stage would a patient refuse Denialto believe he or she is dying and say,“You must be reading thewrong chart”?

At what stage would a patient become Angerupset with the hospital staff and say,“You should have made the diagnosissooner! It is your fault I am in thissituation”?

At what stage would a patient plead Bargainingwith a higher power for forgivenessand healing?

At which stage would a patient Depressionbecome tearful, withdrawn, andapathetic?

At what stage would a patient gain the Acceptanceunderstanding that life has come toan end?

Late Stages of Life: Aging, Death, and Bereavement 21

Page 37: Deja review   behavioral science

What is the difference between Bereavement tends to be less severebereavement and major depression? in symptoms and less pervasive

(ie, centers only around the loss).It is also self-limited and has a shorterduration.However, major depressive episodesmay be superimposed onto, orprecipitated by a major loss withbereavement.

How long does bereavement last? Normal bereavement should begin toresolve within a few months, thoughsome grief may never go away.A DSM-IV-TR diagnosis of majordepression should not be made withinthe first 2 months after a loss.

22 Deja Review: Behavioral Science

Table 3.1 Differences Between Bereavement (Normal Grief) and Major Depression(Abnormal Grief)

Bereavement Depression

Minor sleep disturbances Major sleep disturbances

Some feelings of guilt Feeling of worthlessness

Illusions Hallucinations and delusions

Expressions of sadness Suicidal thoughts and attempts

Minor weight loss (<3 lb) Major weight loss (>8 lb)

Good grooming and hygiene Poor grooming

Attempts to return to normal routine Few attempts to return to normalroutine

Severe symptoms subside in Severe symptoms continue for<2 months >2 months

Moderate symptoms subside in Moderate symptoms subside in<1 month >1 month

Tx: support groups, increased Tx: may include antidepressants,contact with physician, counseling, psychotherapy, antipsychotics,short-acting sedatives if needed and electroconvulsive therapy

Abbreviation: Tx, treatment.

Page 38: Deja review   behavioral science

CLINICAL VIGNETTES

You diagnose a 53-year-old man who has a long history of heavy smoking withmetastatic lung cancer. What stage of grief would he likely be in if he:

Says “I’ll be fine,” and lights up another cigarette.

Denial

Goes to church and prays to live another 3 years, just long enough to see hisdaughter graduate college.

Bargaining

Throws your reflex hammer at a nurse.

Anger

Says “I don’t care—there’s no point in living anyway.”

Depression

Starts to make plans for his hospice care and his surviving family.

Acceptance

An 83-year-old man lost his wife to cancer 1 month ago. He feels sad all the time,always thinking about his wife. He cries at night and cannot sleep. He has notbeen doing any of his normal activities. He also believes he hears his wifewhispering in his ear at night before bedtime. What is your diagnosis?

Bereavement—symptoms have only been present for 1 month after a major loss.If symptoms were to become more severe and last over 2 months, he would needto be screened for a mood disorder using DSM-IV-TR criteria.

A 76-year-old man finds that he has left little impact on the world. He did notspend much time with his children, he is divorced, and he is not happy with anyof the big decisions in his life. Which Erickson stage of life is this man most likelyexperiencing?

Ego Integrity vs Despair

Late Stages of Life: Aging, Death, and Bereavement 23

Page 39: Deja review   behavioral science

This page intentionally left blank

Page 40: Deja review   behavioral science

C H A P T E R 4

PsychoanalyticTheory

25

What are the two major theories of the 1. Topographic theory of the mind mind developed by Freud? (includes three parts):

a. Consciousb. Preconsciousc. Unconscious

2. Structural theory of the mind(also known as the tripartitetheory):a. Egob. Superegoc. Id

Which part of Freud’s topographic mind Preconsciouscontains information that one isunaware of, but can be accessedwith prompting?

Which part of the mind contains The conscious—this part of the mind hasthoughts of which a person is aware? no access to the unconscious. The

unconscious includes what we are notaware of such as sexual drives,aggressive drives, impulses, orfantasies.

How is the expression of the id The ego exerts the learned rules of theregulated? world over the id to control its overt

expression of primitive drives, such assexual urges and aggression. The egomanages and negotiates the drives fromthe id as well as moral restrictions (fromthe superego).

Which part of the mind helps a person The egomaintain relationships?

Page 41: Deja review   behavioral science

Which part of the mind incorporates The superego (our conscience)acquired moral and ethical concepts?

How does the ego deal with conflicts Defense mechanismsbetween the id and the superego?

What type of thinking is primary It is associated with pleasure andprocess? instincts. This process does not involve

logic or time and may not be linear.

Which part of the mind is controlled The id—this is not influenced by realityby primary process thinking? and focuses on pleasure and instinct

(primary processes).

What is psychotherapy? A treatment technique. This treatmentranges from strengthening useful egodefenses (in supportive therapy) tochallenging detrimental ego defensesand uncovering unconscious conflicts(in expressive and analyticpsychotherapy), improving self-esteem,and improving relationships.

What is the purpose of psychotherapy? To improve functionality. It helps tosupport defenses and functioning,reduce anxiety/stress, work on specificconflicts, improve coping skills, havehealthier relationships, and enhanceenjoyment/fulfillment in life.

What are the differences between Time:psychodynamic psychotherapy and • Psychoanalysis: 1 hour,psychoanalysis? 3 to 5 × per week for 3 to

8 years• Psychotherapy: 1 hour,

1 to 2 × per week for 2 to20 years

Level of therapist’s participation:• Psychoanalysis: The therapist mainly

listens to the patient who lies on thecouch and helps guide the patient,clarify, question, gently confront,encourage, interpret, analyze,and so forth.

• Psychodynamic: The therapistis more interactive while bothhe or she and the patient sit in chairsfacing each other, though thetherapist still plays a guiding role.

26 Deja Review: Behavioral Science

Page 42: Deja review   behavioral science

What are the three main 1. Free associationtechniques used in psychoanalysis 2. Dream interpretationsand psychodynamic 3. Transference interpretationspsychotherapy to uncoverthe unconscious?

Which technique is used to describe Free associationthe patient expressing “whatevercomes to mind”?

How does the unconscious mind Through dreams, slips of the tongue,manifest impulses, wishes, and forgetting significant thingsand fears?

What is the purpose of a defense Maintaining the ego or decreasingmechanism? anxiety and maintaining a sense

of self

What are the mature defense Humormechanisms? Altruism

SublimationSuppression

What are the immature defense Acting outmechanisms? Denial

DisplacementDissociationFixationIdentificationIntellectualizationIsolationProjectionRationalizationReaction formationRegressionRepressionSplitting

Which defense mechanism causes a Displacementpatient to take his uncomfortablefeelings toward an unacceptabletarget and aim it at a more acceptabletarget?

Which defense mechanism allows one Sublimationto use a socially acceptable way tocombat an unacceptable impulse?

Psychoanalytic Theory 27

Page 43: Deja review   behavioral science

How does sublimation differ from In displacement, the object or persondisplacement? receiving the negative attention is

“more tolerable” to the individual(kick dog instead of boss), while insublimation, the negative attentionis channeled into an activity that issocially acceptable (vigorousexercise to relieve stress causedby boss).

Which defense mechanism allows one Humorto find amusement in an otherwisedifficult situation?

If a patient uses intellectualization Obsessive-compulsive personalityand isolation of affect as ways to disorder (OCPD)deal with their discomfort, whatpersonality disorder mighthe have?

What defense mechanisms are highly Intellectualization (focus on facts aboutassociated with OCPD? painful things instead of the painful

things—eg, learning about treatmentsfor cancer when diagnosed rather thanfeeling grief over the diagnosis.)Rationalization (making excuses thatseem like a reasonable explanation—eg,I failed out of medical school becauseI didn’t really like biology anyway.)Isolation of affect (eg, I refuse to haveemotions about this topic becauseemotions are too difficult.)Reaction formation (eg, I can’t deal withthe fact that I hate you so I’ll give you agift instead.)

How does rationalization differ from Rationalization occurs when a personintellectualization? uses excuses to explain an uncomfortable

feeling related to an event or person.Intellectualization is when a persondefers to factual information in order todeal with or understand uncomfortablefeelings.

Which immature defense is used Splittingextensively by patients with borderlinepersonality disorder who cannotintegrate the good and bad aspectsof the same person?

28 Deja Review: Behavioral Science

Page 44: Deja review   behavioral science

Which defense mechanism leads a Suppressionpatient to deal with an uncomfortablesituation by “placing it on the backburner”?

How does repression differ from Repression is an immature defensesuppression? mechanism that occurs when the

unconscious causes us to “forget”painful information while suppressionis a mature defense mechanism thatallows us to consciously put off painfulinformation and “deal with it later” inorder to maintain our composure.

Which defense mechanism is associated Regressionwith a person dealing with stressfulsituations in a childlike manner?

In which personality disorder is Histrionicrepression, regression, andsomatization used the most?

Which defense mechanism allows us to Projectionplace our bad feelings about ourselvesonto others?

How does displacement differ from Displacement allows us to place ourprojection? negative emotion about someone else

onto another target, while projectionallows us to place our negativeemotions about ourselves onto anothertarget.

Which defense mechanism causes us Identificationto take on the positive and negativebehaviors of others?

Which personality disorders use Paranoid personality disorderprojection and denial as their primary Schizotypal personality disorderdefense mechanisms?

Antisocial personality disorderBorderline personality disorder

How does denial differ from splitting? In denial, patients ignore all the badaspects about something entirely (Bob’sdog died but he is still feeding himevery night). This is not a consciousaction. With splitting, patients cannotsee the positive and the negative at thesame time (their doctors are either“horrible” or “wonderful” but never“so-so”).

Psychoanalytic Theory 29

Page 45: Deja review   behavioral science

If a man “forgets” to return calls to all Avoidanceof the clients he dislikes today, whichdefense mechanism is at work?

Which two personality disorders Avoidant and dependent personalityprimarily utilize avoidance? disorders

If a man who recently embezzled Altruism—assisting others to avoidmoney donates funds to a homeless feeling bad about oneselfshelter, what defense mechanism ishe using?

Which defense mechanism causes an Acting outindividual to express unacceptablethoughts and feelings in a sociallyinappropriate manner?

Which defense mechanism allows one Denialto avoid a painful situation by acting asif it never happened?

What is the emotional reaction a patient Transference. It occurs when patientshas toward their physician? unconsciously reexperience relationships

from their past in their currentrelationship. This can occur in anyrelationship (not just with therapists orphysicians). It can be a useful tool intherapy.

What is the emotional reaction a Countertransference. It has two parts:physician has to a patient? 1. Feelings the physician has toward

the patient that are related to theirown past (like transference).

2. Feelings that a physician has towardthe patient that resembles how thepatient causes most people to feel(patient specific).

What are the merits of recognizing The physician’s countertransference cancountertransference? influence the way the patient is treated.

It is also a useful insight in psychotherapyto recognize how the patient makesothers feel about them.

30 Deja Review: Behavioral Science

Page 46: Deja review   behavioral science

31

Table 4.1

DefenseMechanism Description

Altruism Guilty feelings are alleviated by kindness to others

Humor Recognizing the amusing nature of an anxiety-provoking or adverse situation

Sublimation Channeling unacceptable impulses into acceptable actions

Suppression Voluntary blocking of an idea or feeling from conscious awareness

Acting Out Undesirable feelings and thoughts are expressed through actions

Denial Refusing to accept facts about an unpleasant reality

Displacement Transferring feeling about one thing to something else

Dissociation Detaching from oneself; emotionally or mentally

Identification Modeling one’s behavior after someone more powerful

Intellectualization Using facts and logic to avoid emotions

Projection Attributing one’s feelings to an external source

Rationalization Proclaiming logical reasons for actions actually performed for other reasons

Reaction formation Process whereby a warded-off idea or feeling is replaced by an unconsciously derivedemphasis on its opposite

Regression Returning to a less mature stage of development

Repression Involuntary blocking of an idea or feeling from conscious awareness

Splitting Belief that people are either all good or all bad

(Continued)

Imm

atu

reM

atu

re

Page 47: Deja review   behavioral science

32

Table 4.1 (Continued )

DefenseMechanism Example

Altruism Criminal makes large donation to charity.

Humor Nervous surgeon jokes about an upcoming procedure.

Sublimation Aggressiveness used to succeed in business ventures.

Suppression Choosing not to think about life stresses during work.

Acting Out A teenager punches a hole in the wall.

Denial An HIV-positive man adamantly states he is healthy.

Displacement Mother yells at her child because she is angry at her husband.

Dissociation Floating above one’s body during a traumatic event.

Identification An abused child becomes a bully.

Intellectualization A newly diagnosed cancer patient calmly describes the pathogenesis of their disease.

Projection A woman who fantasizes about another man thinks her husband is cheating on her.

Rationalization Saying a job was not important, after getting fired.

Reaction formation A patient with sexual thoughts enters a monastery.

Regression An adult clutching a sentimental teddy bear.

Repression Traumatic childhood events remain buried in the unconscious.

Splitting A patient says that all men are cold and insensitive but that women are warm and friendly.

Imm

atu

reM

atu

re

Page 48: Deja review   behavioral science

Psychoanalytic Theory 33

CLINICAL VIGNETTES

A middle-aged woman is in a grocery store. She goes to the bulk candy section andsees gummy bears—her favorite. Part of her mind says she should grab a handfuland stuff them in her mouth. Which part of the mind is telling her this?

The id

A man and a woman are in a car accident. Neither has insurance and both cars aretotaled, and both of them lose their licenses. The woman goes home and says“Well, at least I wasn’t driving a Ferrari—that would have been much moreexpensive.” The man says “I hit her because I was swerving to avoid somethingelse. I should have had my brakes balanced.” What defense mechanisms are theyusing and are they mature or immature defenses?

The woman is using humor—a mature mechanism. The man is using rationalization—an immature mechanism.

A medical student is mistreated by his professor, but later tells his fellow medicalstudents that the professor is “great”! What defense mechanism is he using?

Reaction formation

A medical student’s dog dies. When he is consoled by fellow students, he states,“He was really old and I was expecting him to go soon anyway. Most large dogsonly live until they are 10.” What defense mechanism is he using?

Intellectualization

A medical student fails the last anatomy exam after he studied 12 hours per day inthe corner of the library for 1 month. He tells his friend, “I think I failed becauseI didn’t study enough. Besides, I am not really going to use anatomy later on.”What defense mechanism is he using?

Rationalization

Page 49: Deja review   behavioral science

This page intentionally left blank

Page 50: Deja review   behavioral science

C H A P T E R 5

Learning Theory

35

What is behaviorism? It is a learning theory based on the ideathat behavior is a product of learningthrough association or reinforcement.

ASSOCIATIVE LEARNING VS NONASSOCIATIVE LEARNING

What is associative learning? Learning that occurs when a connectionor pairing is made between a particularstimulus and a particular response

What are associative learning processes? Classical conditioningOperant conditioning

What is nonassociative learning? Nonassociative learning describesbehavior change as a result ofpresentation of one stimulus repeatedly.It also describes learning which has noassociation with an end stimulus (suchas a reward or punishment).

What are nonassociative learning Observational learningprocesses? Habituation

Sensitization

Associative Learning

Imprinting

What is imprinting? Imprinting describes learning thathappens at a specific (usually early)developmental stage. It is usually rapidand is unrelated to the consequences ofthe behavior being learned. During thistime, the animal or person imitates thebehavior of another stimulus.

Page 51: Deja review   behavioral science

What is an example of imprinting? When birds follow the first thing theysee moving after they hatch. The criticaltime period is the few moments afterbirth and the association is made soquickly that the first object that theysee suitably moving is what they willfollow.

Classical Conditioning

What is classical conditioning? Classical conditioning is a way ofpairing a stimulus and response. Asdemonstrated by Pavlov, a novelstimulus (a ringing bell) can be pairedwith an unconditioned stimulus (food)to elicit an unconditioned or naturalresponse (salivation).Thus, if a bell (novel stimulus) isrung every time food (unconditionedstimulus) is presented to the dog,it will be conditioned to associatethe bell with food and will learn tosalivate (natural response) at thesound of the bell. This is theconditioned response.

36 Deja Review: Behavioral Science

UR

CR

US

CS

Bell

1. Normally, unconditioned stimulus leads to unconditioned response.2. Conditioned stimulus paired with unconditioned stimulus.3. Now conditioned stimulus also leads to the response —termed conditioned response.

Note: UR and CR are theaccurate terminology; theresponse itself is still the same.

US: Unconditioned stimulusCS: Conditioned stimulusUR: Unconditioned responseCR: Conditioned response1

2

2

1

3

3

Food Salivation

Salivation

Doctor’s office

Chemotherapy1

2

3

Nausea

Nausea

Medical examplePavlov’s example

Figure 5.1

Page 52: Deja review   behavioral science

What is the term describing a similar Stimulus generalization. The classicstimulus to the conditioned stimulus example was “Little Albert” who waseliciting the same response? conditioned to fear a white rat and was

subsequently afraid of other whitefuzzy things.

What is a medical example of classical Phobias are believed to be results ofconditioning? classical conditioning. For example,

Mary had a frightening experience on aship. She may generalize that fear so thateven the sight of a ship causes heranxiety.

What is extinction? It is the disappearance of the conditionedresponse if the conditioned andunconditioned stimuli are no longerpresented together.

Is it possible for the conditioned Yes, this is called spontaneous recovery.response to be paired with theconditioned stimulus after extinctionhas taken place?

What does learned helplessness mean? The effect of repeatedly pairing anadverse stimulus to the inability toescape, leading to thoughts that noefforts will be successful

What mood disorder may be explained It has been thought that this theory mayby the theory of learned helplessness? explain depression in humans. In this

theory, a person has tried repeatedly butunsuccessfully to control external events.The person then pairs any adverse eventto the inability to do anything aboutthem. The person then becomes hopeless,depressed, and apathetic.

Operant Conditioning

What is operant conditioning? It is the idea that a behavior is learnedbecause of the reward or punishmentassociated with it.

What are the different consequences Positive reinforcementin operant conditioning? Negative reinforcement

PunishmentExtinction

Learning Theory 37

Page 53: Deja review   behavioral science

What are reinforcers? Any event or stimulus that increases thelikelihood of the behavior occurringagain

What is positive reinforcement? Presentation of a rewarding stimulusafter a certain behavior is performed.For example, parents may reward theirchildren with ice cream (positivereinforcer) when their rooms are clean,thus increasing the likelihood that thechildren will clean their rooms.

What is negative reinforcement? Removal of an aversive stimulus after acertain behavior is performed. Forexample, parents may exempt thechildren that clean their rooms fromhaving to take out the trash, thusincreasing the likelihood that thechildren will clean their rooms.

What type of reinforcement increases Positive and negative reinforcementthe likelihood of a behavior occurringagain?

What will decrease the likelihood Punishment (note the contrast betweenof a behavior occurring? negative reinforcement and punishment)

What is shaping? Shaping is learning that occurs when aperson is rewarded for a behaviorwhich is similar to a desired behavior.Subsequently, only behavior which ismore and more similar to the particulardesired behavior is rewarded.

When is shaping used? It is a progressive modification ofbehavior which occurs by reinforcementof behavior which is close to the desiredoutcome.

What is the definition of punishment? Presentation of an aversive stimulus toreduce the likelihood of an unwantedbehavior occurring

What is the definition of extinction Extinction is the disappearance of ain operant conditioning? certain behavior when the reinforcement

is no longer present.

What is an example of extinction? A rat that is initially trained to press abar if rewarded with food will quicklycease to press the bar if food is nolonger obtained by the behavior.

38 Deja Review: Behavioral Science

Page 54: Deja review   behavioral science

What are the different schedules of There are five types of reinforcementreinforcement? schedules:

1. Continuous: every time behavior isperformed

2. Fixed ratio: set number of times3. Variable ratio: random number of

times4. Fixed interval: set amount of time5. Variable interval: random amount of

time

Which reinforcement shows the fastest Continuousextinction when reinforcement istaken away?

What is variable ratio reinforcement? Reinforcement is given at a variabletime interval after the behavior isperformed. This type of reinforcementshows the slowest extinction when thereinforcement is taken away. Slotmachines are an example of variableratio reinforcement.

What is the difference between Classical conditioning refers to behaviorsclassical and operant conditioning? learned by association of stimuli and

responses whereas operant conditioningrefers to behaviors learned by thereward and reinforcement associatedwith them.

NON-ASSOCIATIVE LEARNING

What is observational learning? In observational learning, the observer’sbehaviors change based upon observingthe model’s behaviors. The consequencesof the model’s behaviors, whether theyare positive or negative, have an effecton the observer’s behaviors.

If a person observes others and then Modeling. Modeling is a type ofimitates their behavior, what is that observational learning. Compared tobehavior called? operant learning it is a more efficient

and faster type of learning. Modeling isuseful in acquiring new skills.

What four aspects are needed in order 1. Attention to the modelfor a person to be able to model? 2. Retention of details

3. Motor reproduction4. Motivation and opportunity

Learning Theory 39

Page 55: Deja review   behavioral science

Where in medicine is modeling used? The common saying, “see one, do one,teach one,” is a description of modelingin learning how to do medicalprocedures.

Can modeling have a negative outcome? Yes. Modeling may occur when a childmodels the actions of a parent with aparticular phobia and hence also acquiresthe same phobia or perpetuation ofabuse by an abused person.

Can modeling have a positive outcome? Yes. Modeling may involve othertypes of learning, eg, seeing a rolemodel/mentor’s behavior achievea positive result would then act asa positive reinforcer (which wouldbe an example of operantconditioning).

What is habituation? Habituation occurs when stimuluspresentation results in decreasedresponsiveness.

What is an example of habituation? You may notice the hum of the airconditioner when it first comes on,but due to habituation, yourawareness of that continual humwill decrease and you can focuson your studies.

How is habituation used in medicine? Habituation is used to overcomephobias. Some of the specific techniquesusing habituation are flooding andsystemic desensitization.

How do you describe flooding? It is excessive presentation of thestimulus to achieve quick habituationby preventing escape and forcinga reduction in the associatedbehavior.

What is an example of flooding? One could force an individual withan obsession about germs to toucha toilet.

How do you describe systematic In systematic desensitization, thedesensitization? patient is gradually exposed to

anxiety-producing situations whilesimultaneously teaching relaxation oranxiety-reducing techniques.

40 Deja Review: Behavioral Science

Page 56: Deja review   behavioral science

What is sensitization? Sensitization occurs when stimuluspresentation results in increasedresponsiveness and/or generalization ofresponse to other stimuli.

What is an example of sensitization? Joe is alone in a dark house when hehears a sudden loud noise. He suddenlybecomes more aware of every littlesound in the house.

Intelligence Tests

Which term is used to describe the Intelligenceability to learn, understand, or to dealwith new or trying situations?

Which term did Binet use to describe Mental agethe average intellectual age of peoplewith a specific chronological age?

Which scale is used to determine a Stanford-Binet scaleperson’s IQ?

How is the IQ calculated? (Mental age/chronological age) × 100

If a person has an IQ of 100, what does Mental and chronological age arethat indicate? equivalent.

If a person has an IQ <70, what does Mental retardationthat indicate?

What happens to the IQ throughout the Remains generally stablecourse of a person’s life?

Which IQ range indicates that a person 90 to 109has an average intelligence?

Which IQ range indicates borderline 71 to 89to low average intelligence? 50 to 70

Which IQ range indicates moderate 35 to 55mental retardation?

Which IQ range indicates severe 20 to 40mental retardation?

Which IQ range indicates profound <20mental retardation?

Learning Theory 41

Page 57: Deja review   behavioral science

CLINICAL VIGNETTES

An 8-year-old girl with acute myelogenous leukemia (AML) gets weeklychemotherapy treatments. The chemotherapy makes her very nauseated, but hermother makes up for it by taking her to an ice cream shop by the hospital. Herfavorite flavor is maple nut. After the first month of treatment, she will no longereat ice cream as it causes intense nasusea.

What are the unconditioned stimulus (US), unconditioned response (UR),conditioned stimulus (CS), and conditioned response (CR) in this situation?

US: chemotherapy; UR: nausea; CS: ice cream; CR: nausea

The patient also refuses to eat pancakes with maple syrup. What is thisphenomenon?

Ice cream has been paired with chemotherapy here. The patient has also generalizedthe maple ice cream to the syrup, eliciting the same nausea.

After the patient’s AML is in remission and her treatment is over, her parentsslowly reintroduce her to eating progressively larger amounts of ice cream whilelistening to her favorite calming music. What is this process called?

Her parents are later using the process of systematic desensitization to unpair theCS with the US.

A rat is being conditioned to press a lever in its cage, but not the button next to it.Name the operant conditioning consequence or phenomenon represented by eachstatement:

The rat is given a piece of cheese when it gets close to the lever. Subsequentpieces of cheese are given as it gets closer and closer, finally resulting in a bigpiece when it finally presses it.

Shaping

The rat receives a piece of cheese every time the lever is pressed.

Positive reinforcement

The rat cage floor is electrified, but pressing the lever deactivates it.

Negative reinforcement (note the behavior will increase)

The rat presses the button and a loud and terrifying noise is emitted.

Punishment (note the behavior will decrease)

The rat continues to press the lever, but nothing happens. He eventually stopspressing the lever.

Extinction

The floor of the cage is electrified by the cruel scientist, and no amount of leveror button pressing will turn it off. The rat gives up.

Learned helplessness

42 Deja Review: Behavioral Science

Page 58: Deja review   behavioral science

C H A P T E R 6

Sleep Science andDisorders

43

NORMAL SLEEP

What are the normal stages of sleep? REM (rapid eye movement) and NREM(nonrapid eye movement). NREM isdivided into four stages: 1, 2, 3, and 4.

REM

Stage 1

Stage 2

Stage 3

Stage 4

Awake

Time asleep

Sle

ep s

tage

Figure 6.1

What is slow wave sleep? Slow wave sleep occurs during stages 3and 4 of NREM sleep. It is also knownas delta sleep and is the deepest portionof sleep. Electroencephalogram (EEG)shows delta waves, which are thelowest frequency waves.Delta waves = Deep sleep

Page 59: Deja review   behavioral science

What waveforms are seen in Beta waves. These are of highestREM sleep? frequency.

Where else are beta waves seen? They are found over the frontal lobes ina person who is awake with their eyesopen. These are associated with aperson who is alert and activelyconcentrating. In a person who is awakewith their eyes closed, alpha waves areseen typically over the occipital andposterior parietal lobes.

What waveforms are associated with Stage 1 (light sleep) is associated withStages 1 and 2 of NREM? theta waves and Stage 2 (deeper sleep)

is associated with sleep spindles andK-complexes.

On average, how much time does a REM: 25%normal adult spend in each stage NREM: 75%of sleep? 1. Stage 1: 5%

2. Stage 2: 45%3. Stage 3: 25%4. Stage 4: 25%

What is REM latency? REM latency is the length of time afterfalling asleep before REM sleep occurs.

What is the length of REM latency REM latency in an adult is approximatelyin an adult? 90 minutes. The REM sleep cycle then

repeats itself approximately every90 minutes thereafter.

Besides rapid eye movements, what Pulse, respiration, blood pressure, andother physiologic changes occur in brain oxygen use increase. There isREM sleep? penile/clitoral erection, dreaming, and

decreased skeletal muscle tone.

How is this different from the In NREM, blood pressure, pulse, andphysiologic changes in NREM sleep? respiration are slow. There may also be

intermittent limb movements.Tip: This makes sense—if your bloodpressure (BP) and pulse are up andyour genitals are aroused, NREM sleepwouldn’t be very restful!

Which neurotransmitter usually Serotonin. It is released from the dorsalinitiates sleep? raphe nucleus and is a derivative of

tryptophan. It increases total sleep timeand slow wave activity.Tip: Turkey is high in tryptophan—thisis why you get sleepy after a bigThanksgiving dinner.

44 Deja Review: Behavioral Science

Page 60: Deja review   behavioral science

Which neurotransmitters are involved Acetylcholine (ACh) from the basalin REM sleep? forebrain and norepinephrine (NE) from

the locus ceruleus. ACh increases REMsleep and NE decreases it.

How does REM change with age? Time spent in REM decreases with age.

What effect does dopamine have on the Dopamine increases wakefulness. Thus,sleep cycle? antipsychotics, which block dopamine,

can result in increased sedation.Stimulants, which increase both NE anddopamine, promote wakefulness.

ABNORMAL SLEEP

Approximately how many adults About one in threeexperience sleep disorders every year?

What is the most common type Insomnia, which includes problemsof sleep disorder? initiating and maintaining sleep.

What is a primary sleep disorder? Sleep disturbances that arise fromendogenous sources, not from substanceuse, medical problems, or otherpsychiatric problems. Primary sleepdisorders are divided into two majorcategories:1. Parasomnias2. Dyssomnias

What is the difference between Dyssomnias are due to dysfunctionaldyssomnias and parasomnias? sleep regulation characterized by

problems initiating or maintainingsleep, or excessive daytime sleepiness.Parasomnias involve abnormal behaviorsor physiologic events during sleep,rather than abnormal functioning of theusual mechanisms of sleep. These includesleep terror disorder, sleepwalkingdisorder, and nightmare disorder.

Name the five major dyssomnias 1. Primary insomnia2. Primary hypersomnia3. Narcolepsy4. Breathing-related sleep disorder5. Circadian rhythm sleep disorder

How long must you have problems At least 1 monthwith insomnia before primaryinsomnia can be diagnosed?

Sleep Science and Disorders 45

Page 61: Deja review   behavioral science

What interventions other than Set a regular bedtime, abstain frommedications can be useful caffeine and alcohol, use the bed onlyin insomnia? for sleep and sex, and avoid daytime

naps and strenuous exercise or largemeals just before bedtime. Collectively,this is referred to as sleep hygiene.

What is the subtype of recurrent Kleine-Levin syndromeprimary hypersomnia associated withobesity, impulsivity, hyperphagia,hypersexuality, and disorganizedthought called?

Other than daytime sleepiness and Cataplexy: sudden loss of muscle“sleep attacks,” what symptoms are tone associated with strong emotionsclassically associated with narcolepsy? Hypnagogic and hypnopompic

hallucinations: REM intrusions thatoccur during the transition periodbetween sleep and wakefulness(Hypnagogic symptoms occur whengoing to sleep and hypnopompic occurwhile waking up.)Sleep paralysis: inability to move justbefore going to sleep or awakening

Which class of drugs is normally used Stimulants, eg, Ritalinto treat narcolepsy?

Why are people with breathing-related During the night they frequently stopsleeping disorder chronically sleepy breathing and then are awoken byduring the day? hypoxia. These frequent arousals

prevent the patients from getting deep,restful sleep.

What is the most common cause Obstructive sleep apnea (OSA)of breathing-related sleep disorder Treatment: continuous positive airwayand how is it treated? pressure (CPAP) and possibly removal

of tonsils and adenoids (ideally weightloss would be primary interventionin the obese.)

In a patient with excessive fatigue, Loud snoring and periods of timewhat might the patients’ bed partner where the patient appears to stoptell you about their sleep that might breathinglead you to suspect obstructive sleepapnea (OSA)?

What are the complications from Hypertension, pulmonary hypertension,untreated OSA? and increased all-cause mortality,

cardiovascular disease, andcerebrovascular disease

46 Deja Review: Behavioral Science

Page 62: Deja review   behavioral science

In a person with sleep problems related You risk further compromising theto OSA, why might you avoid patient’s ventilation by respiratorybenzodiazepines? depression.

What are the three most common causes 1. Delayed sleep phase—“night owls,”of a circadian rhythm sleep disorder? more common in adolescents and

tends to improve with age2. Jet lag—typically resolves over

several days3. Shift work—eg, working the

night shift

During which phase of sleep would REMyou expect nightmares to occur in?

What are the similarities between sleep Both are more common in children andterrors and sleepwalking? may involve semicomplex to complex

motor behaviors. Patients tend to beamnestic for both and in adulthood theyare equally prevalent in men andwomen (1% prevalence of sleep terrorsand 2% prevalence of sleepwalking).Both occur in slow wave sleep.

How could you clinically differentiate Sleep terror has strong component ofsleep terror from sleepwalking? autonomic arousal and fear, often

beginning with a terrified scream, and alesser element of semipurposeful motorbehaviors. Sleepwalking has minimalautonomic arousal/fear and motorbehaviors are usually more complex.

What is the treatment of sleep terrors Children will usually grow out of them.and sleepwalking? Benzodiazepines can be helpful for

adults, as can scheduled awakeningsand environmental control.

OTHER SLEEP CHANGES

What polysomnogram (PSG) findings Reduced slow wave sleep (less deltaare characteristic of major depression? waves), frequent nighttime awakenings,

increased sleep latency (time untilfalling asleep), short REM latency(REM cycle starts sooner than normal90 minutes onset), and early morningwaking.Tip: Low serotonin is associated withdepression; therefore you would expectreduced total sleep and slow wave sleep.

Sleep Science and Disorders 47

Page 63: Deja review   behavioral science

A common feature of Alzheimer is Decreased slow wave and REM sleepreduced ACh. Given this, what PSGchanges are seen in an individualwith Alzheimer?

CLINICAL VIGNETTES

A 51-year-old obese man presents to your clinic because of fatigue. He states thathe is sleepy all the time and will often fall asleep at inopportune times—atmeetings, during lunch, and even behind the wheel. He says that he goes to bed at10 PM and wakes up at 6 AM every morning. He denies any substance use. Whatshould he be screened for?

Sleep apnea. Obese patients are at increased risk for this and can often present asdaytime sleepiness as sleep is not restorative. He should be evaluated with a sleepstudy. First-line treatment is CPAP.

A 32-year-old woman is being evaluated because of strange “spells”. She oftenseems to “pass out” when she is startled or when she laughs quite hard. Whatsleep disorder is she most likely suffering from?

Narcolepsy. The described phenomenon is cataplexy.

An 83-year-old woman presents to your office because she feels as though she isspending less time getting a restful sleep at night. She has heard from her friendsthat there are changes in sleep that occur with aging. What stages of sleep tend todecrease with age?

Both REM and later stage deep sleep (3 and 4) decrease in the elderly, as well as adecrease in overall quantity of sleep.

48 Deja Review: Behavioral Science

Page 64: Deja review   behavioral science

C H A P T E R 7

Sexuality

49

SEXUAL DEVELOPMENT

Which sex is the default pattern for Femalesexual development?

Gonad differentiation is dependent Y chromosomeupon the presence of whichchromosome?

Which gene present on the SRY Gene, which makes testis-Y chromosome influences gonad determining factordevelopment?

Which duct system present in male Wolffian duct systemembryos helps form genitalia?

Which duct system in female embryos Mullerian duct systemhelp form female genitalia?

Which organ secretes hormones that Testesdirect the differentiation of maleinternal and external genitalia?

How does exposure to different levels It causes gender differences in certainof hormones during prenatal life areas of the brain.influence humans?

What term describe an individual’s Gender identitysense of being male or female?

At which age(s) does gender awareness 2 or 3 years of agebecome evident?

What term describes the expression Gender roleof gender identity in society?

What term describes the conflict people Gender identity disorderexperience when they feel as if theywere born as the wrong gender?

Page 65: Deja review   behavioral science

According to the DSM-IV-TR No, it is a normal variant.is homosexuality a dysfunctionof sexual expression?

What is the difference between sex Sex refers to the genetics of theand gender? individual (XX or XY), gender is

what a person identifies as(male or female).

What type of behavior in childhood Cross-gender behavior → strongermay be predictive of later homosexual correlation in malesorientation?

Which hormones have been shown Prenatal hormones → low levels ofto influence sexual orientation? androgens in males and high levels of

androgens in females

Which evidence has been reported, Higher concordance rate observed inwhich indicates that genetics plays monozygotic twins than in dizygotica role in homosexuality? twins

On which chromosome have genetic X chromosomemarkers been found in homosexuals?

SEXUAL DEVELOPMENT AND PHYSIOLOGIC ABNORMALITIES

Which disorder is characterized by Androgen insensitivity which is alsocells that are not responsive to called testicular feminizationandrogens and testicles that mayappear as inguinal or labial masses?

What is the genotype of a person with XYandrogen insensitivity?

What is the phenotype of a person with Femaleandrogen insensitivity?

Which disorder is characterized by an Congenital adrenal hyperplasiaadrenal gland that is unable to producethe proper amount of cortisol whichleads to a significantly increasedandrogen secretion?

What is the genotype of a person with XXcongenital adrenal hyperplasia?

What is the phenotype of a person with Female with genitalia that are masculinecongenital adrenal hyperplasia?

50 Deja Review: Behavioral Science

Page 66: Deja review   behavioral science

What is the sexual orientation of 33% Homosexualof patients with congenital adrenalhyperplasia?

Which disorder is characterized by a Turner syndromeshort stature, webbed neck, andstreak ovaries?

What is the genotype of a person with XOTurner syndrome?

What is the phenotype of a person with FemaleTurner syndrome?

HORMONES AND THEIR INFLUENCE ON BEHAVIOR

Which hormone may be decreased by Testosteronean increase in stress?

Which three hormones commonly used 1. Androgen antagonistsin medical treatment of conditions such 2. Estrogensas prostate cancer decrease androgen 3. Progesteroneproduction and in turn reduces sexualinterest and behavior?

Which hormone is believed to play the Testosteronemost important role in sex drive inboth genders?

Which hormone may decrease sexual Progesterone → this hormone is in manybehavior and interest in women? oral contraceptives.

SEXUAL RESPONSE CYCLE

What are the four stages in the sexual 1. Excitementresponse cycle developed by Masters 2. Plateauand Johnson? 3. Orgasm

4. Resolution

What is the primary characteristic of Penile erectionthe excitement stage in men?

What are characteristics of the Clitoral erectionexcitement stage in women? Vaginal lubrication

Labial swellingUterus raises in pelvic cavity → tentingeffect

Sexuality 51

Page 67: Deja review   behavioral science

What characteristics of the excitement Nipple erectionstage are common to men and women? Elevation of blood pressure, pulse, and

respiration

What are characteristics of the recovery Refractory period in which stimulationstage in men? is not possible

What happens to the refractory period Increasesof men as they age?

What is the primary characteristic Minimal or no refractory periodof the recovery stage in women?

What are characteristics of the refractory Physiological systems return to theirperiod that are common to both men prestimulated states (eg, cardiovascular,and women? respiratory)

Muscle relaxation

SEXUAL DYSFUNCTION

Which term(s) describe problems in Sexual dysfunction stages of the sexual response cycle?

Which disorder is characterized by Dyspareunia pain associated with sexual intercourse?

Which gender is most likely to Femalesexperience dyspareunia?

Which disorder is characterized by Female sexual arousal disorder → occursinability to maintain vaginal lubrication in approximately 20% of womenthroughout the duration of a sexual act?

Which disorder is characterized by Hypoactive sexual desire decreased interest in sexual activity?

Which disorder is characterized by Male erectile disorder also calledproblems with maintaining erections? impotence

Which disorder is characterized by an Orgasmic disorder → this disorder caninability to achieve an orgasm? be lifelong or acquired.

Which disorder is characterized by Premature ejaculation anxiety and ejaculation beforea man desires?

Which stage of the sexual response Plateau phase → absent or reduced cycle is affected by prematureejaculation?

52 Deja Review: Behavioral Science

Page 68: Deja review   behavioral science

Which disorder is characterized by Sexual aversion disorder avoidance or aversion to sexualactivity?

Which disorder is characterized by Vaginismus painful muscular spasms in the outerone-third of the vagina making pelvicexamination or sexual intercoursedifficult?

Which behavioral treatment stimulates Sensate-focus exercise a person’s senses during sexualactivity to reduce the pressure oneexperiences when trying to achievean erection or orgasm?

What is the primary goal of the squeeze To treat premature ejaculationtechnique?

Which segment of the nervous system Parasympatheticis used to initiate an erection?

Which segment of the nervous system Sympatheticis used to initiate an ejaculation?

What is a mnemonic to remember this? Point and Shoot

Which behavioral technique(s) are used Hypnosisto reduce anxiety associated with Relaxation techniquessexual performance?

Systematic desensitization

Which drug is used to treat erectile Sildenafil citrate (Viagra)dysfunction by blocking thephosphodiester-5 (PDE-5) enzymethereby inhibiting cyclic guanosinemonophosphate (cGMP)?

What is the role of cGMP in sexual It is a vasodilator which allows anstimulation of the penis? erection to persist.

Which drug is used to increase the Apomorphine (Uprima) availability of dopamine in the brainin patients with erectile disorderand female arousal disorder?

What injection method is used to treat Intracorporeal injection of vasodilatorserectile dysfunction?

Which vasodilators are commonly Papaverineinjected in intracorporeal injection? Phentolamine

Sexuality 53

Page 69: Deja review   behavioral science

PARAPHILIAS

What term describes the use of Paraphiliaunusual objects of sexual desire orunusual sexual activities?

Which paraphilia is most common and Pedophilia → perpetrator must be atis characterized by a person who least 16 years of age and at least 5 yearsachieves sexual gratification from older than victimchildren <14 years of age?

Which paraphilia is characterized by Exhibitionismexposing genitals to strangers?

Which paraphilia is characterized by Fetishismachieving sexual gratification frominanimate objects?

Which paraphilia is characterized by Frotteurismrecurrent urges to rub against or touch a noncompliant person in asexual manner?

Which paraphilia is characterized by Necrophiliaa sexual gratification with corpses?

Which paraphilia is characterized by Sexual sadism causing humiliation and physicalsuffering to achieve sexualgratification?

Which paraphilia is characterized by Sexual masochism being the recipient of humiliation andphysical suffering to achievesexual gratification?

Which paraphilia is characterized by Transvestic fetishism achieving sexual gratification bywearing women’s clothing?

Which paraphilia is characterized by Voyeurism watching others engage in sexualactivity and undress?

INFLUENCE OF MEDICAL CONDITIONS ON SEXUALITY

What effect does a myocardial Decreased libidoinfarction have on sexual activity? Erectile dysfunction

54 Deja Review: Behavioral Science

Page 70: Deja review   behavioral science

Which postmyocardial infarction Patients who can tolerate increases inpatients can resume sexual activity? heart rate from 110 to 130 bpm

Which problem is common in Erectile dysfunction diabetic men?

What are the two main causes of 1. Diabetic neuropathyerectile dysfunction in diabetics? 2. Vascular changes

What is the primary treatment for Metabolic control monitored byerectile dysfunction in diabetics? hemoglobin A1c level

What effect does spinal cord Decreased fertilitydysfunction have on sexual Erectile dysfunctionfunctioning in men?

Orgasmic dysfunctionReduced testosterone levelsRetrograde ejaculation into the bladder

What effect does pregnancy have on Decreased sex drive → most common.sexual functioning? Increased sex drive and pelvic

vasocongestion may occur.

During what time period prior to 4 weeks before expected deliverypregnancy should a womancease sexual activity?

EFFECTS OF DRUGS AND NEUROTRANSMITTERS ON SEXUALITY

Decreased availability of which Dopamine (eg, chlorpromazine,neurotransmitter(s) causes a decrease haloperidol)in erection? Norepinephrine β (eg, propranolol,

metoprolol)

Increased availability of which Serotonin (eg, fluoxetine, sertraline,neurotransmitter(s) causes a trazodone)decrease in ejaculation and orgasm?

Increased availability of which Dopamine (eg, levodopa)neurotransmitter(s) causes an increase Norepinephrine in α2 in the peripheryin erection? (eg, yohimbine)

Which drug(s) of abuse causes Alcoholincreased libido with acute use? Marijuana

Which drug(s) of abuse causes Amphetaminesincreased libido? Cocaine

Sexuality 55

Page 71: Deja review   behavioral science

Which drug(s) of abuse causes erectile Alcohol → with chronic usedysfunction due to increased estrogenavailability as a result of liver damage?

Which drugs(s) of abuse causes reduced Marijuana → with chronic usetestosterone levels in men and loweredpituitary gonadotropin levels in women?

Which drug(s) of abuse causes reduced Heroinlibido and inhibited ejaculation? Methadone

CLINICAL VIGNETTES

A news program reports on a man who is causing problems in the local subway.As the doors open he rubs up against unsuspecting victims and then runs away.What is the name for this disorder?

Frotteurism

A 32-year-old male who is happily married does not become sexually arousedunless his wife wears a specific black negligee. He has always had an obsessionwith objects to give him sexual gratification. Which sexual paraphilia does thispatient have?

Fetishism

A 25-year-old female experiences intense painful vaginal spasms whenever shegoes to get a pelvic examination and engages in sexual intercourse. She has begunpsychological counseling to treat her condition. Which type of sexual dysfunctiondoes she suffer from?

Vaginismus

56 Deja Review: Behavioral Science

Page 72: Deja review   behavioral science

C H A P T E R 8

Abuse and Aggression

57

CHILD AND ELDER ABUSE AND NEGLECT

What are the primary types of child Physicaland elder abuse? Sexual

Emotional

What are the primary traits of a Low socioeconomic statuschild abuser? Young parents

Substance abuseSocial isolationStress in the householdHistory of victimization by spouse orcaretaker

What traits in a child make them more Child that has already been abusedlikely to be abused? Low birth weight, premature infant

Learning disability or language disorderAdopted child or stepchildrenHyperactive

What is the most common age range <3 years of ageof children that are abused?

What are key signs of child neglect? Lack of proper nutritionPoor personal hygiene

What are common sites of bruises on Lower back and buttocks → areas nota victim of child abuse? commonly injured during normal

activities

What type of marks might one see on a Linear bruises and marks, often atvictim of child abuse? multiple stages of healing

What are characteristics of the fractures Fractures at different stages of healingon a victim of child abuse? Spiral fractures → due to twisting limbs

08-Quinn_Ch08_p057-064.qxd 3/31/10 3:01 PM Page 57

Page 73: Deja review   behavioral science

What are the two primary burn types 1. Cigarette burnson victims of child abuse? 2. Immersion burns on legs, feet, or

buttocks → child is immersed inscalding water

What are other warning signs for child History inconsistent with level of child’sabuse/non-accidental trauma? motor development, bite marks,

and any facial trauma or bruising

What is the classic physical finding Retinal hemorrhageof shaken baby syndrome?

What type of cultural healing practices Cupping (use of heated cups to suctionmay leave markings that are important out illness)to distinguish from child abuse? Coining or Spooning (rubbed on body

leaving linear marks)

What are the primary traits of an Low socioeconomic statuselder abuser? Social isolation

Substance abuseClose relationship to the abused (eg,spouse, offspring) → person with whomthe elder lives and receives financialsupport from

What are the primary traits of an elder Some decline of mental functioningwho experiences elder abuse? (eg, dementia)

Economical or physical dependence onothersNot likely to report injuries as abuse →will state that they injured themselves

What are key signs of elder neglect? Lack of proper nutritionPoor personal hygieneLack of proper medication or healthaids (eg, prescription drugs, dentures,cane)

What are characteristics of the bruises Bilateral arm bruises from beingseen on a victim of elder abuse? grabbed

What is the annual incidence of More than 250,000 caseschild sexual abuse cases in theUnited States?

Has the likelihood of reporting child Increasedsexual abuse increased or decreasedwhen compared with the past?

58 Deja Review: Behavioral Science

08-Quinn_Ch08_p057-064.qxd 3/31/10 3:01 PM Page 58

Page 74: Deja review   behavioral science

Which gender is more likely to report Girls → 25% will report vs 12% boyssexual abuse during their lifetime?

Which gender is most likely to Malesbe the perpetrator of childsexual abuse?

Will the perpetrator of a child sexual Relative or acquaintance (eg, father,abuse more likely be a stranger or uncle, friend of the family, etc)someone the child knows?

What are some traits of child sexual Interpersonal relationship problemsabusers? (eg, marriage problems)

History of substance abuseMay have a history of pedophilia

What is the primary age range of 9 to 12 years of agechildren who are victims ofsexual abuse?

What emotions is the victim of a child Guiltsexual abuse likely to experience? Shame

Fear of abuser’s response if he or shenotified someone else of his or herexperience

What are the common physical signs Sexually transmitted infectionof child sexual abuse? (eg, human papillomavirus [HPV],

herpes, chlamydia)Recurrent urinary tract infection(UTI)Genital or anal injuryNote: Physical signs may be absent in avictim of child sexual abuse.

What are common psychological signs Inappropriate knowledge about sexualof child sexual abuse? events out of proportion for a given age

rangeExcessive initiation of sexual activitywith peers

What are common physical signs of Genital bruisingelder sexual abuse? Vaginal bleeding in women

What types of emotional abuse do Lack of caregiver attention and lovechildren experience? Physical neglect

Caregiver rejection

Abuse and Aggression 59

08-Quinn_Ch08_p057-064.qxd 3/31/10 3:01 PM Page 59

Page 75: Deja review   behavioral science

What types of emotional abuse do Neglect of needed care (eg, medical,elders experience? hygiene, etc)

Economic exploitation

How many cases of child and elder 1,000,000 cases eachabuse are reported annually?

What is the physician’s responsibility By law, the physician must report theif they suspects child or elder abuse? case to appropriate social service

agency.

DOMESTIC PARTNER ABUSE

What are primary findings seen Broken bonesin women who are victims of Bruisesdomestic violence?

Blackened eyes

What factor can greatly increase the If the abused person leaves the abuser.likelihood of an abused person beingkilled by their abuser?

What is the primary gender of the Maleperpetrator of domestic violence?

What are the characteristics of the Substance abuseabuser in a domestic violence situation? Angry

Threatens to kill the abusedApologetic after abuse has occurred

What are the characteristics of the May be pregnantabused in a domestic violence situation? May not report abuse to police

May not leave the abuserBlame themselves for the abuseEmotional and financial dependence onthe abuser

What characteristic is common to both Low self-esteemthe abused and the abuser in adomestic violence situation?

What is the role of the physician, in The physician does not have mandatoryterms of reporting, if notified of reporting since the abused is generally adomestic violence abuse? competent adult. The physician can

provide emotional support andencourage the abused to report theviolence.

60 Deja Review: Behavioral Science

08-Quinn_Ch08_p057-064.qxd 3/31/10 3:01 PM Page 60

Page 76: Deja review   behavioral science

SEXUAL VIOLENCE

What is the definition of sexual assault? A person commits a sexual assault whenhe or she uses force or the threat of forceto touch another person sexually in away that person does not want or whenthat person cannot give consent becauseof physical or mental inability.Note: Sexual assault is the legal term forrape.

What are examples of force that can be Manipulationused to commit a sexual assault? Coercion

Physical forceUse of weaponsUse of isolationUse of substances—alcohol and otherdrugs

What is the definition of consent? Giving permission by giving a “yes”response

What term describes oral and anal Sodomypenetration?

What is the primary age group <25 years of ageof a rapist?

What is the racial background of a Perpetrator will generally be the sameperpetrator of sexual assault? race as the victim.

Which substance is most frequently Alcoholused in cases of sexual assault?

What percentage of rapes are Approximately 85% of rapesacquaintance rapes (ie, the victimknows the perpetrator)?

What percentage of rapes are reported Approximately 10%—rape is the mostto the police? underreported violent crime in the

United States.

What is the age group most likely to 16 to 24 years of ageexperience a sexual assault?

What is the most common place In the victim’s homefor a sexual assault to occur?

Abuse and Aggression 61

08-Quinn_Ch08_p057-064.qxd 3/31/10 3:01 PM Page 61

Page 77: Deja review   behavioral science

Which disorder do/may sexual assault Posttraumatic stress disordersurvivors experience?

Which type of treatment is the best Group therapyoption for survivors of sexual assault?

Is it possible for spouses to be Yesconvicted of committing a sexualassault against each other?

What terms describes consensual sex Statutory rapethat is considered rape?

With which group(s) of people would Laws vary by state—usually consensualconsensual sex be considered age is either 16 or 18, but some statesstatutory rape? have age difference limits (eg, not over

3 years difference if under 18 years old)Profoundly disabled persons

What is the role of the physician Laws vary by state—depending on ageif they suspect a sexual assault? of the victim, this may fall under

mandatory reporting for child or elderabuse; in adult cases it is the victims’choice to report.

AGGRESSION

What has happened since the 1990s Decreasedto the incidence of homicide in theUnited States?

Which weapon is most commonly used Gunsin a homicide?

What are risk factors for being victims Male sexof homicide? Low socioeconomic status

Alcohol and drug users<65 years of age

Which racial group is most likely to be Young African American malesaffected by a homicide?

How does violence on television or Direct correlation with increasedvideo games influence aggression aggression in childrenin children?

Which gender is most likely to be most Malesaggressive?

62 Deja Review: Behavioral Science

08-Quinn_Ch08_p057-064.qxd 3/31/10 3:01 PM Page 62

Page 78: Deja review   behavioral science

Why are many body builders and some Use of androgenic or anabolic steroidsprofessional athletes likely to show increases aggression.increased aggression?

Use of which drugs is associated with Alcoholincreased aggression? Amphetamines

CocainePhencyclidineMarijuana (high doses)

Which neurotransmitter(s) is associated Dopaminewith increased aggression? Norepinephrine

Which neurotransmitter(s) is associated Serotoninwith decreased aggression? γ-Aminobutyric acid (GABA)

What type of injury is most likely to be Head injuryassociated with violence?

CLINICAL VIGNETTES

A family who has newly immigrated to the United States comes to your familypractice to establish care. The son in the family has been suffering from a chroniccough and the parents are worried. Upon examination you find well-demarcated,circular, purpuric lesions on his back. They don’t look accidental. What should youdo?

Ask the family about their cultural practices. This could represent a culturalhealing technique such as “cupping” used to heal the boy’s cough. It is importantto distinguish this from child abuse.

A 21-year-old female college student decides to go on a date with her boyfriend of2 years. She consumes a large volume of alcohol while out. When they return toher dorm room, the boyfriend tries to coerce her into having sex with him. Shepasses out without agreeing. Since they’ve had had sex before, he cites their long-term relationship as proof of consent. Would this incident be considered sexualassault?

Yes—a long-term relationship (or even marriage) does not substitute or replaceconsent. Even if the woman agreed to intercourse, an intoxicated person is unableto give consent and the act would still be considered sexual assault.

Abuse and Aggression 63

08-Quinn_Ch08_p057-064.qxd 3/31/10 3:01 PM Page 63

Page 79: Deja review   behavioral science

This page intentionally left blank

Page 80: Deja review   behavioral science

C H A P T E R 9

Suicide

65

SUICIDE

What is important to keep in mind There is only a limited amount ofwhen approaching the topic of suicide information that you will be askedon the United States Medical Licensing regarding suicide. Always think safetyExam (USMLE) Step 1? first when given a question regarding a

psychiatric disorder. Statistics,demographics, and risk factors, andcomorbid medical disorders willprobably be the focus of the examcontent. Many of the questions willcome in the form of a case scenariowritten to ascertain if you know thecorrelation between suicide andcomorbid mental health diagnoses suchas depression, bipolar disorder, and/orchemical dependence.

Where does suicide rank as a cause of As of 2002, suicide ranks 10th as thedeath in the United States? leading cause of death.

What are risk factors for committing White male, >65 years oldsuicide? Gun in the house or easy access to

firearmsComorbid depression, substance abuse,and/or other psychiatric illnessRecent loss or stressorSerious medical illnessFeelings of hopelessness andimpulsivity

Who commits suicide more frequently, Men are four times more likely tomen or women? complete suicide, though women are

more likely to attempt suicide.

Page 81: Deja review   behavioral science

What accounts for this difference? Men tend to use more lethal means(firearms, hanging) and women lesslethal (pill overdose).

What age group has the highest • The elderly (65 and older). Despitesuicide rate? being only 14% of the population,

they comprise about 18% to 25%of the total suicide percentage.

• The elderly suicide rate is 40 per100,000 persons in comparison to thenational US rate of 12 per 100,000.

What has happened to the suicide rate It has risen significantly over the lastin child and adolescent groups over 40 years with a mortality rate of 12%,the last 40 years? though declined since 1990.

What is the most significant risk factor A previous attempt.of suicide? You should become extremely cautious

if the attempt is recent (eg, within3 months) and if the nature of theattempt is well-thought out anddeliberate.

What are other risk factors for suicide 1. Persons >45 years(in decreasing order)? 2. Alcohol dependence

3. History of violence or aggression4. Male gender

How is suicide most commonly Firearms are the number one methodcommitted or attempted? for completed suicide, regardless of

gender.Other common methods includejumping, hanging, overdose,and drowning.

Is there racial disparity among those Yes, whites commit suicide more thanwho commit suicide? any other group. However, the suicide

rate among young black males is slowlyincreasing.

What are the most common comorbid Major Depressive Disordermedical disorders associated Anxiety Disorderswith suicide?

Posttraumatic Stress Disorder (PTSD)SchizophreniaBipolar DisorderPersonality DisordersSubstance Abuse

66 Deja Review: Behavioral Science

Page 82: Deja review   behavioral science

At what stage, in treated major For adults, suicide is most attempteddepression, is suicide attempted? after pharmacologic treatment has

begun. The patient has more energy andis better able to function but low moodand hopelessness may persist.

Are there special concerns about There is FDA black boxchildren and adolescents treated warning for all antidepressants thatfor depression? use in children and adults younger than

25 may increase suicidal thoughts andbehaviors. When starting such amedication in this population, frequentand close monitoring for suicidality isimportant.

When do you assess for suicidality? At every follow-up assess for:• Ideation• Plan (possibility and practicality)• Intent to carry out the plan• Access to firearms

What is the difference between active Passive: Patient would like to be deadand passive suicidal ideation? (eg, “I wish I were dead!”)

Active: Patient actually wants to harmhim/herself (eg, “I am going to shootmyself with a gun.”)

Can suicide attempts be predicted? There is no reliable means to predictsuicide attempts with a reasonabledegree of specificity and sensitivity.

How can suicidal ideation be managed? If suicide risk is imminent, the patientmust be detained and the appropriatemental health care sought.If suicide risk is not imminent,assessment and defusing risks isimportant (eg, removing firearms fromhouse, involving family membersif possible).Antidepressant medication should bestarted and the patient should befollowed closely and seen frequently.

What antidepressant would be the first Aggressive SSRI (selective serotoninchoice in a suicidal patient? reuptake inhibitor) therapy is the best

choice—avoid monoamine oxidase(MAO) inhibitors and tricyclics in apatient with active suicidal ideation asthey can be lethal in an overdose.

Suicide 67

Page 83: Deja review   behavioral science

What are protective factors from • Marriage: Single persons that aresuicide? divorced, have never married, or

widowed have higher suicide ratesthan married persons. Living aloneand limited social and family supportare also risk factors.

• Employment: Persons who areunemployed have higher suiciderates. However, among employed UScitizens, those working in professionalroles (doctors, lawyers, and in lawenforcement) have higher suiciderates than nonprofessional persons.

• No family history of a completedsuicide or an attempted suicide isprotective because once suicideoccurs, it seemingly decreases thesocial “taboo” of suicide in the family.

CLINICAL VIGNETTES

A patient in your outpatient clinic presents with suicidal ideation, but no currentintent to harm himself. You would like to decrease the probability of himcommitting suicide and so prescribe him a high-dose SSRI. Will this eliminate hissuicide risk?

Though aggressive treatment is indicated and untreated depression carries a highrisk of suicide, treatment with SSRI alone will not necessarily eliminate risk ofsuicide. Screening for associated risk factors such as access to firearms and otherlethal means is also indicated.

68 Deja Review: Behavioral Science

Page 84: Deja review   behavioral science

C H A P T E R 1 0

Genetic Basisof Behavior

69

GENETIC STUDIES

What type of study uses a family tree Pedigree studyto show the occurrence of traits anddiseases throughout generations?

What type of study compares the Family risk studyfrequency of disease in a proband(affected individual) with its frequencyin the general population?

What type of study compares Adoption studymonozygotic and dizygotic twinsto determine the effects of geneticfactors from environmental factorsof disease?

This term describes if both twins have Concordancea given trait.

What type of twins is more likely Monozygotic twinsto have a higher likelihood of havinga disease that is genetic in origin?

PSYCHIATRIC DISORDERS GENETICS

What is the prevalence of schizophrenia 1%in the general population?

In which gender is schizophrenia more Equal in males and femaleslikely?

Which persons have a higher likelihood Persons with a close genetic relationshipof developing schizophrenia?

Page 85: Deja review   behavioral science

Genetic markers on which 1, 6, 8, and 13chromosome(s) are associated withschizophrenia?

Does schizophrenia have a familial Yes—the closer a relative you are, thecomponent? more likely you are to develop

schizophrenia; but concordance in twinsis not 100%.

What is the likelihood of developing 10%schizophrenia if you are a first-degreerelative of an affected individual?

What is the likelihood that a child 40%will develop schizophrenia if bothparents have the disorder?

What is the likelihood that the 50%monozygotic twin of a patient withschizophrenia will develop thedisorder?

What is the likelihood of developing 25%any mood disorder if you are a first-degree relative of a person with bipolardisorder?

What is the likelihood of developing 60%any mood disorder if someone is a childwith both parents who have bipolardisorder?

What is the likelihood of developing 80% to 90%bipolar disorder if you are themonozygotic twin of a person withthe disorder?

Is the genetic component stronger for Bipolar disorderschizophrenia or bipolar disorder?

In which gender is the lifetime Femalesprevalence of a major depressivedisorder higher?

What is the percentage of males who 10%will develop a major depressivedisorder in their lifetime?

What is the percentage of females 15% to 20%who will develop a major depressivedisorder in their lifetime?

70 Deja Review: Behavioral Science

Page 86: Deja review   behavioral science

Do personality disorders have a higher Yesconcordance in monozygotic twinsdemonstrating that they have a geneticcomponent?

If a person has antisocial personality Alcoholismdisorder, what psychiatric condition(s) Attention-deficit hyperactivity disorderwill be prevalent in relatives? (ADHD)

If a person has avoidant personality Anxiety disorderdisorder, what psychiatric condition(s)will be prevalent in relatives?

If a person has borderline personality Major depressive disorderdisorder, what psychiatric condition(s) Substance abusewill be prevalent in relatives?

If a person has histrionic personality Somatization disorderdisorder, what psychiatric condition(s)will be prevalent in relatives?

If a person has schizotypal personality Schizophreniadisorder, what psychiatric condition(s)will be prevalent in relatives?

NEUROPSYCHIATRIC DISORDER GENETICS

In which disease is there a diminution Alzheimer diseaseof cognitive functioning and alikelihood of genetic influence?

Which chromosome has three copies in Chromosome 21Down syndrome, and is also implicated Note: Down syndrome patients oftenin some cases of Alzheimer disease? develop early-onset Alzheimer.

What other chromosome(s) have been Chromosomes 1 and 14identified to be associated withAlzheimer disease?

Which gene has been implicated in The gene encoding Apolipoproteinmany cases of Alzheimer disease? E4 (Apo E4)

In which chromosome is the Apo E4 Chromosome 19gene located?

What disease has an abnormal gene Huntington diseaseon the short end of chromosome 4?

What is the most common genetic cause Down syndromeof mental retardation?

Genetic Basis of Behavior 71

Page 87: Deja review   behavioral science

What is the second most common Fragile X syndromegenetic cause of mental retardation?

What disorder, characterized by verbal Tourette disorderand motor tics, has a geneticcomponent?

ALCOHOLISM GENETICS

What is the prevalence of alcoholism Four times more prevalentin children of alcoholics comparedto the general population?

What is the concordance rate of 60%alcoholism in monozygotic twins?

What is the concordance rate of 30%alcoholism in dizygotic twins?

Which gender offspring of alcoholics Male offspringis more likely to become alcoholicsthemselves?

In which age group is the genetic <20 years of ageinfluence of alcoholism strongestin males?

CLINICAL VIGNETTES

A 17-year-old woman comes to your office worried that she might becomeschizophrenic. She hasn’t had any symptoms, but she says that some of her familymembers are schizophrenic. What would be important historical information toassess her risk?

How closely related she is to those family members with schizophrenia. The moregenetically similar to them she is, the higher her risk is.

A 41-year-old Down syndrome patient has had a gradual deterioration in cognitivefunctioning. Lately, she has been unable to remember her home address, and herdaughter has noticed that she has difficulty remembering names of people she hasknown throughout the duration of her life. Her primary care physician diagnosesher with Alzheimer disease. Which chromosome has been implicated in Alzheimerdisease in Down syndrome patients?

Chromosome 21

72 Deja Review: Behavioral Science

Page 88: Deja review   behavioral science

A 19-year-old male is brought to the emergency room by his college roommate fordelusions, hallucinations, and disorganized speech that has occurred for the last7 months. The roommate is concerned that the patient may have schizophrenia. Thepatient’s father was diagnosed with schizophrenia at age 22. Not taking into accountthese new symptoms, what is the lifetime likelihood of this patient developingschizophrenia?

The patient is a first-degree relative of an affected individual—he therefore has a10% chance of developing schizophrenia.

Genetic Basis of Behavior 73

Page 89: Deja review   behavioral science

This page intentionally left blank

Page 90: Deja review   behavioral science

C H A P T E R 1 1

Neurochemistry inBehavioral Sciences

75

NEUROANATOMY

What are the two divisions of the 1. Central nervous system (CNS)nervous system? 2. Peripheral nervous system (PNS)

What are the two main components 1. Brainof the CNS? 2. Spinal cord

Which brain structures connect Corpus callosumthe cerebral hemispheres? Commissures: anterior, posterior,

hippocampal, and habenular

Which hemisphere is usually the Left hemispheredominant hemisphere?

What is the primary role of the left It governs our ability to expresshemisphere? ourselves in language.

Which hemisphere is usually the Right hemispherenondominant hemisphere?

What is the primary role of the right It governs perceptual functions and thehemisphere? analysis of space, geometrical shapes,

and forms.

What are the components of the Nerve fibers outside the CNS includingperipheral nervous system? cranial nerves, spinal nerves, and

peripheral ganglia

How many cranial nerves are there? 12 cranial nerves

How many spinal nerves are there? 31 pairs of spinal nerves: 8 cervical,12 thoracic, 5 lumbar, 5 sacral, and1 coccygeal

Page 91: Deja review   behavioral science

In which direction does the PNS carry Motor information: away from the CNSmotor and sensory information Sensory information: to the CNSto the CNS?

What are the components of the Sensory neurons and motor neuronsautonomic nervous system? that run between the CNS (especially

the hypothalamus and medulla oblongata)and various internal organs.

BRAIN LESIONS

What will be the neuropsychiatric Deficits in concentration, judgment,consequences of a frontal lobe lesion? motivation, and orientation

DisinhibitionPersonality and emotional changes

What will be the neuropsychiatric Right parietal lobe → (contralateralconsequences of a parietal lobe lesion? neglect) result in neglecting part of the

body or spaceLeft parietal lobe → verbal deficits

What will be the neuropsychiatric Hallucinationsconsequences of a temporal lobe lesion? Memory deficits

Personality changes

What will be the neuropsychiatric Bilateral damage to hippocampus leadsconsequences of a hippocampus lesion? to massive anterograde and some

retrograde amnesia.

Unilateral damage of hippocampusleads to memory storage and retrievalproblems.

What will be the neuropsychiatric Kluver-Bucy syndrome → uninhibitedconsequences of an amygdala lesion? behavior, hyperorality, hypersexuality

What will be the neuropsychiatric Sleep-wake cycle changesconsequences of a reticular systemlesion?

What will be the neuropsychiatric Tremor or other involuntary movementsconsequences of a basal ganglia lesion? as seen in Parkinson or Huntington

diseases

What will be the neuropsychiatric Decreased satiety → leads to obesityconsequences of a hypothalamuslesion of the ventromedial nucleus?

76 Deja Review: Behavioral Science

Page 92: Deja review   behavioral science

What will be the neuropsychiatric Decreased hunger → leads to weight lossconsequences of a hypothalamus lesionof the lateral nucleus?

What will be the neuropsychiatric Disturbances of parasympatheticconsequences of a hypothalamus lesion activityof the anterior hypothalamus? Disturbances of body cooling

What will be the neuropsychiatric Disturbances of heat conservationconsequences of a hypothalamus lesionof the posterior hypothalamus?

What will be the neuropsychiatric Change in sexual urges and emotionsconsequences of a hypothalamus lesionof the septate nucleus?

What will be the neuropsychiatric Disturbances of circadian rhythmconsequences of a hypothalamus lesionof the suprachiasmatic lesion?

NEUROTRANSMITTERS

What are the four main steps involved 1. Presynaptic neuron stimulation.in neurotransmitter release? 2. Neurotransmitter release.

3. Neurotransmitter moves acrosssynaptic cleft.

4. Neurotransmitter acts onpostsynaptic neuron receptors.

What are the two different types of 1. Excitatory: increase neuron firingneurotransmitters? 2. Inhibitory: decrease neuron firing

What are the two main excitatory and 1. Excitatory: glutamateinhibitory neurotransmitters 2. Inhibitory: γ-aminobutyric acidin the CNS? (GABA)

Which factors contribute to the 1. Affinity of receptorsmagnitude of reaction 2. Number of receptorsneurotransmittershave on neurons?

What are the three major classes of 1. Amino acidsneurotransmitters? 2. Biogenic amines

3. Peptides

How are neurotransmitters removed Reuptake by the presynaptic neuronfrom the synaptic cleft? Degradation by enzymes (eg, monoamine

oxidase)

Neurochemistry in Behavioral Sciences 77

Page 93: Deja review   behavioral science

Why is this important in pharmacology? These are common targets forpsychotropic drug actions (eg, SSRIs,MAO inhibitors)

78 Deja Review: Behavioral Science

Table 11.1 Neurotransmitter Alterations in Psychiatric Conditions

AlzheimerDisease Anxiety Depression Mania Schizophrenia

Acetylcholine ↓

Dopamine ↓ ↑ ↑

GABA ↓

Norepinephrine ↑ ↓

Serotonin ↓ ↓ ↑

AMINES

Which amines are included in the Catecholaminesbiogenic amines, which are also called Ethylaminesthe monoamines?

IndolaminesQuaternary amines

What is the monoamine theory It proposes that there is an underlyingof depression? neuroanatomical basis for depression

due to deficiencies of centralnoradrenergic and/or serotonergicsystems.

Why are metabolites of monoamines They may be present in higher levelsmeasured in psychiatric studies? than the primary monoamines.

What type of biogenic amine Catecholamineis dopamine?

In which psychiatric condition(s) is an Mood disordersaltered level of dopamine evident? Parkinson disease

Schizophrenia

How is dopamine synthesized? By the conversion of tyrosine todopamine by tyrosine hydroxylase

tyrosine hydroxylasetyrosine ⎯⎯⎯⎯⎯⎯⎯⎯⎯→ dopamine

Page 94: Deja review   behavioral science

What is the main metabolite Homovanillic acid (HVA)of dopamine?

In which psychiatric condition(s) can Psychotic disordersthere be an increased concentration Schizophreniaof HVA?

In which psychiatric condition(s) can Alcoholismthere be a decreased concentration Depressionof HVA?

Parkinson disease

What type of biogenic amine is Catecholaminenorepinephrine?

What behavioral factors does Anxietynorepinephrine alter? Arousal

LearningMemoryMood

How is norepinephrine synthesized? Dopamine is converted tonorepinephrine by β-hydroxylase.

β-hydroxylasedopamine ⎯⎯⎯⎯⎯⎯→ norepinephrine

Where are most noradrenergic neurons Locus ceruleuslocated in the brain?

What are the metabolites of 3-Methoxy-4-hydroxyphenylglycolnorepinephrine? (MHPG)

Vanillylmandelic acid (VMA)

In which psychiatric condition(s) can Severe depressionthere be a decreased concentrationof MHPG?

In which medical condition is Pheochromocytoma → a tumor of thethere an increased concentration adrenal medullaof VMA?

What type of biogenic amine Indolamineis serotonin? Note: Another name for serotonin is

5-hydroxytryptamine (5-HT).

What behavioral factors does Impulse controlserotonin alter? Mood

SleepSexuality

Neurochemistry in Behavioral Sciences 79

Page 95: Deja review   behavioral science

If serotonin levels are increased, which Moodbehavioral factors will be improved? Sleep

If serotonin levels are increased, which Sexual functioningbehavioral factors will be impaired?

If serotonin levels are decreased, which Impulse controlbehavioral factors will be impaired? Sleep

Note: Patient is likely to experiencedepression.

How is serotonin synthesized? Tryptophan is converted to serotonin bytryptophan hydroxylase and an aminoacid decarboxylase.

Where are most serotoninergic cell Dorsal raphe nucleusbodies located in the brain?

Which pharmacologic agents are used Antidepressants—eg, selective serotoninto alter the level of serotonin in reuptake inhibitors (SSRIs)the brain?

What is the primary metabolite of 5-Hydroxyindoleacetic acid (5-HIAA)serotonin?

In which psychiatric condition(s) is Alcoholismthere a decreased concentration Bulimiaof 5-HIAA?

Impulsive behaviorPyromania: uncontrollable desire to setthings on fireSevere depressionTourette syndromeViolent behavior

What type of biogenic amine Ethylamineis histamine?

Which pharmacologic agents block Antipsychotic drugsthe histamine receptor? Tricyclic antidepressants (TCAs)

What are side effects of the histamine Increased appetite → contributing toreceptor blockade? weight gain and obesity

Sedation

What type of biogenic amine Quaternary amineis acetylcholine (ACh)?

Where is acetylcholine normally Neuromuscular junctionsfound in the body?

80 Deja Review: Behavioral Science

Page 96: Deja review   behavioral science

Which psychiatric conditions are Alzheimer diseaseassociated with a decrease in Down syndromecholinergic neurons?

Movement disorders

How is acetylcholine synthesized? Acetyl coenzyme A (CoA) and cholineare converted to acetylcholine bycholine acetyltransferase in cholinergicneurons

How is acetylcholine degraded? Acetylcholine esterase (AChE) degradesacetylcholine into acetate and choline.

Which pharmacologic agents have been Donepezilshown to reduce the degradation Tacrineof acetylcholine?

Note: These agents can slow theprogression of diseases such asAlzheimer disease.

What are the three primary amino acid 1. GABAneurotransmitters? 2. Glutamate

3. Glycine

Which amino acid neurotransmitter(s) Glutamateare excitatory?

Which amino acid neurotransmitters(s) GABA—primary inhibitoryare inhibitory? neurotransmitter

Glycine

Which pharmacologic agents alter Barbiturates: alter duration of GABAduration and frequency of GABA? channel opening

Benzodiazepines: alter frequency ofGABA channel opening

Which neurotransmitter regulates Glycineglutamate activity?

Which pathologic conditions may Cell death mechanismsglutamate play a role in? Epilepsy

Neurodegenerative diseasesPsychotic disorders (eg, schizophrenia)

NEUROPEPTIDES

What are the two endogenous opioids? 1. Endorphins2. Enkephalins

Neurochemistry in Behavioral Sciences 81

Page 97: Deja review   behavioral science

What behavioral factors do endogenous Anxietyopioids alter? Mood

PainSeizure activityTemperature regulation

Which factor does endogenous Placebo effects → endogenous opioidsopioids alter in research studies? are thought to play a major role in the

placebo effects seen in research studies.

Which neuropeptide(s) has been Substance Pimplicated in aggression and pain?

Which neuropeptides have been Somatostatinimplicated in Alzheimer disease? Vasoactive intestinal peptide (VIP)

Which neuropeptides have been Oxytocinimplicated in mood disorders? Somatostatin

Substance PVasopressinVIP

Which neuropeptides have been Cholecystokinin (CCK)implicated in schizophrenia? Neurotensin

CLINICAL VIGNETTES

A man in your neighborhood has had a recent accident that caused brain damage.You notice his personality is much different now and he seems to have troublewith higher thought processes. He now also uses profanity more often. What partof his brain was most likely damaged?

The frontal lobe

82 Deja Review: Behavioral Science

Page 98: Deja review   behavioral science

S E C T I O N I I

PsychiatricDisorders andTreatment

Page 99: Deja review   behavioral science

This page intentionally left blank

Page 100: Deja review   behavioral science

C H A P T E R 1 2

Psychotic Disorders

85

INTRODUCTION

What is the most used book of criteria DSM-IV-TRfor the diagnosis of psychiatricdisorders?

What are the five axes used in the 1. Axis I: Major psychiatric disorderDSM-IV-TR? 2. Axis II: Personality disorders and

mental retardation3. Axis III: Medical conditions4. Axis IV: Environmental factors5. Axis V: Global assessment of

functioning (0-100)

What is an important criterion The symptoms cause significantof most DSM-IV-TR disorders? social or functional impairment.

This distinguishes a disorder froma normal variant.

What strategy will help when learning Try to focus on differences betweenthe different types of disorders? similar disorders—they may have to

do with duration, severity, or subtlefeatures present or absent in a disorder.

DISORDERS

What is psychosis? Significant impairment in realitytesting (ability to distinguish realfrom imaginary)

What are the clinical hallmarks Delusions: fixed false beliefs, despiteof psychosis? evidence to the contrary

Hallucinations—usually auditoryDisorganized speech (thought disorder)Grossly disorganized or catatonic(stupor and bizarre posturing) behavior

Page 101: Deja review   behavioral science

What is the difference between An illusion is the misperception ofa hallucination and an illusion? an actual sensory stimulus (eg, seeing

a pool of water on the road aheadduring a hot summer day),whereas hallucinations areperceptions in the absence ofan external stimulus.

Name the six major primary psychotic 1. Brief psychotic disorder (<1 monthdisorders. of symptoms)

2. Schizophreniform disorder (1 to6 months of symptoms)

3. Schizophrenia (>6 months ofsymptoms)

4. Schizoaffective disorder5. Delusional disorder6. Shared psychotic disorder

Other than primary psychotic disorders, • Mood disorders: Major depression orwhat other types of psychiatric illness manic episodes may include psychoticoften manifest psychotic symptoms? symptoms, but only during a mood

disturbance.• Substance use: Acute intoxication

(especially with cocaine, lysergic aciddiethylamide [LSD], phencyclidine[PCP], and amphetamines) orwithdrawal (especially alcohol).Anytime a patient has tactilehallucinations, you should thinkabout drugs.

• Personality disorders: May beassociated with brief (not sustained)periods of psychosis.

• Cognitive disorders: Both deliriumand dementia demonstrate psychosis.Often delirium will have visualhallucinations in addition to cloudedsensorium.

• Psychosis due to a general medicalconditions such as Vitamin B12deficiency, Multiple Sclerosis (MS),Systemic Lupus Erythematosus (SLE),uremia, etc.

• Narcolepsy: May havehypnagogic and hypnopompichallucinations.

86 Deja Review: Behavioral Science

Page 102: Deja review   behavioral science

Schizophrenia

Which of the primary psychotic Schizophrenia. The incidencedisorders is the most common? in the adult population is

around 1%.

What are the DSM-IV-TR criteria Two or more characteristic symptoms:for Schizophrenia? • Delusions

• Hallucinations• Disorganized speech• Grossly disorganized behavior• Negative symptomsAnd social/occupational dysfunctionAnd disturbance lasting 6+ months(including any prodrome orresidual)And not due to Schizoaffective disorderor General Medical Condition orsubstance use

What are negative symptoms? Negative symptoms (think deficits) includeaffective flattening, alienation (socialwithdrawal), alogia (poverty ofspeech), and avolition (lack ofmotivation).

What are considered the positive Usually this refers to the hallucinations,symptoms of schizophrenia? delusions, bizarre behavior, and thought

disorder.

What is a “first-rank” symptom? A particularly bizarre delusion, orhallucinations consisting of either avoice running commentary on thepatient’s activities or two voicesconversing. A first-rank symptommay satisfy both symptomrequirements of the DSM-IV-TRcriteria above.

How does the criterion of Abnormalities in thought processes anddisorganized speech, (also referred thought formationto as thought disorder) manifestin schizophrenia?

Psychotic Disorders 87

Page 103: Deja review   behavioral science

Give at least two examples of abnormal • Word salad: Words and phrases arethought formation. combined together in incoherent

manner.• Neologisms: Creation of new words.• Echolalia: Repeating the same word

over and over (almost like a mentalstutter).

• Loose associations: Illogical shiftingbetween unrelated or obliquelyrelated topics.

• Thought blocking.• Circumstantial and tangential

thought: Circumstantial thoughteventually gets to the point;tangential never does.

88 Deja Review: Behavioral Science

A A B A B

X Y

BNormal Circumstantial Tangential

WX

YZ

What is meant by the prodromal and These are periods of time before andresidual phases of schizophrenia? after active psychotic periods,

respectively. These periods aregenerally characterized by attenuatedsymptoms of the active phase, eg, socialwithdrawal, peculiar behavior, or oddaffect.During these periods a person wouldseem strange but would not necessarilymeet criteria for psychotic.

How long must you have symptoms for At least 6 months including prodromalbefore schizophrenia can be diagnosed? and residual phases, but they must have

at least 1 month of active characteristicsymptoms (above), as well as decline infunction.

How does schizophreniform disorder The duration is <6 months, but is atdiffer from schizophrenia? least 1 month. It also does not require a

decline in function.

What would you call psychosis lasting Brief Psychotic Disorder (which lasts<1 month? over 24 hours, but <1 month).

Page 104: Deja review   behavioral science

Does gender affect the development There is no difference in the prevalenceof schizophrenia? of schizophrenia between men and

women; however, the age of onset isaffected. Men tend to develop itbetween 15 and 25 years of age andwomen between 25 and 35 years of age.

What other epidemiological factors Having a first degree relative withare correlated with increased risk schizophrenia increases a patients’ riskof schizophrenia? of schizophrenia tenfold. Being born in

the cold winter months or in an area ofhigh population density have also beenassociated with increased risk (thoughless so).

What is downward drift? This is the tendency of schizophrenicsto be of lower socioeconomic status.It is generally thought that this is dueto inability to function well in society,causing a “drift” into lowersocioeconomic classes.

What is the dopamine hypothesis? This is the classic understanding ofschizophrenia, which attributes thesymptoms of schizophrenia tohyperactivity of the dopaminergicsystem. Many other theories havebeen postulated, however, especiallyinvolvement of Glutamate.

Are there any structural brain changes Increase in size in the lateral andassociated with schizophrenia? third ventricles, generalized atrophy

of the cortex, and frontal lobeabnormalities are often associatedwith schizophrenia.

What are the five major subtypes 1. Paranoidof schizophrenia? 2. Residual

3. Catatonic4. Disorganized5. Undifferentiated

Which form tends to have the best Paranoid. They also tend to be slightlysocial functioning? older at onset, and have prominent

hallucinations and delusions, with alesser component of disorganization.

Which form is the least common? Catatonic. Prior to the developmentof antipsychotics, this form wasmore common.

Psychotic Disorders 89

Page 105: Deja review   behavioral science

What is catalepsy and waxy flexibility? Catalepsy is the ability of a catatonicpatient to hold a seemingly uncomfortableposition for extended periods of time.Waxy flexibility is the slight resistancegiven to moving the limbs, after whichthe patient will often hold the newposition given.

What is the most predictive of overall Level of premorbid functionprognosis in schizophrenia?

Overall, schizophrenia is associated Abrupt onset, female gender, presencewith repeated psychotic episodes, and of mood symptoms, and old age at onseta chronic downhill course. Whatfeatures are associated with a somewhatbetter prognosis?

Are patients with schizophrenia at risk Yes! Over half of schizophrenicsof suicide? will attempt suicide at some point in

their lives, and 10% will die from it.

What is the primary treatment for Antipsychotics.schizophrenia?

What is the difference between typical Typical antipsychotics are older andand atypical antipsychotics? tend to work by antagonizing dopamine.

Comparatively they have higher ratesof extrapyramidal symptoms (EPS) andtardive dyskinesia (TD).Atypical antipsychotics are newer, havecomplex mechanisms, have less EPSand TD, are more expensive, and havemore metabolic side effects.

What is EPS? This includes tremor, rigidity, akathisia(inner restlessness), and acute dystonias(muscle spasm).

How do you treat EPS? Anticholinergics (eg, diphenhydramineand benztropine)

What is tardive dyskinesia? Abnormal movements of the face,trunk, extremities, and mouth thatmay happen after prolonged exposureto antipsychotic medications.

How do you treat tardive dyskinesia? Early identification and removalof the offending antipsychotic drug.With early recognition, remission ratesare reasonably high. Benzodiazepinescan be used for mild persistent cases.

90 Deja Review: Behavioral Science

Page 106: Deja review   behavioral science

What are some differences between EPS can develop quite quickly, whereasEPS and TD to remember? TD tends to come on after prolonged

exposure to medicines—months to years.TD is not helped by anticholinergicsand may become worse. TD is alsomore often permanent.

What is a potentially fatal side effect Neuroleptic malignant syndromeof antipsychotic treatment? (NMS). This is an idiosyncratic

reaction that is more common inyoung men, usually after recentlystarting a new antipsychotic.Mortality is nearly 20%.

How does NMS present? Fever, muscle rigidity, altered mentalstatus, and autonomic instability

What is the first thing you should do Stop the antipsychotic! Then care isif a patient presents to the ER primarily supportive.with NMS?

Other Psychotic Disorders

What is the DSM-IV-TR criteria Must meet criteria for either a majorfor Schizoaffective Disorder? depressive episode, a manic episode,

or a mixed episode at the same timeas meeting characteristic symptoms ofschizophrenia. The schizophrenicsymptoms must persist in theabsence of mood symptoms.

What are the two types of 1. Bipolarschizoaffective disorder? 2. Depressed

How could you differentiate A person that has a mood disorder withbetween a mood disorder with psychotic symptoms will not havepsychotic symptoms (eg, major psychosis without a mood disturbance.depressive disorder [MDD] with Schizoaffective patients have psychosispsychosis or Bipolar Mania) from at some point even when they are notschizoaffective disorder? experiencing an affective disturbance.

Tip: Look at the names! For example, inschizoaffective disorder, the emphasis ison the schizo. The primary problem is apsychotic disorder that sometimes hasa mood component, whereas in MDDwith psychosis, the emphasis is onMDD. The main problem is a mooddisorder that when severe, may havea component of psychosis.

Psychotic Disorders 91

Page 107: Deja review   behavioral science

What is a delusional disorder? Patients with delusional disorder tendto have an isolated, fixed, nonbizarredelusion (eg, the Internal RevenueService [IRS] is after them, or their partneris cheating). Even if the delusion isunfounded, it is plausible. They are notdisorganized in thoughts or affect, andpatients do not meet criteria forschizophrenia.

What are some types of delusions 1. Erotomanic: Someone is in love withthat may occur in this disorder? patient.

2. Somatic.3. Grandiose.4. Jealous: Wife/husband is cheating.5. Persecutory: Patient is being

mistreated.

What is a shared psychotic disorder? A rare disorder where the patientbelieves the delusions of another personwith a primary psychotic disorder.

CLINICAL VIGNETTES

A 27-year-old male presents to your clinic with a 4-year history of hallucinationsand delusions for which he has been intermittently managed with antipsychotics.He is currently untreated. He also complains of 4 months of depressed mood,weight loss, insomnia, fatigue, and loss of interest in activities. He denies usingany illicit substances or having any other medical conditions. What is the mostlikely diagnosis?

Schizoaffective Disorder. Note that there have been psychotic symptoms withoutaffective symptoms present, but now also meets criteria for a depressive episode.

A 76-year-old man presents with 1 year of worsening depressive symptoms. He hastrouble falling asleep, feels worthless, cannot concentrate, and has thoughts ofdeath. Over 3 years ago his wife passed away from cancer. For 6 months now hehas adamantly stated that the cancer was his fault and that he was the one thatkilled his wife, despite all evidence to the contrary. He also often hears her voicescolding him when no one is around. What is the most likely diagnosis?

Depression with Psychotic Features. Note the mood-congruent delusion in thesetting of depression and psychotic symptoms that only appear during affectiveepisode. Also remember schizophrenia rarely presents in this advanced age.

92 Deja Review: Behavioral Science

Page 108: Deja review   behavioral science

C H A P T E R 1 3

Mood Disorders

93

What are the DSM-IV-TR criteria for a Five or more of following symptomsmajor depressive episode? present for at least 2 weeks (one of

which must be either depressed moodor anhedonia):Depressed moodAnhedoniaFatigueIncrease or decrease in sleepIncrease or decrease in appetite or weightDecrease in ability to concentrateFeelings of worthlessness or guiltPsychomotor retardationRecurrent thoughts of death or suicidalideation

What is anhedonia? Loss of pleasure in all or almost allactivities

What is SIG E CAPS? It is a mnemonic for depression:Sleep disturbances (mainly insomnia)Loss of InterestExcessive GuiltLoss of EnergyLoss of ConcentrationLoss of AppetitePsychomotor retardation or agitationSuicidal ideation (or recurrent thoughtsof death)

What is the prevalence of major 5% to 12% for men and 10% to 20%depressive disorder (MDD)? for women, with a 2:1 female to

male ratio

Page 109: Deja review   behavioral science

Does MDD appear at a certain age? The mean age of onset is 40.

What are the medical conditions that Thyroid dysfunction (particularly can cause or mimic a depressive hypothyroidism)episode? Malignancy

Stroke (Post-stroke depression) Neurodegenerative disordersMononucleosisAcquired immunodeficiency syndrome(AIDS)SyphilisParkinson diseaseStrokeMultiple sclerosisLupusNutritional deficiencyMenopause

What medications can cause depressed Accutane (isotretinoin)mood? Reserpine

Beta-blockersSteroids (eg, prednisone)MethyldopaInterferon (used to treat viral hepatitis)Oral Contraceptive Pills

What is double depression? This term is used when a patient meetscriteria for both MDD and dysthymicdisorder.

What is “postpartum onset” depression A major depressive episode happening(aka postpartum depression, PPD)? within 4 weeks of delivery (some believe

within 12 months of delivery is stillPPD)

What is the diagnostic criteria for Depressed mood for a least 2 years,dysthymic disorder? plus two more symptoms of major

depression, but not meeting criteriafor MDD

How can you differentiate dysthymic Dysthymic disorder must last longer,disorder from MDD? but is less severe.

How do you treat depression The first-line treatment option is thepharmacologically? selective serotonin reuptake inhibitors

(SSRIs) (see Table 13.1).

94 Deja Review: Behavioral Science

Page 110: Deja review   behavioral science

Are there any other antidepressants Yes, there are other antidepressantsalso considered as first-line treatment with different mechanism of actionoptions? (see Table 13.2).

Mood Disorders 95

Table 13.1 SSRIs and Their Unique Properties

BrandGeneric Name Name Side Effects Other

Citalopram Celexa Standard Fewest medication(Sexual side interactionseffects, nausea,vomiting,headache, anxiety,insomnia)

Escitalopram Lexapro Standard Isomer ofcitalopram, verysimilar

Fluoxetine Prozac Standard Long half-life,drug-druginteractions

Paroxetine Paxil Standard, plus Short half-life, canhas anticholinergic cause withdrawaleffects that can symptoms (exceptcause weight gain, CR form), drug-constipation, drug interactionsand sedation

Sertraline Zoloft Most GI side Very few drug-drugeffects interactions

Abbreviations: GI, gatrointestinal.

Page 111: Deja review   behavioral science

96 Deja Review: Behavioral Science

Table 13.2 Other Antidepressants Indicated for the Treatment of Depression

Generic Brand Mechanism SideName Name of Action Effects Other

Bupropion Wellbutrin Unknown, likely Seizures, ContraindicatedZyban (for via dopamine insomnia, in patients with

smoking and headache, seizures,cessation) norepinephrine nausea, bulimia, and

vomiting, anorexiaconstipation, Used to treatand tremor sexual side

effects causedby SSRIs

Mirtazapine Remeron Presynaptic alpha-2- Highly Agranulocytosisadrenergic sedating, in 0.1% antagonist (↑ ↑appetite, Also used tonorepinephrine ↑weight ↓ nauseaand serotonin)also potentantagonist of5-HT2 and5-HT3

Trazodone Desyrel Weakly inhibits Highly Used as aserotonin sedating, sleeping aidreuptake orthostatic

hypotensionPostsynaptically Uncommon

antagonizes but can5-HT2 cause

priapism(painfulerection)andarrhythmias

Nefazodone Serzone Similar to Headache, FDA black-boxtrazodone dry mouth, warning due

blurred to risk of livervision, failuresomnolence,andorthostatichypotension

Page 112: Deja review   behavioral science

What are tricyclic antidepressants Older antidepressants (also known as(TCAs)? heterocyclics) are currently considered

a second-line treatment option.

Why are tricyclics a second-line Greater side effects and lethality intreatment option? overdose

What are the side effects of tricyclic Anticholinergic effects: dry mouth,antidepressants? blurred vision, constipation, and

confusionAlpha-blocking effects: sedation alsocaused by antihistaminic effects,orthostatic hypotension, and cardiacarrhythmiasMay lower the seizure thresholdAre contraindicated in glaucomaMust be used with caution in urinaryretention

Mood Disorders 97

Table 13.2 Other Antidepressants Indicated for the Treatment of Depression (Continued )

Generic Brand Mechanism SideName Name of Action Effects Other

Venlafaxine Effexor Serotonin and Nausea, Can increasenorepinephrine dizziness, blood pressurereuptake sexualinhibitor, also dysfunction,inhibitor of headache,dopamine and dryreuptake at mouthhigh doses

Duloxetine Cymbalta Similar to Nausea, dry Also indicatedvenlafaxine mouth, for neuropathic

constipation, paindiarrhea,dizziness,andinsomnia

Page 113: Deja review   behavioral science

What are the most commonly used Amitriptyline, desipramine, imipramine,tricyclics? and nortriptyline

Are there any other medications to Yes, the monoamine oxidase inhibitorstreat depression? (MAOIs), considered third-line option.

MAOIs are rarely used any more.

Why are MAOIs a third-line option? The main reason for not using themis due to the drug-drug interactionsand diet restrictions. If these are notstrictly followed, they may causeserotonin syndrome andhypertensive crisis.

What is serotonin syndrome? A clinical entity that consists ofhyperthermia, muscle rigidity, andaltered mental status. This syndromeis seen when MAOIs are combinedwith SSRIs (mostly), Demerol,pseudoephedrine, and othermedications that can increaseserotonin and norepinephrine.

When does a hypertensive crisis occur Hypertensive crisis originates whenin the context of MAOI use? patients who are on MAOIs ingest food

rich in tyramine, like wine and cheese,or take medications such assympathomimetics, bronchodilators,DOPA, etc.

When are MAO inhibitors particularly Atypical depression (those withefficacious? increased appetite and sleep) and

treatment-resistant depression

Are there any other ways to treat Yes, by using psychotherapy ordepression? electroconvulsive therapy in severe

depression

What type of psychotherapy is used Cognitive-behavioral therapy, whichto treat depression? attempts to recognize negative thoughts

or behaviors and then tries to changethemPsychodynamic psychotherapy, whichfocuses on self-understanding andinner conflictsInterpersonal therapy, which examinesthe patient’s problems in relation totheir symptoms and how to deal withthose problems

98 Deja Review: Behavioral Science

Page 114: Deja review   behavioral science

What is electroconvulsive therapy It is the induction of a generalized(ECT)? seizure by applying electric currents

to the brain.

When is ECT used? It is mainly indicated for patients withrefractory or psychotic depression, butit can also be used for the treatment ofmania.

What are the main side effects Short-term memory lossof ECT?

What are the DSM-IV-TR diagnostic A 1-week period of elevated,criteria for a Manic Episode? expansive, or irritable mood (or less

if hospitalized) with at least threeof the following (four if moodirritable):Grandiosity or inflated self-esteemDecreased need for sleepMore talkativeFlight of ideas/racing thoughtsDistractibilityIncreased goal-directed activityInvolvement in pleasurable and riskyactivities

What is the difference between Hypomanic episodes last 4 days orhypomania and mania? more and do not require hospitalization

or involve psychotic features. Ingeneral, hypomania is less severethan mania.

What is a mixed episode? A 1-week or longer period in whichcriteria for both manic and majordepressive episodes are met.

How do you diagnose Bipolar I The patient has one or more manic orDisorder? mixed episodes. Usually there is also a

depressive episode, but this is notrequired for the diagnosis.

Mood Disorders 99

Page 115: Deja review   behavioral science

Table 13.3 Mood Stabilizers for Chronic Treatment of Bipolar Disorder and UniqueProperties (Either I or II)

Generic Name Brand Name Side Effects Other

Carbamazepine Tegretol, Sedation, GI, Induces Carbatrol, reversible mild metabolism ofEquetro leukopenia, itself and other

reversible mild medications,increase in LFTs, decreasing itstremor, own level duringhyponatremia therapy

Associated withaplastic anemia andagranulocytosis

Must monitorlevels

Lamotrigine Lamictal Rash may occur Good forin 10% of depressed state,individuals, may rapid cycling,rarely and mixed statesprogress to Start low and goStevens-Johnson slow to avoid rashsyndrome (rare) No levelhas been monitoringreported required

Lithium Eskalith Tremor, Narrownephrogenic therapeuticdiabetes index, can causeinsipidus, toxicity with ↑hypothyroidism, sweating,GI symptoms, NSAIDs, orteratogen, thiazides weight gain diuretics

Must monitorlevels

Valproic acid Depakote Rare fatal Inhibits hepatotoxicity, metabolism ofagranulocytosis other

Teratogen: folic medicationsacid antagonist (↑ their level)→neural tube Must check LFTsdefects and monitor

Benign elevation levelof transaminases,GI side effects,sedation, tremor,alopecia,weight gain

Abbreviation: LFTs, Liver function tests; NSAIDs, nonsteroidal anti-inflammatory drugs.

100

Page 116: Deja review   behavioral science

How do you diagnose Bipolar II The patient has one or more majorDisorder? depressive episodes, plus one or more

hypomanic episodes (but never anymanic episodes).

Are there any drugs that induce mania? Yes, steroids and appetite suppressantsare the main culprits. Cocaine (crack)and amphetamines may also induce it.Previously undiagnosed bipolarpatients who have only been treatedfor a major depression may have theirmanic episodes precipitated bytreatment with antidepressants.

What is cyclothymic disorder? Cyclothymic patients meet criteria fordysthymic disorder and experiencehypomanic episodes.

What is the prevalence of bipolar Bipolar I has a lifetime prevalence ofdisorder? 0.5% to 1% and a male to female ratio

of 1:1. While, bipolar II has a lifetimeprevalence of 0.5% and is more commonin women than in men.

CLINICAL VIGNETTES

You are the third-year medical student on the psychiatry consult service. You andyour resident are called to the ER for a patient who was recently admitted to theinpatient psychiatry service for depression and suicidal ideation. While in thehospital, he complained of having trouble sleeping and was given a medication tohelp both his depression and sleep problems. The patient also received aprescription for this medication when he was discharged. He now presents to theER with a 3-hour history of a painful erection. What drug was the patient mostlikely given?

Trazodone

A patient you have been treating for depression with SSRIs comes to you for acheckup. When you ask how he is he says “The last 3 days have been fantastic!”and launches into a very long and tangential story about what he has been doing.The patient has been very productive at work, though that might have to do morewith the small amount of sleep that he has been getting, but he claims that he isfar too strong to need sleep. What is your new diagnosis?

Bipolar, type II. This meets criteria for a hypomanic episode, but because of thelack of psychosis or need for hospitalization it is not yet mania. He is also veryfunctional during this time.

Mood Disorders 101

Page 117: Deja review   behavioral science

A 29-year-old woman comes to the ER worried about a new rash. She has neverhad a rash like this before. She has no risk factors that you can identify and hasn’tbeen hiking in any areas with poison oak. She says she has just started therapywith an anticonvulsant drug to treat her bipolar disorder. She has no allergies thatshe knows of.

What drug is she likely taking?

Lamotrigine, a maintenance therapy for bipolar disorder. She is experiencing adrug rash, which is common with this drug.

What should you do at this point?

Stop the medication

What is the serious complication you would like to avoid?

Stevens-Johnson Syndrome. Though rash is common, rarely some patients willprogress to Stevens-Johnson Syndrome.

102 Deja Review: Behavioral Science

Page 118: Deja review   behavioral science

C H A P T E R 1 4

Anxiety Disorders

103

What are the most prevalent psychiatric Anxiety disordersdisorders?

What are the major types of anxiety Generalized anxiety disorder (GAD)disorders? Panic disorder

Social phobiaSpecific phobiaObsessive-compulsive disorderAgoraphobiaPosttraumatic stress disorder (PTSD)Acute stress disorder (ASD)

What is the most prominent feature The presence of a sense of impendingof all anxiety disorders? doom or threat

What differentiates anxiety disorders Anxiety out of proportion to actualfrom healthy anxiety? threat which causes significant distress

or impairment

What are the DSM-IV-TR diagnostic Six months of excessive anxiety andcriteria for GAD? worry, more days than not, that are

difficult to control. Plus three ormore of the following:RestlessnessFatigueTensionSleep disturbanceIrritabilityDecreased concentrationNote: The worry may not be aboutsymptoms of another disorder, eg,worrying about having a panicattack is panic disorder, not GAD.

Page 119: Deja review   behavioral science

What GI disease is associated Irritable bowel syndrome; 50% ofwith GAD? patients also have GAD.

What is the male to female ratio of 1:2GAD prevalence?

What are important nonpsychiatric Substance-induced anxietydifferentials to consider for GAD? Substance withdrawal anxiety

General medical conditionsPheochromocytomaHyperthyroidismElectrolyte abnormalities

What is the initial pharmacological Selective serotonin reuptake inhibitortreatment for GAD? (SSRI)

VenlafaxineBuspironeMirtazapine

What medications give rapid relief, Benzodiazepinesbut have significant long-term abusepotential?

PANIC DISORDER

What are the DSM-IV-TR criteria for a Four or more symptoms, which comepanic attack? up abruptly:

AnxietyPalpitationsSweatingDizzinessTremblingShortness of breathChest painNauseaFeeling of chokingChillsFear of dyingFear of going crazy or losing controlFeeling detached from one’s self

What characterizes panic disorder? Recurrent spontaneous panic attacksand anxiety about future attacks, theirimplications, or changing behavior toavoid attacks

104 Deja Review: Behavioral Science

Page 120: Deja review   behavioral science

What differentiates panic attacks from A panic disorder develops when thepanic disorders? patient has recurrent panic attacks and

has anticipatory fear about future panicattacks.

What is the male to female ratio for 1:3panic disorder?

What is the age of onset of panic Mid-twentiesdisorder?

Is family history of panic disorders Yes. Evidence does support anrelevant? upregulation of adrenergic output

responsible for stimulating anxietycenters in the brain which predisposescertain people to panic attacks.

What are the most common psychiatric Agoraphobia. Patients have anticipatorysequelae of panic attacks? fear which prevents them from venturing

outside alone.

What is the most effective psychotherapy Cognitive behavioral therapy includingfor panic disorder? relaxation techniques

What is the pharmacologic treatment SSRIs (first-line)of panic disorder? Tricyclic antidepressant (TCA),

monoamine oxidase inhibitors (MAOIs)(second-line)

What are the important nonpsychiatric Arrhythmiasdifferential diagnoses for panic Anginadisorders?

HypoxiaHyperthyroidismSubstance-induced intoxication orwithdrawal

What phobia may occur with AgoraphobiaPanic Disorder?

OBSESSIVE-COMPULSIVE DISORDER

What are the diagnostic criteria for Presence of obsessions and/or compulsiveobsessive-compulsive disorder (OCD)? behavior that are time consuming and

cause significant distress or impairmentof functioning

What is an obsession? Persistent, recurrent thoughts whichcause irrational anxiety

Anxiety Disorders 105

Page 121: Deja review   behavioral science

What is a compulsion? A repetitive behavior that reducesanxiety—behavior may not be directlyrelated to the obsession

What differentiates OCD from OCD patients are aware that theirobsessive-compulsive personality anxiety and compulsions aredisorder (OCPD)? unreasonable. OCPD patients are

orderly, inflexible, and perfectionistic—but see nothing wrong with it.

What is the male to female ratio 1:1for OCD?

What is the age of onset of OCD? Late adolescence to early adulthood

What common psychiatric diagnoses Major depressionare associated with OCD? Generalized Anxiety Disorder

Tourette syndromeNote: It is common for patients withTourette to have OCD, but it is notnearly as common for patients withOCD to have Tourette.

What is the most effective psychotherapy Cognitive-behavioral therapyfor OCD? (exposure-response prevention,

flooding, thought stopping)

What is the pharmacologic treatment High-dose SSRI (first-line treatment)of OCD? Clomipramine (second-line treatment)

What are the important differential Specific phobiadiagnoses for OCD? GAD

Body dimorphic disorderTrichotillomania

SOCIAL PHOBIA

What are the diagnostic criteria Persistent fear of social situationsfor social phobia? Anxiety or panic attack when exposed

to social situationAvoidance or distress of feared situationresults in functional impairment

What characterizes social phobias? Excessive, irrational fear of public orsocial situations, in which the patientcould be scrutinized for his or herperformance.

106 Deja Review: Behavioral Science

Page 122: Deja review   behavioral science

What differentiates social phobias Social phobias involve fear of publicfrom specific phobias? situations where scrutiny could occur.

On the other hand, specific phobiasinvolve fear of specific objects orsituations not associated with scrutiny.

What is the male to female ratio 1:1for social phobia?

What is the age of onset of Adolescence following childhoodsocial phobias? shyness

What are the most common Fear of public speakingsocial phobias? Fear of public performances

Fear of answering questions in class

What is the most effective Cognitive-behavioral therapy includingpsychotherapy for social phobias? relaxation techniques

What is the pharmacologic treatment SSRIs (first-line treatment)of social phobias? Beta-blockers

What medications are effective Benzodiazepines—should be avoided infor immediate relief of symptoms, those with substance abuse potentialbut must be used cautiously?

What are the important differential Specific phobiasdiagnoses for social phobia? Panic disorder

OCD

POSTTRAUMATIC STRESS DISORDER

What are the DSM-IV-TR criteria for Exposure to a traumatic event involvingposttraumatic stress disorder (PTSD)? threat to self or others, plus 1 month of:

Reexperiencing the event (flashbacks,dreams, reliving the trauma, etc)Avoidance of situations and thoughtsassociated with the traumaIncreased arousal

What is increased arousal in PTSD? Anger or irritabilityInsomniaHypervigilanceIncreased startle responseDifficulty concentratingNote: Diagnosis requires two or moreof above.

Anxiety Disorders 107

Page 123: Deja review   behavioral science

What differentiates PTSD from acute Patients with PTSD have symptoms forstress disorder? >1 month that can begin >1 month after

the traumatic event occurs. Patientswith acute stress disorder have symptomslasting from 2 days to 1 month whichbegin within 1 month of traumaticevent.

What differentiates PTSD from PTSD involves life-threatening eventsadjustment disorder? (rape, war, etc)

Adjustment disorders involve non-life-threatening events (divorce, death ofothers)

What is the male to female ratio 1:2for PTSD?

How is PTSD classified? Acute PTSD: Symptoms last <3 months.Chronic PTSD: Symptoms last>3 months.Delayed-onset PTSD: Symptoms begin>6 months after the life-threateningevent.

What is the most effective psychotherapy Cognitive-behavioral therapy (CBT)for PTSD? and support groups

What is the pharmacologic treatment SSRIs (first-line treatment)of PTSD? Mood stabilizers

What medication is particularly effective Prazosin (alpha-blocker)for PTSD nightmares?

What are important differential Acute stress disorderdiagnoses for PTSD? Adjustment disorder

OCDMalingering

SPECIFIC PHOBIA

What are the symptoms of a specific Fear of specific situationsphobia? Anxiety

PalpitationsSweatingHeadacheRestlessness

108 Deja Review: Behavioral Science

Page 124: Deja review   behavioral science

What characterizes specific phobias? Excessive, irrational fear of specificobjects or situations

What differentiates specific phobias Specific phobias involve fear of specificfrom social phobias? objections or situations not associated

with being scrutinized.

What is the male to female ratio for 1:2specific phobia?

What is the age of onset of specific Mostly childhood but can begin atphobias? anytime

What are the most common types of Animalsspecific phobias? Storms

HeightsIllnessInjury

What is the most effective psychotherapy Floodingfor specific phobias? Gradual desensitization

HypnosisCBT

What is the pharmacologic treatment Benzodiazepines for certain phobiasof specific phobias? such as flying

What are important differential Social phobiadiagnoses for specific phobias? OCD

GADPanic disorder

ADJUSTMENT DISORDER WITH ANXIETY

What is the main characteristic of Emotional or behavioral symptomsadjustment disorder with anxiety? associated with an identifiable stressor

During what time period should the Within 3 months of initial stressorsymptoms present after stressor occurs?

When should symptoms of adjustment Within 6 months of initial symptoms ifdisorder with anxiety resolve? stressor removed

What are common causes of adjustment Divorcedisorder? Relocation

Attending a new school

Anxiety Disorders 109

Page 125: Deja review   behavioral science

What should be ruled out before Bereavementdiagnosing adjustment disorders? Any Axis I diagnosis

How does adjustment disorder with Occupational impairmentanxiety present? Social impairment

Scholastic impairment

What are important differential Bereavementdiagnoses? Major depressive disorder

Acute stress disorderPTSD

CLINICAL VIGNETTES

A 47-year-old woman describes herself as a “worrier for most of my life.” Shedoesn’t worry about anything in particular, but she lays awake at night thinkingabout things. It’s exhausting and she can’t seem to get anything else done—ofcourse, it doesn’t help that she can’t concentrate anyway. What is the most likelydiagnosis?

Generalized Anxiety Disorder

A 32-year-old man was involved in an industrial accident that killed his bestfriend. He was right next to him when it happened, and he remembers every bit ofit. In fact, sometimes he feels like he relives it. He has recurrent nightmares aboutthe event, can’t seem to get to sleep, and can’t face going back to work.

What is the most likely diagnosis?

PTSD

How would you treat his nightmares?

His nightmares may be responsive to medical therapy with Prazosin.

A 26-year-old female presents to your clinic with complaints about episodes of shortnessof breath, palpitations, tingling around her mouth, and blurry vision. She has onlyexperienced two of these episodes, but she fears that she may have more. She asksyou to start her on some medication to help with these fears and the episodesthemselves. Her last menstrual period was 7 weeks ago, and she has a history ofcocaine abuse.

What class of medication would be most useful in providing immediate reliefof symptoms for this patient?

Benzodiazepine

Why might this class of medications be contraindicated in this patient?

She may be pregnant (most benzodiazepines are pregnancy category D) and hasa history of substance abuse making treatment with benzodiazepines problematic.

110 Deja Review: Behavioral Science

Page 126: Deja review   behavioral science

C H A P T E R 1 5

Cognitive Disorders

111

Name the three major categories 1. Deliriumof cognitive disorders. 2. Dementia

3. Amnestic disorders

DELIRIUM

What is delirium? Delirium is a disturbance of consciousnessand attention that usually develops overa short period of time. The confusionand memory impairment is not betteraccounted for by a preexisting dementia.

What causes delirium? Any disorder that causes a disruptionof brain physiology, most commonly amedical or surgical condition, medication.The cause is often multifactorial. Elderlypatients and those with underlyingdementia are at higher risk.

While most substances of abuse can Alcohol (delirium tremens),cause delirium with acute intoxication, benzodiazepines, and barbiturateswhich three are most likely to causedelirium related to withdrawal?

What factors predispose to delirium? Acute medical illnessAge: elderly and young childrenPreexisting brain damage: dementia,cerebrovascular disease, tumorA history of deliriumAdvanced Cancer

What are some medications physicians A long list, including:give patients that contribute Anticholinergics (including Benadryl)to delirium?

BenzodiazepinesLevodopa and other dopaminergicsOpioidsCardiac medications, eg, Beta-BlockersMany antibiotics

Page 127: Deja review   behavioral science

How common is delirium? Very common. Roughly 15% to 20% ofgeneral hospital patients will have anepisode of delirium. In patients over theage of 65, the prevalence can be as highas 30%!

Clinically, what is the hallmark Altered level of consciousnessof delirium? (especially attention and level of

arousal). It typically develops over aperiod of hours to days and mentalstatus alterations wax and wane duringthe day, with periods of lucidity.

In addition to alterations in Altered sleep-wake cycleconsciousness, what other features Perceptual disturbanceare often found in a patient

Impaired memory and orientationwith delirium?Nocturnal worsening of symptomsPsychomotor agitation

How do you test for delirium? Though there is no lab test for delirium,a good cognitive tool like the mini-mental status examination can helpidentify delirium. Cognitive disturbancefrom delirium tends to fluctuate.

How do you treat delirium? Treat any medical cause, stop medicationsthat may be contributing (if possible),orient the patient frequently, and placein a well-lit room during the day.

While treating the cause of a delirium, Low doses of antipsychotics, likehow do you manage a delirious haloperidol or an atypical antipsychoticpatient? can be helpful for agitation and

hallucinations.

DEMENTIA

What is the core symptom of dementia? Memory impairment

In order to diagnose dementia, what At least one of the following:must be present in addition to memory Aphasia (language and naming problems)impairment?

Apraxia (impaired ability to do learnedmotor tasks, like using objects)Agnosia (difficulty recognizing oridentifying objects)Disturbance of executive function (theability to plan, organize, and carry outtasks, judgment)

112 Deja Review: Behavioral Science

Page 128: Deja review   behavioral science

How does dementia differ from the As we age, we are less able to learn newnormal memory changes of aging? information, and we process information

at a slower speed.However, these changes do not normallyinterfere with the basic functioning.

When diagnosing dementia, what other It is crucial that you do not miss adisorders in your differential are key delirium or a depression. In the elderly,to rule out? it is not uncommon for them to report

multiple memory complaints. If youmisdiagnose this as dementia, you willmiss a potentially reversible cause ofmemory impairment. Likewise, if youmiss a delirium, you may miss apotentially serious medical problem.Additionally, there are several potentiallyreversible causes of dementia that youshould look for, including: neurosyphilis,vitamin B12, thiamine, and folatedeficiencies, and normal pressurehydrocephalus.

What is the prevalence of dementia? Incidence/prevalence increases withage. The prevalence is approximately1.5 % in those over 65 years of age. Theprevalence increases to 20% after age 85.

What is the most common type Alzheimer Disease (AD) representsof dementia? about 50% to 60% of dementias. The

second most common form is vasculardementia (formerly multi-infarctdementia)Others include front temporal (Pickand Creutzfeldt-Jakob), Parkinson,Huntington, and humanimmunodeficiency virus (HIV)dementias.

What is the classical clinical course Slow, gradual onset of memory lossfor Alzheimer disease? and cognitive impairment (often there

are problems with judgment, moodsymptoms, and behavioral disturbancesas well). The disease is progressive anddeath usually occurs within 3 yearsafter diagnosis.

How does this differ from the course Vascular dementia classically has aof vascular dementia? stepwise decline, as opposed to the slow

and steady decline in Alzheimer. Onsetof deficits may be abrupt, and with goodcontrol of cardiovascular risk factors thecourse may remain relatively stable.

Cognitive Disorders 113

Page 129: Deja review   behavioral science

What are the major risk factors for Age, family history, Apo E4 allele, andAlzheimer disease? Down syndrome

On postmortem examination, what Neurofibrillary tangles and senilechanges are normally seen in the (amyloid) plaquesbrain of an Alzheimer disease patient?

What areas of the brain show cell loss While there is also often global corticalin Alzheimer disease? atrophy, neuronal degeneration is

classically in the cholinergic neuronsof the nucleus basalis of Meynert.

What class of medications is used Cholinesterase inhibitors such asto slow the progression of Alzheimer donepezil, rivastigmine, anddisease? galantamine may be used. Memantine

(Namenda) is an NMDA (N-methyl-D-aspartate) receptor antagonist thatshows promise for slowing progressionin AD.

114 Deja Review: Behavioral Science

Table 15.1 Delirium vs Dementia

Delirium Dementia

Onset Develops quickly (hours to days) Develops over weeksto years

Key feature Impaired attention and level of Impaired memoryconsciousness with normal level

of consciousness

Course Fluctuates within the course Usually stable withinof a day with lucid periods a day

Worsens at night May worsen at night(sundowning)

Occurrence Most common in elderly and Increases with ageyoung children

Psychiatric Hallucinations and delusions Hallucinations andSymptoms may be present delusions may be

present

Physical Abnormal EEG Normal EEGfindings Acute medical illness No acute medical

illness

Prognosis Symptoms tend to resolve with Usually progressivetreatment of underlying cause

Page 130: Deja review   behavioral science

AMNESTIC SYNDROMES

How do amnestic syndromes differ In amnestic syndromes, the disturbancefrom dementia? of function is isolated to memory, while

other cognitive functions remainrelatively intact (unlike dementia).

Which brain structures are affected The bilateral mediotemporal structuresin amnestic syndromes? (eg, mammillary bodies, hippocampus,

fornix)

Damage to mediotemporal structures Thiamine deficiency. Often this isis associated with what vitamin related with chronic alcohol abusedeficiency? (Korsakoff syndrome)

Name at least four other etiologies 1. Traumatic brain injuryof amnestic syndromes. 2. Herpes encephalitis

3. Cerebrovascular disease4. Hypoxia

CLINICAL VIGNETTES

A 79-year-old African American woman has dementia. Her son has heard about thegenetic basis for Alzheimer disease and is worried about his risk of developing thedisease. Looking over her chart you see that she was high functioning 10 years priorand then first suddenly became rather mildly cognitively impaired about 7 yearsago. She continued at that level until 5 years ago when she developed significantmemory problems and needed some part-time help at home. Two years ago sheagain got worse and required assisted living. What do you tell her son about hisrisks of Alzheimer disease?

His risk is the same as the normal population. The patient’s history with a“stepwise decline” is suggestive of vascular dementia, not AD.

You are on the psychiatric consult service and called to see a psychotic patient inthe ICU. The resident there tells you that the patient—a 57-year-old woman withno prior psychiatric history—is having new-onset schizophrenia. She seemed finethis morning, but this evening she has hallucinations and is yelling at the nursingstaff. What is the likely diagnosis in this patient? What can be done to help, asidefrom medications?

This patient most likely has delirium and almost absolutely does not haveschizophrenia. You should be wary of diagnosing an older individual with a new-onset psychotic disorder, especially when underlying medical illness is activelyinvolved. Delirium is so common in acutely sick patients that it is called “ICUpsychosis.” First and foremost, trying to alleviate the medical condition (or stoppingthe offending medication) which has precipitated the delirium is the first courseof action. Environmental factors may help, such as cues to help orient patients totime, date, and place.

Cognitive Disorders 115

Page 131: Deja review   behavioral science

A 57-year-old man with a history of end-stage liver disease and peptic ulcersunderwent surgery to repair a perforated ulcer 2 days ago. He is agitated, confused,and he believes bugs are crawling on him. He appears floridly delirious and youbegin his workup. His blood work reveals no evidence of infection, his bloodpressure is becoming more labile, and heart rate is increasing. Your attendingphysician shows up and is very concerned that he may seize. What is the mostlikely etiology for the delirium?

Alcohol withdrawal

116 Deja Review: Behavioral Science

Page 132: Deja review   behavioral science

C H A P T E R 1 6

Somatoform Disorders

117

What are the primary types Somatization disorderof somatoform disorders? Conversion disorder

HypochondriasisBody dysmorphic disorderPain disorder

What is factitious disorder? A disease in which the patientintentionally feigns illness to assumethe sick role, but with no other obviousmotivation (aka Munchausensyndrome)

What is malingering? Intentional feigning of illness for thepurpose of gaining a conscious reward

What is the primary difference between Somatoform disorders are unconscioussomatoform disorders and factitious behaviors, whereas behaviors indisorder or malingering? factitious disorders and malingering

are consciously derived by the patient.

Table 16.1

Conscious Behavior? Motivation Apparent?

Somatoform No No

Conversion No Yes

Factitious Yes No

Malingering Yes Yes

Are there any physical examination No. Disease etiology must be ruled outfindings which are common in these patients; however, there is noin somatoform disorders? medical condition that can account for

their symptoms.

Page 133: Deja review   behavioral science

What are some disorders with SLE, Multiple Sclerosis, Myasthenianonspecific symptoms that should gravis, Hyperparathyroidism, Thyroidbe thought of before somatoform disturbance, Porphyria, Malignancydisorder is diagnosed?

Should patients with somatoform No. Supportive treatment withdisorders be told that they are suggestions that psychotherapy mayimagining their symptoms? alleviate their distress is associated with

a better prognosis. Remember that painis a subjective experience.

What is a good way to frame Reassurance that there is no seriousthe possibility of a lack of general underlying medical cause, but withoutmedical problem? dismissing their symptoms—this may

alleviate some anxiety.

What is a good statement to make “Many of my patients with similarwhen discussing the possibility symptoms have found a lot of benefitof psychiatric treatment with a patient? from...” [SSRIs, psychotherapy, etc]

SOMATIZATION DISORDER

What are the diagnostic criteria Patients must manifest four pain,of complaints seen in somatization two gastrointestinal, one sexual, anddisorder? one neurologic symptoms, all of which

cannot be fully explained by medicaletiology.

What is the age of onset of Somatic complaints must begin priorsomatization disorder? to age 30.

What are common patterns of behaviors These patients often have had multipleseen in these patients? exploratory surgeries and visit multiple

doctors.

Can the symptoms occur in the presence Yes, however, the complaints will oftenof a true medical condition? be in excess of what would be normally

expected.

What is the prevalence of somatization 0.2% to 2% in women and <0.2% in mendisorders in the United States?

What are risk factors for somatization High socioeconomic statusdisorder? History of abusive and failed relationships

Family history of somatization

What is the most common first Complaints relating to mensesmanifestation of this disorderin women?

118 Deja Review: Behavioral Science

Page 134: Deja review   behavioral science

What is the treatment for somatization Regular brief appointments with thedisorders? primary care provider are often useful

in reassuring these patients. Unnecessarylaboratory tests or procedures should notbe performed. In addition, psychotherapymay be used.

CONVERSION DISORDER

From which body system are Neurologic (motor and/or sensory)complaints derived from in conversion disorders?

What are the most common Sudden onset of blindnessmanifestations seen? Paralysis

ParesthesiasSeizures

What is a common presentation? Dramatic onset of symptoms that areoften physiologically impossible

What are common findings on physical Abnormalities do not have anatomicalexamination of these patients? distribution and neurologic examination

is normal.

What is a common association Often stressful life events precede thewith the onset or exacerbation development of symptoms.of symptoms?

What medical disorder can present Multiple sclerosissimilarly to a conversion disorder?

What percentage of individuals Very few—approximately 4%diagnosed with a conversion disorderin fact have a true neurologicalcondition?

Do patients with true conversion No. Patients with sudden onset ofdisorders sustain injury as a result blindness do not run into objects,of their condition? and those with paralysis may still

inadvertently move when distracted.

Are patients with conversion disorder No. These patients are often calmdistressed over their condition? regarding their pseudoneurologic

deficits, termed la belle indifference.

Note: This is not diagnostic of conversiondisorder.

Somatoform Disorders 119

Page 135: Deja review   behavioral science

Do patients with conversion disorder Nointentionally derive any gain fromtheir symptoms?

What is the prognosis and duration Symptoms typically remit within 2 weeks,of symptoms seen in conversion with favorable prognosis seen in paralysisdisorders? and blindness, and a poor prognosis

associated with seizures.

What is the treatment for conversion Reassurance that the symptoms willdisorders? improve usually results in resolution of

the symptoms (self-limiting), however,conversion disorder often reoccurs later.Psychotherapy can be helpful.

HYPOCHONDRIASIS

What is the main characteristic Preoccupation with having a seriousof hypochondriasis? disease, despite medical reassurance

of health status

Are there true physical symptoms Yes, however, the symptoms areseen in this disorder? misinterpreted by the patient as being

of a greater significance.

What are the most common presenting Nauseasymptoms seen in hypochondriasis? Abdominal pain

Chest painPalpitations

Is there a gender predominance seen No, men and women are equallyin hypochondriasis? affected.

What are common associations with Often the person has experiencedthe development of hypochondriasis? serious illness in childhood or knows

someone who has died or sufferedthrough a serious medical condition.

What are common behaviors seen Doctor shopping is common, as thesein these patients? patients are resistant to suggestions that

there is no significant medical etiologyto their symptoms.

What is the prevalence of 1% to 5%hypochondriasis?

What is the treatment for Group therapy and frequent reassurancehypochondriasis? with regular but brief visits to primary

care physician

120 Deja Review: Behavioral Science

Page 136: Deja review   behavioral science

BODY DYSMORPHIC DISORDER

What is the main characteristic Preoccupation with a defect in physicalof body dysmorphic disorder? appearance

Is the defect always imagined No. In some patients the defect may beby the patient? imagined, but in others an exaggeration

of a true physical feature may be present.

What are the two components of body 1. Perceptual: Perceptual relates to thedysmorphic disorder? accuracy of the individual’s body.

2. Attitudinal: Attitudinal relates to thefeelings the person has toward his orher body.

What are risk factors for the development Family history of a mood disorder orof body dysmorphic disorder? obsessive-compulsive disorder

Where do these patients often present? Often these patients present todermatologist and plastic surgeons.

What are the most common features Facial featuresviewed as defective in these patients? Hair

Body build

What is the most common comorbid Depressionpsychiatric disorder associated withbody dysmorphic disorder?

What are some common behaviors Excessive groomingseen in this disorder? Avoidance of mirrors

Excessive exerciseAvoidance of public activities

What other diagnoses must be Anorexia nervosaconsidered in the differential? Gender identity disorder

Narcissistic personality disorder

Do surgical procedures and alterations No. These treatments tend to worsentend to improve the patient’s view the disorder, leading to intensified orof his or her physical defect? new preoccupations with physical

appearance.

What is the recommended treatment Antidepressants (such as selectivefor body dysmorphic disorder? serotonin reuptake inhibitors [SSRIs])

(only if comorbid mental illnesses suchas depression or anxiety are present)and cognitive behavioral therapy

Somatoform Disorders 121

Page 137: Deja review   behavioral science

FACTITIOUS DISORDER

What is the primary characteristic Intentional simulation of illnessof factitious disorder?

Are patients with factitious disorder Yes. This is the distinguishingor malingering aware of the false characteristic between factitiousnature of their somatic complaints? and somatoform disorders.

How is factitious disorder different The only obvious conscious goal offrom malingering? factitious disorder is to assume the “sick

role”—there is no clear benefit or gain.

Which gender is more prevalent Femalein factitious disorders?

What occupation has the highest Health-care workersprevalence of factitious disorders?

What disease presentations are often Hematuria (from adding blood to urineseen in factitious disorder? or from the use of anticoagulants) and

hypoglycemia (from insulin injection)

What laboratory finding would indicate Low C-peptide levelself-injection of insulin?

What are common behaviors seen in Requests for analgesicspatients with factitious disorder? Extensive knowledge of medical

terminologyEager desire to undergo medicalprocedures and operationsTraveling to different locations,hospitals, emergency rooms, etc

What are common histories of patients Patients often have dramatic historieswith factitious disorder? with extensive details about their

symptoms.

What is the appropriate treatment Recognition and confrontation in afor factitious disorder? nonaccusatory manner

What is the prevalence of factitious 0.5% to 0.8%—very difficult to determinedisorder?

Which gender is most commonly seen Femalesin factitious disorder?

What is a well-known variant Factitious disorder by proxy (akaof factitious disorder? Munchausen syndrome by proxy)

122 Deja Review: Behavioral Science

Page 138: Deja review   behavioral science

What is the primary characteristic Intentional simulation of illness inof factitious disorder by proxy? another person

Who is the most common A parent (most often a mother) oftenperpetrator seen in factitious stimulating illness in her childdisorder by proxy?

What is the treatment for factitious Same as in factitious disorder, however,disorder by proxy? if a child is involved, the case should be

managed as child abuse and reported tothe appropriate agencies.

MALINGERING

What is the primary characteristic Intentional production of symptoms forof malingering? secondary gain

How is malingering different from The goal of malingering is to obtain afactitious disorder? concrete or material gain.

What are common scenarios in which Individuals wanting to avoid jail timemalingering is often seen? or military recruitment, seeking financial

compensation, etc

Which gender is most prevalent Malesin malingering?

What is Ganser syndrome? A variant of malingering in whichpatients give ridiculous answers toquestions in order to avoidresponsibility for their actions

In what populations is Ganser Prison inmatessyndrome most commonly seen?

CLINICAL VIGNETTES

A 71-year-old man comes to your office for evaluation of a right foot paralysis. Hecomplains that he is unable to move his right foot ever since he was involved in acar accident, during which he was driving and the teenaged passenger of the othervehicle was seriously injured. His neurologic examination is normal. What is themost likely diagnosis?

Conversion disorder. Note the stressful event, normal neurologic examination,and the fact that it is his driving foot that is affected.

Somatoform Disorders 123

Page 139: Deja review   behavioral science

A 26-year-old female nurse is admitted after being found by a friend unconsciousin the hallway. On finger stick, her glucose is 17. She has no history of diabetes.

What psychiatric disorder might you suspect if medical workup is negative?

Factitious disorder. Note conscious behavior (likely insulin overdose), but noobvious motivation/gain.

What laboratory testing can you do, and how would it support your diagnosis?

A C-peptide level would be useful, and if it was low you would suspect exogenousinsulin overdose, therefore supporting your diagnosis.

A 37-year-old man presents to the ED complaining of “10/10 right flank painradiating to the testicle.” He specifically requests 2 mg of IV hydromorphone andstates he is allergic to both NSAIDs and Tylenol. A nurse reports that she saw himputting drops of blood into his urine collection cup. What is the most likelydiagnosis?

Malingering

A 12-year-old girl and her parents present to your office with a chief complaint ofdysphagia for 3 days. According to the girl’s parents, who are very concerned andanxious, the girl choked while eating lunch 3 days ago and the Heimlich maneuverwas performed to expel the food. Since then she has been unable to swallow anysolids or liquids. Additionally, she is unable to swallow any of her normal oralsecretions, and has been spitting constantly in a container. On examination, thegirl is very quiet, calm, and somewhat unconcerned about her condition.Assuming physical examination is normal and an upper gastrointestinal (GI)series fails to reveal any pathology, what is the likely diagnosis?

Conversion disorder

124 Deja Review: Behavioral Science

Page 140: Deja review   behavioral science

C H A P T E R 1 7

Personality Disorders

125

What is personality? Personality is “the set of characteristicsthat defines the behavior, thoughts, andemotions of individuals,” and ispersistent over time.

What constitutes a personality Personality disorders occur whendisorder? a particular feature or trait of an

individual’s personality becomesinflexible or maladaptive and impairssocial, occupational, or personalfunctioning.

What diagnostic criteria are common The patterns are pervasive andto all personality disorders? begin by early adulthood, lead to

significant distress or impairment,and are not better explained by anAxis I disorder.

Which axis of the DSM-IV-TR They are Axis II diagnoses.do personality disorderdiagnoses fall under?

How are the personality disorders Into clusters—A, B, and Cdivided?

Which personality disorders fall Paranoid, schizoid, and schizotypalunder cluster A?

What is the common theme among They are considered odd or eccentriccluster A disorders? traits.

Which personality disorders fall under Histrionic, narcissistic, antisocial, andcluster B? borderline

What is the common theme among They are described as dramatic,cluster B disorders? emotional, and erratic.

Page 141: Deja review   behavioral science

Which personality disorders fall under Avoidant, dependent, andcluster C? obsessive-compulsive

What is the common theme among They tend to be anxious and fearful.cluster C disorders?

What is a good way to remember these They are Mad, Bad, and Sad.clusters? A = Mad, B = Bad, C = Sad.

CLUSTER A: THE MAD

How is schizoid personality disorder Patients have “a pervasive pattern ofdescribed? detachment from social relationships

and a restricted range of expression ofemotions in interpersonal settings.”

What are the symptoms/criteria for Four or more of the following:schizoid personality disorder ? No desire or enjoyment of close

relationshipsChoice of solitary activitiesLittle interest in having sexual experiencesEnjoyment of few activitiesLack of close friendsApparent indifferenceEmotional coldness/detachment/flattened affect

How is paranoid personality disorder Paranoid personality disorder involvesdefined? a “pervasive and unwarranted suspicion

and mistrust of people, hypersensitivityto others, and an inability to deal withfeelings.”

What are the criteria/symptoms Four or more of the following:of paranoid personality disorder? Suspicion of exploitation or

deceitfulness on the part of othersPreoccupation with unjustified doubtsReluctance to confide in othersReading hidden demeanings orthreatening meanings into benignremarks or eventsPersistently bearing grudgesPerception of attacks on his or hercharacter or reputation to which heor she reacts quickly/angrilyRecurrent suspicions

126 Deja Review: Behavioral Science

Page 142: Deja review   behavioral science

What is the best treatment for paranoid Psychotherapy and possiblypersonality disorder? antipsychotic medications to manage

agitation and paranoia (overt delusionsare not usually seen)

How is schizotypal personality disorder Patients with schizotypal personalitydefined? disorder are usually described as strange

or odd in behavior, appearance, and/orthinking. However, they are neitherfrankly delusional nor psychotic.

What are the symptoms of schizotypal Ideas of referencepersonality disorder? Odd beliefs/magical thinking/believe

that they have “special powers”Unusual perceptual experiencesOdd thinking and speechSuspiciousness/paranoiaInappropriate/constricted affectOdd behavior/appearanceLack of close friendsExcessive social anxiety

What percentage of patients with 10% to 20%—ie, most do notSchizotypal Personality Disorder go on to develop schizophrenia?

CLUSTER B: THE BAD

Which personality disorders fall under Histrionic, narcissistic, antisocial,cluster B? and borderline

What is the common theme among They are described as dramatic,cluster B disorders? emotional, and erratic.

What other psychiatric diagnoses Mood disorders are quite prevalent,do patients with cluster B disorders as well as somatization disorders,often carry? and substance abuse/dependence

How are patients with antisocial Charming, but manipulative. They oftenpersonality disorder usually described? have a history of criminal activities and

many have a history of substance abuse.They show no remorse for their actions,even if others are harmed.

What prior childhood diagnosis must Conduct disorderhave been present in order to bediagnosed with antisocial personalitydisorder as an adult?

Personality Disorders 127

Page 143: Deja review   behavioral science

What are the diagnostic criteria for Three or more of the following:antisocial personality disorder? Failure to conform to social norms and

lawsDeceitfulnessImpulsivityIrritability and aggressivenessDisregard for the safety of selfor othersIrresponsibility (often unemployment)Lack of remorse for harm to others

What is the best treatment of antisocial Group therapy with setting ofpersonality disorder? boundaries for behavior, selective

serotonin reuptake inhibitors(SSRIs), and treatment of underlyingsubstance abuse/misuseif present.These individuals are very refractoryto treatment.

How is borderline personality Borderline personality disorder involvesdisorder defined? the primary feature of instability. This is

seen in terms of the patient’s self-image,interpersonal relationships, and mood.(Think Angelina Jolie’s character inGirl Interrupted.)

What defense mechanism is prevalent Splitting—eg, people are either all goodin patients with borderline personality or all bad.disorder?

What are the symptoms/diagnostic Five or more of the following:criteria of borderline personality Frantic efforts to avoid real or imagineddisorder? abandonment

Unstable or intense interpersonalrelationshipsIdentity disturbance, impulsivityRecurrent suicidal or self-mutilatingbehaviorMarked reactivity of moodChronic feelings of emptinessInappropriate/intense angerLack of control of angerTransient stress-related paranoidideation

128 Deja Review: Behavioral Science

Page 144: Deja review   behavioral science

What are the most prevalent symptoms Unstable relationships and mood/of borderline personality disorder? affect lability

What developmental characteristics Many were severely abused as children.are often present in patientswith borderline personalitydisorder?

What is the best treatment for borderline Psychotherapy and mood stabilizationpersonality disorder? with either antidepressants,

carbamazepine, or valproate.Patients may also require short-term antipsychotics for treatmentof psychosis.

What drug class should be avoided in Benzodiazepines, because of addictivepatients with borderline personality potential and with overdosedisorder?

How is narcissistic personality disorder Narcissistic personality disorder isdescribed? defined as a grandiose sense of self-

importance along with extremesensitivity to criticism. These patientshave “little ability to sympathize withothers, and are more concerned aboutappearance than substance.” (Think BillMurray’s character in Groundhog Day.)

What are the symptoms/diagnostic Five or more of the following:criteria of narcissistic personality Grandiose sense of self-importance/disorder? exaggeration of achievements and

talentsPreoccupations with ideals of success,power, brilliance, beauty, or love Belief that he or she is special or unique,and can only be understood orappreciated by high-status individualsSense of entitlementLacks empathy for othersManipulative or exploitive of othersRequires excessive admirationBelieves others envy him or her,or envies othersArrogant

What other psychiatric conditions Mood disorders and other clusterare associated with narcissistic B traitspersonality disorder?

Personality Disorders 129

Page 145: Deja review   behavioral science

What is the best treatment of Psychotherapy (either group ornarcissistic personality disorder? individual)

How is histrionic personality disorder Histrionic personality disorder can bedescribed? identified by the flamboyant/attention-

seeking behaviors of patients. They areextremely emotional, and may presentas very attractive and seductive.(Think Scarlett O’Hara in Gone withthe Wind.)

What are the symptoms/diagnostic Five or more of the following:criteria of histrionic personality Persistent need to be the center ofdisorder? attention

Inappropriately sexual/seductive/provocativeRapidly shifting, shallow emotionsBelieves relationships are more intimatethan they actually areImpressionistic speech lacking detailMelodramaticUses physical appearance to attractattentionSuggestible/easy to manipulate

What other psychiatric disorders are Mood disorders and somatizationoften associated with histrionic disorderspersonality disorder?

What is the best treatment of histrionic Psychotherapy and antidepressants forpersonality disorder? underlying mood disorders

CLUSTER C: THE SAD

How is avoidant personality disorder Individuals with avoidant personalityoften described? disorder are often shy and timid.

They are very self-critical, have lowself-esteem, and are preoccupiedwith fears of rejection orembarrassment.

What other personality disorder may Schizoid personality disorderavoidant personality disorder bemistaken for?

130 Deja Review: Behavioral Science

Page 146: Deja review   behavioral science

How can avoidant personality disorder Patients with avoidant personalityand schizoid personality disorder be disorder want to have interpersonaldifferentiated? relationships but are afraid of rejection;

whereas, schizoid personality disorderpatients do not wish to haverelationships with others.

What are the symptoms/diagnostic Four or more of the following:criteria of avoidant personality Avoidance of occupations that involvedisorder? interaction with others.

Fear of intimacy/lack of intimaterelationships for fear of ridicule.Unwilling to be involved with peopleunless certain of being liked.Preoccupation with criticism orrejection.Believes they are socially inept andinferior to others.Feelings of inadequacy inhibit socialinvolvement.Reluctance to become involved in newactivities.

What other psychiatric conditions Social phobia, specific phobia, andare often seen in patients with avoidant agoraphobiapersonality disorder?

What is the best treatment of avoidant Psychotherapy and assertivenesspersonality disorder? training, SSRIs, Beta-Blockers

How is dependent personality disorder Patients are passive and may let othersdescribed? direct their lives and make important

decisions. (Think Bill Murray’scharacter in What about Bob?)

What are the symptoms/diagnostic Five or more of the following:criteria of dependent personality Inability to make decisions withoutdisorder? advice

Refusal to assume responsibilityHas difficulty expressing disagreementDifficulty initiating projectsNeed for excessive nurturing andsupportFeelings of discomfort and helplessnesswhen alonePersistent need to be in a relationshipUnrealistic fears of being left alone

Personality Disorders 131

Page 147: Deja review   behavioral science

What other psychiatric disorders are Depression and anxiety disorderscommon in individuals with dependentpersonality disorder?

What is the best treatment for dependent Psychotherapy and assertivenesspersonality disorder? training, treatment of comorbid

conditions

How is obsessive-compulsive Individuals have extreme perfectionistpersonality disorder described? tendencies and inflexibility, a

preoccupation with orderlinessand control.

How does obsessive-compulsive Individuals with obsessive-compulsivepersonality disorder (OCPD) differ personality disorder do not havefrom obsessive-compulsive disorder intrusive thoughts (obsessions) or(OCD)? actions that they carry out to relieve

the anxiety provoked by those thoughts(compulsions).They also do not believe they have aproblem; patients with OCD know theiractions are irrational, whereas OCPDpatients are more likely to thinkeveryone else has the problem.

What are the symptoms/diagnostic Four of more of the following:criteria of obsessive-compulsive Preoccupation with rules/details/personality disorder? organizations—often such that the point

of the activity is lostPerfectionismExcessive devotion to work andproductivityInflexible about moral or ethicalvaluesWill not discard unneeded objects(Pack rats)Reluctant to delegate to othersCheap/frugal in order to hoardmoneyRigidity and stubbornness

What is the best treatment for Psychotherapyobsessive-compulsive personalitydisorder?

132 Deja Review: Behavioral Science

Page 148: Deja review   behavioral science

CLINICAL VIGNETTES

Your patient is an excellent medical student. He always studies for 6 hours a night,every night, and at the same time. He checks his orders three times before he turnsthem in. He does everything himself because he is convinced no one can do asgood of a job as him. He keeps every note he’s ever written about a patient. No oneelse in the class likes him, but he is convinced that’s just because they are jealousof him—he is just fine. He does not do well and is up for dismissal from medicalschool. What is his most likely diagnosis?

Obsessive-compulsive personality disorder

A young man comes into your office complaining of lack of social relationships.He works alone as a night security guard. He got invited to a work party once, butwas too afraid to go. He would like to make friends, but feels he isn’t worthy ofsocializing. He is sure they would only make fun of him anyway. What is the mostlikely diagnosis?

Avoidant personality disorder

Personality Disorders 133

Page 149: Deja review   behavioral science

This page intentionally left blank

Page 150: Deja review   behavioral science

C H A P T E R 1 8

Dissociative Disorders

135

What are the primary characteristics Sudden memory loss of time periods,of dissociative disorders? events, and people

Detachment from one’s selfDerealizationBlurred sense of identity

What are the four major dissociative 1. Dissociative amnesiadisorders? 2. Dissociative fugue

3. Dissociative identity disorder4. Depersonalization disorder

Which conditions are included in the Substance abusedifferential diagnosis of dissociative Seizure disordersdisorders?

Head injuryPosttraumatic stress disorderMalingering

Which disorder is associated with Dissociative amnesiaan inability to remember importantpersonal information?

Which group of people is most Young adult femaleslikely to suffer from dissociativeamnesia?

What is the primary trigger for Often follows a psychologicallydissociative amnesia? traumatic event

What treatment modality is used Psychotherapyfor dissociative amnesia?

What disorder is associated with an Dissociative fugueinability to remember importantpersonal information and wanderingaway from home to adopt a newidentity?

Page 151: Deja review   behavioral science

How long does it normally take for Minutes or days; may last for yearsthe amnesia to resolve in a personwho is experiencing dissociativeamnesia or dissociative fugue?

What treatment modality is used Supportive psychotherapyfor dissociative fugue? Hypnosis

Which controversial disorder is Dissociative identity disorderassociated with a person having“multiple personalities”?

What are the DSM-IV-TR criteria Presence of two or more distinct for Dissociative Identity Disorder? identities or personality states

At least two of these identities takecontrol of the persons behaviorInability to recall personal informationNot due to substance use or a generalmedical condition

Which gender is most likely Femaleto develop dissociative identitydisorder?

What conditions may dissociative Borderline personality disorderidentity disorder resemble? Schizophrenia

What treatment modality is used Psychotherapy and hypnotherapyfor dissociative identity disorder?

Which disorder is associated with Depersonalization disorderrepeated episodes of detachmentand unreality about one’s own body,social situation, or the environment(derealization)?

What treatment modality is used Psychotherapy; pharmacologicfor depersonalization disorder? interventions utilized for associated

anxiety or depression

CLINICAL VIGNETTES

A war veteran claims he cannot remember the beach assault during which his bestfriend was killed. He denies any substance use. What dissociative disorder maythis be?

Dissociative amnesia (Note: This could also be “Post-Traumatic Stress Disorder.”)

136 Deja Review: Behavioral Science

Page 152: Deja review   behavioral science

A 42-year-old stock broker fails to return home from work one day. Two years laterhe is found living in a nearby small town and working as a mechanic at a localshop. He does not recall any of the past events of his life and denies having everbeen a stock broker. What state is this called?

Dissociative fugue (Think Jason Bourne in the movie/book The Bourne Identity.)

A woman often feels as though she is floating outside of her body, watching heractions from above. It has started to interfere with her ability to stay employed.

What is the most likely diagnosis?

Depersonalization disorder.

What about if this feeling had only happened one time during a time of extremedanger?

If it had only happened once, this diagnosis cannot be made—dissociative symptomsare common in life-threatening situations.

A 52-year-old man convicted for murder claims that he does not remember hiscrimes. He has been witnessed to apparently be “possessed” by otherpersonalities, during which he does not remember important information abouthimself. What is a possible psychiatric diagnosis and what is on the differential?

Possibly (though very unlikely) dissociative identity disorder. More likely this couldbe due to substance use or malingering.

Dissociative Disorders 137

Page 153: Deja review   behavioral science

This page intentionally left blank

Page 154: Deja review   behavioral science

C H A P T E R 1 9

Substance AbuseDisorders

139

What are the DSM-IV-TR diagnostic Three or more of the following:criteria for substance dependence? Tolerance.

Withdrawal.More substance taken than wasintended.Failure to control use, or a desire tostop using.Much time is spent either obtainingthe substance, using the substance,or recovering from its effects.Social or occupational activities arereduced because of use.Use continues despite physical orpsychological impacts of use.

How is tolerance defined? Either a need for increased amounts ofsubstance to achieve the same effect ordiminished effect with use of the sameamount

How is withdrawal defined? A withdrawal syndrome characteristicof the drug or the same or similarsubstance is taken to avoid thatwithdrawal syndrome.

What are the DSM-IV-TR diagnostic Does not meet substance dependencecriteria for substance abuse? criteria and one of following:

Substance use results in failure atwork, school, or home.Recurrent use in hazardous situations(eg, driving).Recurrent substance-related legalproblems.Use despite social or interpersonalproblems made worse by use.

Page 155: Deja review   behavioral science

What pathway is involved in the The “dopamine reward pathway” ischemical rewards of drug use? thought to play a large role—it projects

from the ventral tegmental area (VTA) tothe nucleus accumbens.

What factors affect how quickly and • Route of administration: The fasterto what magnitude the chemical through the blood-brain barrier,rewards are felt after ingestion the greater the euphoria and higherof a drug? likelihood of addiction (eg, IV >

smoking > oral [pills])• Chemical composition of the drug

(increasing purity → increased andfaster effects)

• Genetic differences between people(relates to receptor stimulation)

• Associated stimuli (ie, drugparaphernalia, other conditionalstimuli)

What are the three important 1. There is a common associationobservations about withdrawal between tolerance and a specificthat Himmelsbach made? withdrawal syndrome.

2. The nature of the withdrawalsyndrome is opposite to theacute effects of the drug.

3. The withdrawal syndrome is mostintense when the drug leaves thebrain rapidly.

What are the four central tenets 1. The positive reinforcements/rewardthat must be addressed in order effects of the drug must be reduced.to successfully treat drug dependence? 2. The negative reinforcements

(withdrawal symptoms) must betreated, either by giving a substitutedrug or by symptomatically treatingthe effects of drug removal.

3. Detoxification—complete removalof the drug of dependence from thepatient’s system.

4. Relapse prevention by reducing thedesire for the drug or by reducingcravings for the drug.

How should withdrawal symptoms Withdrawal symptoms may bebe prevented/treated during prevented/treated by administeringdetoxification? a substitute drug with a similar effect

(eg, methadone for heroin,benzodiazepines for EtOH), or treatingthe withdrawal symptomatically(eg, treating diarrhea andGI symptoms).

140 Deja Review: Behavioral Science

Page 156: Deja review   behavioral science

How can the positive reinforcement Giving specific receptor antagonistseffects of drugs be reduced? to prevent the binding of receptors

by the drug of dependence; therefore,precipitating withdrawal and preventingthe effects of the drug (eg, naloxone ornaltrexone treatment for opiates)Converting reward to punishment (eg,the use of disulfiram [Antabuse] foralcohol dependence)Giving dopamine or opiate antagoniststo cause general inhibition of thereward pathwaysNegative discriminative stimuli (eg, tellingthe patients that their drug of choicewill be ineffective; therefore, they avoid it)

How does disulfiram work? Disulfiram inhibits aldehydedehydrogenase in the liver. Alcohol isthen unable to be fully metabolized,leading to flushing, headache, andnausea from accumulation of thealdehyde intermediate.

What side effect of opiate antagonists Anhedoniamay result in compliance problems?

What are the three major ways 1. Substitute drugs that act as agonistsof reducing withdrawal? for the same receptor, and therefore,

prevent severe withdrawal (ie, usingmethadone to treat heroin or opiatedependence)

2. Substitution by partial agonist for thesame receptor, thereby, preventingsevere withdrawal and counteractingthe effects of the drug if it is taken

3. Substitution of a different route ofadministration to prevent some ofthe adverse effects of the drug itselfwhile still preventing withdrawal (ie,using the nicotine patch for smokingcessation)

What criteria should be used when Substitute drugs should be:selecting a substitute drug for treating Less rewardingwithdrawal?

Less damagingMore manageableAble to allow the patient to be morefunctional (eg, methadone)

Substance Abuse Disorders 141

Page 157: Deja review   behavioral science

What are the major problems with To use positive reinforcementusing positive and negative treatments, the patient must undergoreinforcement treatments? detoxification first.

Negative reinforcement treatmentsmay lead to polydrug abuse,especially if substitution methodsare used.

What is the basic principle behind Detoxification uses the principle ofdetoxification? substitution with a drug of cross-

dependence or different route ofadministration to allow for safer,slower withdrawal from the drugof dependence. It should not beused to precipitate withdrawal.

What percentage of patients who 75%—most within the first yearbecome drug-free relapse?

What modalities have been associated Joining self-help groups → providingwith decreased incidence of relapse? a different type of peer pressure and

reinforcementCognitive therapy → helps developnew and different coping skills

What is the most likely reason Cravings is the most common reasonfor relapse? for relapse. Cravings are caused by

memories of the positive rewards ofdrug use or by conditioning cues thatare endogenous or exogenous.

How can cravings be treated to prevent Cravings may be treated by reducingrelapse? the desire for the drug, providing a

substitute for the drug reward, reducingendogenous cues for cravings (ie, byusing anxiolytics or antidepressants),reducing the conditional anticipationof the reward (ie, giving naltrexoneto an alcoholic), or by reducingpseudowithdrawal symptoms.

What are some of the medical effects Increased risk of lung diseaseof drug abuse? and cancer

Increased risk of humanimmunodeficiency virus (HIV),hepatitis, and other infections withintravenous (IV) drug useAcute and chronic toxicities

142 Deja Review: Behavioral Science

Page 158: Deja review   behavioral science

What are some of the nonmedical Sociological problems such as violence,consequences of drug abuse? crime, and poverty

Acute impairment leading to reducedcognition or restraintOther risky behaviors (eg, increasedlikelihood of sexual violence)

What are the acute symptoms of ethanol Vomiting (with risk of possibletoxicity? aspiration due to decreased mental

status)Respiratory depressionComaDeath

What are the chronic symptoms Psychiatric symptoms (depression,of ethanol toxicity? hallucinations)

Neurological signs (dementia, vascularproblems, and neuropathies)GI tract malfunction (cirrhosis of theliver, pancreatitis, and GI cancer)Cardiovascular disease (cardiomyopathy,hypertension)

What are the clinical manifestations Pre- and postnatal retardation of growthof fetal alcohol syndrome? and cognition

Facial abnormalities (short palpebralfissures, thin vermillion border of theupper lip, smooth philtrum, andflattened midface)Central nervous system (CNS)damageAttention deficitsTendency for risk-taking behaviors, suchas substance abuse

What is the most common form Caffeine dependence, which is seen inof drug dependence? 60% to 70% of the population

What are the deleterious effects In acute intoxication there can be of caffeine? cardiovascular side effects

(palpitation, arrhythmia, increasein BP). Caffeine withdrawal cancause headache.However, there is little to no evidenceof adverse effects with chronic, casualcaffeine use.

Substance Abuse Disorders 143

Page 159: Deja review   behavioral science

What is the mechanism of action Cocaine is a stimulant that works byof cocaine? preventing the reuptake of catecholamine

transmitters, such as dopamine andnorepinephrine, in the brain andautonomic nervous system.

What are the physiologic effects Vasoconstrictionof cocaine use? Tachycardia

HyperthermiaHypertensionCardiac dysrhythmiasStrokePsychosis

What is the most likely mechanism Dopamine potentiationof psychosis in chronic cocaine users?

What is the mechanism of action Amphetamines block both the reuptakeof amphetamines? of dopamine and norephinephrine at

the synapse, as well as cause the releaseof stored catecholamines.

What are the acute and chronic effects Vasoconstrictionof amphetamine use? Tachycardia

HyperthermiaHypertensionCardiac dysrhythmiasStrokePsychosisNeurotoxicity

What is the mechanism of action Dissociative anesthetics work byof dissociative anesthetic drugs like blocking N-methyl-D-aspartate (NMDA)ketamine and phencyclidine? receptors and sigma receptors in the CNS.

What are the effects of dissociative Amnesiaanesthetic use? Confusion

Delusions and hallucinationsViolent behaviorHyperthermia

What is the mechanism of action Marijuana is a tetrahydrocannabinoidof marijuana? that works on cannabinoid receptors, a

member of G protein–linked receptors.These affect monoamine andγ-aminobutyric acid (GABA) neuronsin the basal ganglia, hippocampus, andcerebellum.

144 Deja Review: Behavioral Science

Page 160: Deja review   behavioral science

What are the chronic toxicities Poor memory and motivation associated with marijuana use? Testosterone suppression, gynecomastia

Chronic obstructive pulmonary disease(COPD)ImmunosuppressionLow fetal birth weight

Where are the opioid receptors located Periaqueductal grey matter →in the brain and what is the function responsible for analgesiaof each receptor? Area postrema → responsible for nausea

and vomitingVentral medulla → responsible forrespiratory depressionEdinger-Westphal nucleus →responsible for the pinpoint pupilresponse (due to extreme miosis)Nucleus accumbens → responsible foreuphoria

What are the three classic types 1. Muof opioid receptors? 2. Kappa

3. Delta

What receptors do most clinically used Muopiates such as morphine work on?

What endogenous peptide transmitters Enkephalins act on mu and deltawork at opioid receptor and which receptors, main actions are at delta receptors do they act on? receptors.

Beta-endorphins act on mu and deltareceptors equally.Dynorphins act on kappa receptors.

Describe the mechanism of action Opioid receptors are G protein–coupledof opioids at their receptors: receptors with seven membrane-

spanning segments. Activation of thereceptors causes changes in cyclicadenosine monophosphate (cAMP),Ca2+, etc, leading to inhibition ofneuronal excitation.

How do strong agonists cause Strong agonists have high affinitythe effects of opioids? for the receptor and produce a

conformational change that activatesthe receptor.

How do antagonists work? Antagonists have high affinity for thereceptor and do not activate the receptor.

Substance Abuse Disorders 145

Page 161: Deja review   behavioral science

What is the difference between strong Both strong and partial agonists haveand partial agonists? high affinity for the receptor; but unlike

strong agonists, partial agonists havelow efficacy to activate the receptorresulting in a weak effect. Because ofthis high affinity/low efficacy, they canantagonize/block the effects of astronger agonist.

What is the difference between opiates Opiates are derived from theand opioids? opium poppy, while opioids

are synthetic or semisyntheticderivatives of opiates.

How do kappa and mu receptors differ? The difference between kappa receptorsand mu receptors is that activation ofkappa receptors produces less analgesia,less respiratory depression/asphyxia(floor effect), and produce dysphoriainstead of euphoria.

Which opioids/opiates are strong Morphinemu receptor agonists? Fentanyl

EtorphineHeroinHydromorphoneOxycodoneMeperidine

Which opioids/opiates are partial Buprenorphinemu receptor agonists? Pentazocine

What relative of morphine without Dextromethorphansedative effect is used to suppresscough?

Which opioids/opiates are mu receptor Nalorphineantagonists? Naltrexone (nonnarcotic)

Which opioids/opiates are kappa Buprenorphinereceptor antagonists? Naltrexone (nonnarcotic)

What drug can be used as a narcotic Naltrexone/Naloxone (aka Narcan)reversal agent?

Which opioids/opiates are metabolized Oxycodoneby CYP2D6? Codeine

146 Deja Review: Behavioral Science

Page 162: Deja review   behavioral science

What is the significance of being In drugs that are metabolized bymetabolized by CYP2D6? CYP2D6, the active metabolite is

morphine.

Which opioid receptor agonist does not Loperamide, which works by reducingwork systemically and can be used as GI tract motilityan antidiarrheal agent?

What is the clinical indication for Methadone is used to treat withdrawalmethadone? from heroin while acting to decrease

cravings and risk from associatedlifestyle (legal, HIV, hepatitis, etc).

What are the symptoms of narcotic In narcotic overdoses, CO2 drive is overdose? reduced resulting in respiratory

depression and cyanosis anddiminished mental status/coma.

What is the route of absorption The very low molecular weight asof ethanol in the body? well as its water and lipid solubility

allow for rapid absorption of ethanolfrom the GI tract and entry into thebrain.

What is the limiting factor in ethanol Absorption of ethanol is limited only absorption? by the surface area of the stomach.

What is the effect of ethanol’s water Its water solubility allows absorbedsolubility? ethanol to be distributed throughout

the body water.

What is the mechanism of action Ethanol’s mechanism is not completelyof ethanol? known, but likely has to do with

alteration of membrane fluidity aswell as actions on GABA receptors,glutamate NMDA receptors, andthe serotonin system.

What is the physiologic effect of GABA is the major inhibitoryethanol’s action on GABA receptors? transmitter in the brain, and ethanol

potentiation of GABA causes anxiolysisand possibly the reward effects byincreasing dopamine release fromthe nucleus accumbens.

What is the physiologic effect of Glutamate is a major excitatoryethanol’s action on glutamate receptors? transmitter in the brain, and ethanol

inhibits glutamate causing amnesia andanesthetic effects and possibly rewardeffects.

Substance Abuse Disorders 147

Page 163: Deja review   behavioral science

What is a useful and evidence-based CAGE questions:way to screen for alcohol abuse? Ever felt the need to Cut down your

drinking?Have you felt Annoyed by criticismof your drinking?Ever felt Guilty about your drinking?Ever taken an Eye opener (a drink firstthing in the morning)?

How many drinks are needed for Four standard drinks = a BAC of 0.08% an 180-lb man to reach the legallimit (0.08% blood alcohol content[BAC])?

How many drinks are needed for an Three standard drinks = a BAC of 0.10%140-lb woman to reach the legal limit (over the legal limit)(0.08% BAC)?

How is ethanol eliminated from Ninety-five percent of the ethanolthe body? ingested is metabolized in the liver.

How is ethanol metabolized in Alcohol metabolism is a zero-orderthe liver? process in which alcohol is converted

by alcohol dehydrogenase to aldehydes,and then the aldehydes are convertedby aldehyde dehydrogenaseto acetate.

What is zero-order elimination Zero-order elimination occurs when aof a drug? constant amount of drug is eliminated

from the body per unit of time,regardless of the drug dose orserum concentration.

148 Deja Review: Behavioral Science

Time (h)

Con

cent

ratio

n of

pla

sma

Figure 19.1 Zero-order elimination. (Drug has a constant elimination per unit time, regardless ofconcentration.)

Page 164: Deja review   behavioral science

What is the limiting factor in ethanol The limiting factor in alcoholmetabolism by the liver? metabolism is the availability of

nicotinamide adenine dinucleotide(NAD) and nicotinamide adeninedinucleotide plus hydrogen (NADH),which is used by aldehydedehydrogenase.

Which metabolic process is responsible The toxic effects of alcohol are related tofor the toxic effects of alcohol? metabolism of alcohol to aldehydes, the

buildup of which causes liver and tissuedamage, flushing, nausea, andheadache.

What are the chronic effects Development of toleranceof alcohol ingestion? Psychological dependence

Physiologic dependence (this includestolerance and withdrawal)Huge host of medical problems,including liver disease, cancer, etc

What are the three types of tolerance? 1. Metabolic tolerance: allows formore rapid metabolism andexcretion

2. Behavioral tolerance: involveslearning to perform a task whileintoxicated as if the person is notintoxicated

3. Neuroadaptation: involves theevolution of alterations in the brainthat help to overcome the effects ofthe drug

Substance Abuse Disorders 149

Time (h)

Ser

um d

rug

conc

entr

atio

n

Figure 19.2 First-Order Elimination. (Drug elimination is concentration dependent—more iseliminated as serum concentration is raised.)

Page 165: Deja review   behavioral science

What are symptoms of minor Minor signs occur early and subsidewithdrawal from chronic alcohol use? within 2 days. They include anxiety,

tremulousness, insomnia, GI upset,palpitations, diaphoresis, and headache.

What are withdrawal seizures? Tonic-clonic movements occurring from2-48 hours after the last drink. This mayprogress to delirium tremens (DT).

What are alcoholic hallucinations? Visual, tactile, or auditory hallucinationsfrom 12-48 hours after the last drink.Vital signs remain normal.

What is delirium tremens (DTs)? An alcohol withdrawal syndrome withhallucinations, altered mental status,and seizures accompanied by vitalsign abnormalities (tachycardia,hypertension, fever). Vital signabnormalities may be the first clueto diagnosis. DT may lead to death.

What is the earliest that DTs occur? 48 hours after the last drink, thoughmay be longer

How is alcohol withdrawal treated? Benzodiazepines for suppression ofwithdrawal symptomsVitamins (particularly thiamine) to treatunderlying vitamin deficiencies seen inlong-term alcoholismAntihypertensives to suppressunderlying hypertension that canbe made worse by the physiologicwithdrawal process

What is the relapse rate in the first year 75%after alcohol detoxification?

What are the most common reasons Memories/conditioningfor relapse in alcoholics? Peer pressure

GeneticsRapid reinstatement of physiologicdependence

What types of treatment can be used Support groups (eg, AA)to prevent relapse in alcoholics? Psychological treatment

Pharmacotherapy (naltrexone,disulfiram, or acamprosate)

150 Deja Review: Behavioral Science

Page 166: Deja review   behavioral science

CLINICAL VIGNETTES

A 48-year-old man comes in for an elective surgery. His operation is a great successand there are no major complications. Three days after his surgery he becomesvery disoriented, and yells nonsense at the nurses. His HR is 110, his BP is 154/92,and he is febrile. His family asks you if there is anything they can do. What is akey piece of history you could ask the family about?

Ask about his drinking habits. He may be in alcohol withdrawal since he haslikely not had a drink since surgery.

A 26-year-old woman just had her third DUI arrest. She gets drunk every nightwith her friends and has never had any regrets about doing it. She has lost fourjobs because she is unable to show up on time in the morning. She claims shedoesn’t have a problem because it only takes her 5 beers to get drunk—the sameamount as when she started drinking 5 years ago. She now spends most of her daydoing odd jobs so that she can afford to go out drinking. Her doctor told her thather liver is suffering, but she continues to drink. What is her likely diagnosis?

She meets criteria for substance dependence, despite her lack of tolerance. She spendsmost of her time involved in substance-seeking behavior, has had occupationalimpairment, and has had a physical impact from use as well. She cannot bediagnosed with substance abuse as dependence takes precedence.

A 19-year-old college student is brought to the ER by his fraternity brothers at 3 AM

after a Friday night party. The young man is arousable to painful stimuli only. Hispupils are only 1 mm, but reactive. You notice his respiratory rate is quite low. Hisfriends say there has been a lot of drinking at the party as well as some pills.

What drugs has the patient most likely ingested?

Alcohol and opiates (note pinpoint pupils)

What drug should be administered immediately after this patient arrives in the ER?

Naloxone

Substance Abuse Disorders 151

Page 167: Deja review   behavioral science

This page intentionally left blank

Page 168: Deja review   behavioral science

C H A P T E R 2 0

Eating Disorders

153

What are the two major eating disorders 1. Anorexia Nervosaaccording to the DMV-IV-TR? 2. Bulimia Nervosa

What are the two subtypes of Anorexia 1. Restricting typeNervosa? 2. Binge-Eating/Purging type

What distinguishes the subtypes? The presence of either regular bingeeating or purging behavior is Binge-eating/purging type. The absence ofeither is restricting type.

What is the biggest distinguishing In anorexia there is a refusal to maintaincharacteristic between Anorexia a body weight above 85% of ideal.and Bulimia? Bulimic patients may be of normal

weight.

What are the DSM-IV-TR diagnostic All of the following:criteria for anorexia nervosa? Severe weight loss (weighs <85% of

ideal body weight)Intense fear of gaining weightAmenorrheaDisturbed perception of body weight, ordenial of seriousness of weight loss, orundue influence of weight on self-evaluation

What are the associated physical Lanugoor biological findings in anorexia Melanosis coli (in those that use laxatives)nervosa?

AnemiaLeukopeniaElectrolyte imbalance

How is amenorrhea defined? Absence of a menstrual period for atleast three cycles

Page 169: Deja review   behavioral science

What is lanugo? Fine, downy body hair, especially seenon the trunk

What is melanosis coli? Blackened areas on the colon, seen withlaxative abuse

What is a long-term risk of anorexia? Osteopenia, Mitral Valve Prolapse,Amenorrhea, Electrolyte Disturbance,and Death

What are some of the warning signs Excessive dieting, exercise, use ofof anorexia? laxatives/diuretics/enemas

Abnormal eating habitsBody image disorder/body dysmorphicdisorderFear of becoming fatDecreased libido

What is the typical profile of a patient Anorexia is most commonly seen inwith anorexia? adolescent to young adult females

who are very high achieving (eitheracademically, athletically, or both).There is often a lot of conflict withinthe family, sometimes described as acontrolling or overly protective mother.

Do patients with anorexia typically Anxiety disorders are commonly seen,have coexisting disorders? as are mood disorders and substance

abuse.

What is the best treatment for anorexia Nutritional therapy and psychotherapynervosa? including cognitive-behavioral therapy

(CBT) can be helpful. Pharmacotherapy isnot terribly helpful.

What are the DSM-IV-TR diagnostic All of the following:criteria for bulimia nervosa? Recurrent binge eating, with both a

large amount of food and a lack ofcontrol during the episodeCompensatory behavior to preventweight gain (vomiting, laxatives,exercise, etc)Binge and purge behaviors occur atleast twice a week for 3 monthsSelf-evaluation influenced by bodyshape and weightDoes not qualify for anorexia nervosadiagnosis

154 Deja Review: Behavioral Science

Page 170: Deja review   behavioral science

What are common associated findings Erosion of tooth enamelin persons with bulimia nervosa? Parotitis (inflammation of parotid

gland)Calluses on the dorsal surface of thehandsElectrolyte imbalanceOften normal body weight (may beslightly overweight)

What is the cause of erosion of the Repeated exposure to gastric acidtooth enamel? secondary to induced vomiting

What is parotiditis? Swelling or infection of the parotidglands, usually secondary tovomiting

What causes the development of Scraping fingers along teeth whilecalluses on the back of the hands? inducing vomiting

What common medical consequences Hypokalemiaare seen with repeated vomiting? Metabolic Alkalosis

Endocrine disturbancesGrowth disturbanceBradycardia and dysrhythmias

What is a traumatic consequence Esophageal varices and/or Mallory-Weissof the repeated induced vomiting tears from repeated retchingseen in bulimia nervosa?

How can purging be accomplished? Induced vomitingLaxative/diuretic/enema useExcessive exercise

Do Bulimics have to vomit or use No. Even excessive exercise can laxatives to meet criteria? be a “purging” behavior.

What are some of the psychosocial Poor self-imagefeatures of patients with bulimia? Depression and other mood disorders

What is the best treatment for bulimia Psychotherapynervosa? Behavior therapy

Antidepressants—selective serotoninreuptake inhibitors (SSRIs) preferredNutrition educationRegular mealsHealthy exercise

Eating Disorders 155

Page 171: Deja review   behavioral science

What is Eating Disorder Not Otherwise A grab-bag of disordered eating thatSpecified (NOS)? does not meet criteria for Anorexia or

Bulimia. This includes:• Purging behavior without true binge

eating• Anorexia criteria without amenorrhea• Restricting type anorexia without

significant weight loss (ie, no bingeeating/purging to meet bulimiacriteria)

• Bulimia criteria except at a lowerfrequency than required fordiagnosis

• Chewing and spitting out largeamounts of food

• Binge eating without purgingbehavior

CLINICAL VIGNETTES

A 17-year-old woman comes to your office complaining of fatigue. Upon carefulexamination you see erosion of the enamel on her molars. You check an electrolytepanel and find her to be hypokalemic.

What are the possible psychiatric disorders that may be present?

This patient may have anorexia nervosa, bulimia nervosa, or eating disorder NOS.

How do you distinguish anorexia nervosa and bulimia nervosa?

To distinguish these disorders you would need to know her body weight vs herideal body weight.

What is the likely cause of her hypokalemia? If you took an arterial blood gas,what would you expect the pH to be?

Her hypokalemia is likely due to recurrent vomiting—this will also cause ametabolic alkalosis (think loss of stomach H+), so you would expect her pH to behigh (alkalemic).

156 Deja Review: Behavioral Science

Page 172: Deja review   behavioral science

A 12-year-old male comes into your office asking for help in losing weight to makethe wrestling team. He states that he has been exercising significantly to loseweight, but with no effect. He is terribly afraid of gaining weight, believes he ishorribly fat, and hates himself for weighing so much. His ideal body weight is 100 lband he currently weighs 90 lb. He states he’d like to weigh 80 lb and requests youto give him diuretics to help him do so, which you refuse to do.

What is his current likely diagnosis?

His current diagnosis is Eating Disorder NOS—he would meet criteria for anorexianervosa, but he does not weigh <85% of ideal body weight.

If the patient began to use diuretics against your advice and lost 10 lb, would itchange?

If he began to use diuretics and went down to 80% of his ideal body weight, hewould qualify for a diagnosis of Anorexia Nervosa, Binge-eating/purging type(note that he does not need to binge eat to qualify; purging alone is enough).

What if he started binge eating as well as using diuretics and still weighed 90 lb?

If he is both binging and purging but does not weigh under 85% of his ideal bodyweight, he would qualify for a diagnosis of Bulimia Nervosa.

Eating Disorders 157

Page 173: Deja review   behavioral science

This page intentionally left blank

Page 174: Deja review   behavioral science

C H A P T E R 2 1

Child Psychiatry

159

PERVASIVE DEVELOPMENT DISORDERS

What are the primary characteristics Failure to acquire or early loss ofof the pervasive development communication and social interactiondisorders? skills

What are the pervasive development Autistic disorderdisorders? Asperger disorder

Rett disorderChildhood disintegrative disorder

Which disorder is characterized by Autistic disordersignificant communication problems,difficulty in forming socialrelationships, repetitive behavior,and unusual abilities?

What term describes the unusual Savantabilities (eg, memory, calculation skills)that some autistic patients have?

The onset of autistic disorder must be 3 years of agebefore what age?

What is the risk of autism in Increased risk due to genetic componentmonozygotic twins and siblings?

The incidence of autistic disorder is Congenital anomaliesincreased in which conditions? Perinatal complications

Congenital rubellaPhenylketonuriaFragile X syndromeTuberous sclerosis

What has been shown in multiple Vaccinationanalysis not to be associated with autism?

Page 175: Deja review   behavioral science

Which brain abnormalities are Seizuresassociated with autism? Electroencephalogram (EEG)

abnormalitiesAnatomic and functionalabnormalities

Which gender is most likely to be Males; Four times more likelyaffected by autism?

Which gender is more severely Femalesaffected with mental retardationin autism?

How is autistic disorder treated? Therapy aimed at increasingcommunication, social, and self-careskills

What is the IQ of many autistic patients? Generally low; may have normalnonverbal IQ

Which disorder is a milder form Asperger disorderof autism?

What are the primary deficits in a Problems forming social relationshipspatient with Asperger? Repetitive behavior

Acute interest in obscure topics

Which areas of functioning are normal Cognitive and verbal skillsin Asperger but are usually deficientin autistic patients?

Which disorder is characterized by a Rett disorderdecrease in social, verbal, and cognitivedevelopment after a period of normalfunctioning?

What are the primary characteristics Stereotyped hand-wringing movementsof Rett disorder? Poor coordination

Impaired language developmentLoss of hand skillsLoss of social engagementDeceleration of head growth

Which gender is primarily affected Femalesby Rett disorder?

What is the genetic inheritance of X-linkedRett disorder?

160 Deja Review: Behavioral Science

Page 176: Deja review   behavioral science

What happens to males affected They die before birth.by Rett disorder?

Which rare disorder is characterized Childhood disintegrative disorderby a diminution of cognitive, motor,social, and verbal development after2 to 10 years of normal functioning?

Which gender has the highest Boysincidence of childhooddisintegrative disorder?

DISRUPTIVE BEHAVIOR DISORDERS

What are the primary characteristics Improper behavior; problems withof disruptive behavior disorders? school performance and social

relationships

Which disorders are classified Conduct disorderas disruptive behavior disorders? Oppositional defiant disorder

Which disorder is characterized by Conduct disorderinsistent behavior that violatessocial norms, deviation from societaland parental rules, property destruction,and aggressive behavior?

What are examples of the behaviors Arsonthat violate social norms that are Theftcommon in patients with conduct

Animal harmdisorder?Assault

If a person is 18 years or older and Antisocial personality disorderstill exhibits the symptoms ofconduct disorder, which disordermight they have?

Which disorder is characterized by Oppositional defiant disorderpersistent disobedient, defiant,and negative behavior towardfigures in authority?

What are the primary treatment Psychotherapy and a Structuredmodalities for the disruptive environment, sometimes behavior disorders? pharmacotherapy

Child Psychiatry 161

Page 177: Deja review   behavioral science

ATTENTION-DEFICIT HYPERACTIVITY DISORDER

What are the primary characteristics Inattentionof attention-deficit hyperactivity Hyperactivitydisorder (ADHD)?

ImpulsivityImpairment in multiple settings(eg, both school and home)

Which gender is most likely to be Boysaffected by ADHD?

By definition, ADHD symptoms Before 7 years of agemust be evident by which age to begiven the classification of ADHD?

How long must the symptoms of ADHD At least 6 monthsbe present to be given the classificationof ADHD?

What percentage of the general child 3% to 7%population in the United States isaffected by ADHD?

What is the intelligence level of persons Normal intelligencewith ADHD?

What percentage of ADHD patients 20%have symptoms that persist intoadulthood?

What are the primary treatment Stimulants—usually amphetaminesmodalities of ADHD?

What is a non-stimulant medication Atomoxetinefor individuals with ADHD greaterthan the age of 6?

Which single drug is most widely Methylphenidateprescribed for ADHD?

OTHER NEUROPSYCHIATRIC DISORDERS OF CHILDHOOD

Which disorder is characterized by Tourette syndromechronic motor and vocal tics andinvoluntary use of profanity?

162 Deja Review: Behavioral Science

Page 178: Deja review   behavioral science

In which age group is Tourette Usually 7 to 8 years of age; onsetsyndrome most likely to be usually by 21 years of agediagnosed?

Tourette syndrome has a high ADHDcomorbidity with which other Obsessive-compulsive disorderpsychiatric disorders?

What is the primary treatment Dopamine-blocking antipsychoticsfor Tourette syndrome? (if interfering with social interactions)

What is a treatment for Tourette Botox (for tics) or Deep brain stimulationsyndrome that is refractoryto medications?

Which disorder is characterized by Separation anxiety disorderexcessive and inappropriate anxietyconcerning separation from parents,caretakers, and their home andproduction of physical complaintsto avoid going to school?

What is the most common age of 7 to 8 years of ageonset in a person who presentswith separation anxiety disorder?

Which treatment modalities are most Psychotherapy, especially cognitive-effective in the treatment of separation behavioral therapyanxiety disorder?

Which disorder is characterized by Selective mutisma refusal to verbally communicatein some or all social situations inwhich the child may communicatewith gestures?

In which gender is selective mutism Girlsmore common?

Selective mutism has a poor 10 years of ageprognosis if it persists afterwhich age?

What is the primary trigger for the onset Stressful life eventsof separation anxiety disorder andselective mutism?

What is the most common treatment Family and behavioral therapymodality for selective mutism?

Child Psychiatry 163

Page 179: Deja review   behavioral science

CLINICAL VIGNETTES

A distressed mother comes to your clinic with her 4-year-old daughter. She claimsher daughter was completely normal up until last year when she had a regressionof her social and cognitive skills—right after her last batch of immunizations. Themother is now considering suing the vaccine manufacturer for causing her child tohave autism. What can you tell the mother is the most likely diagnosis of the child?

Childhood disintegrative disorder

A father brings his 9-year-old son in to evaluate him for ADHD. His teacher toldthe father that his son is impulsive, inattentive, and can’t stay in his seat. Thefather says that neither he nor his mother has noticed these symptoms anywhereelse, but he believes the teacher. He asks if his son should be started onmedication. What should you advise the father?

The child shouldn’t be started on any medications at this time. The child’sbehavior is limited only to the school setting and does not meet criteria for ADHD.

A 14-year-old male is brought to a psychiatrist because of his disregard for rulesset by authority figures at home and in school. He has set numerous fires in hisneighborhood. His parents are concerned that he seems to have no regard for thefeelings of others. Which disorder is this patient most likely to be diagnosed with?

Conduct disorder

A 3-year-old girl presents with problems forming social relationships. She engagesin repetitive behavior, and she has a strong interest in learning all about thedifferent types of bubble gum in the world. Her mother states that the girl has nothad any cognitive deficits and has had no developmental language delay. Whichcondition is the patient most likely to be diagnosed with?

Asperger disorder

164 Deja Review: Behavioral Science

Page 180: Deja review   behavioral science

C H A P T E R 2 2

Psychopharmacology

165

What is the only property of Tolerance does not develop to thebenzodiazepines to which antianxiety/anxiolytic effects of benzos;tolerance does not develop? tolerance may develop to the hypnotic,

muscle relaxant, and anticonvulsanteffects (eg, benzodiazepines should notbe used for long-term seizure control).

Which benzodiazepines are considered Alprazolamanxiolytics? Chlordiazepoxide

ClonazepamClorazepateDiazepamLorazepam

Which benzodiazepines are considered Quazepamhypnotics (used to facilitate sleep)? Midazolam

EstazolamFlurazepamTemazepamTriazolam

What drug can be used to reverse Flumazenil (Romazicon)—though it isthe effects of benzodiazepines? used with caution as it can precipitate

withdrawal seizures

What is the general mechanism Benzodiazepines target the GABAAof action of benzodiazepines? (γ-aminobutyric acid A) chloride channel

receptor, resulting in an increase in thereceptor’s affinity for and causing theion channels to open more frequently, thusallowing more chloride ions to passthrough.

Is there a danger in taking Benzodiazepines can cross the placenta,benzodiazepines during pregnancy? and therefore should not be taken

during pregnancy if possible.They are categories D and X.

Page 181: Deja review   behavioral science

How many phases are there in There are three phases.benzodiazepine metabolism? 1. Phase 1: The R1 and R2 residues are

oxidized.2. Phase 2: The R3 residue is

hydroxylated.3. Phase 3: The hydroxyl compounds

are conjugated with glucuronic acid.

Where does benzodiazepine In the livermetabolism occur?

Which benzodiazepines skip phase 1 Desmethyldiazepamand/or phase 2 of metabolism and are Oxazepamtherefore safer to give to patients

Temazepamwith liver failure?LorazepamMidazolamTriazolam

What are the symptoms of Anxietybenzodiazepine withdrawal? Insomnia

IrritabilityDelirium, PsychosisWeaknessTremorSeizures

What are the possible side effects Drowsinessof benzodiazepines? Confusion

Motor incoordinationCognitive impairmentAnterograde amnesia

Which benzodiazepines are considered Triazolamto be high potency? Alprazolam

Clonazepam

Which benzodiazepines are considered Diazepamto be low potency? Chlordiazepoxide

Oxazepam

Which benzodiazepines are most Chlordiazepoxidecommonly used for treatment Lorazepamof alcohol withdrawal seizures

Diazepam(delirium tremens)?

166 Deja Review: Behavioral Science

Page 182: Deja review   behavioral science

Which benzodiazepines are indicated Diazepam and lorazepamfor treatment of status epilepticus,as they can be given intravenously (IV)?

Why are benzodiazepines safer Less potential for abusepharmacologic agents to use to Higher therapeutic indextreat anxiety than barbiturates?

What is the mechanism of action Barbiturates target the GABAA chlorideof barbiturates? channel receptor and its action on

chloride entry into the cell, whichresults in membrane hyperpolarization.There is an increase in the duration ofthe chloride channel opening and adecrease in neuron excitability.

What is the mechanism of action and It is a 5-hydroxytryptamine receptor 1Acommon use of Buspirone? (5-HT1A) (serotonin) agonist that may be

used to treat anxiety, particularly usefulin those for whom benzodiazepinetherapy is contraindicated (the elderlyand those with a history of substanceabuse).

Which short-acting antianxiety agent Zolpidem tartrateis used to treat insomnia?

What is the most commonly prescribed SSRIs (selective serotonin reuptakeclass of drugs for the treatment inhibitors)of depression?

What is the mechanism of action They work by inhibiting neuronalof SSRIs? uptake of serotonin, thereby increasing

the synaptic concentration of serotonin.

What are the names of commonly Fluoxetineprescribed SSRIs? Fluvoxamine

ParoxetineSertralineCitaprolam

What serious side effects have a greater Anticholinergic effects, sexualincidence associated with the SSRI dysfunction, and withdrawal syndromeParoxetine?

What caution should you take while There is a black box warning aboutusing SSRIs in children and adolescents? increased suicidal thoughts in patients

<25 years—warn your patients andmonitor carefully.

Psychopharmacology 167

Page 183: Deja review   behavioral science

Which SSRIs are now available Fluoxetine (Prozac)in generic form (and may be a better Paroxetinechoice for patients with a limited

Sertralinebudget)?Citalopram

Why does fluoxetine and paroxetine They have greater P-450 inhibition thanhave a greater association with drug the other SSRIs.interactions than other SSRIs?

Why is insomnia a troubling side effect It causes the most central nervousof fluoxetine? system (CNS) activation of the

SSRIs.

Which SSRI is most likely to cause Sertraline (Zoloft)gastrointestinal disturbances?

Which SSRI is currently only Fluvoxamineindicated for obsessive-compulsivedisorder?

What is the mechanism of action It is thought to work mostly on dopamineof bupropion (Wellbutrin)? and norepinephrine, though the exact

mechanism is unknown.

What is the major indication Major depressive disorderfor bupropion?

What is another common indication Smoking cessationfor bupropion?

What is a unique side effect It can lower the seizure threshold,of bupropion that should be thought especially in quick release formulasof before prescribing it in epileptic and at high doses.patients?

What is a common indication for the use Insomniaof trazodone?

What is a problematic side effect of the Priapism (painful, persistent erection)antidepressant trazodone seen mostlyin males?

What is the mechanism of action Tricyclic antidepressants inhibit theof tricyclic antidepressants (TCA)? neuronal reuptake of both serotonin

and norepinephrine, thus, increasingthe availability of serotonin andnorepinephrine at the synapticcleft.

168 Deja Review: Behavioral Science

Page 184: Deja review   behavioral science

What are the names of commonly Amitriptylineprescribed tricyclic antidepressants? Clomipramine

DoxepinDesipramineImipramineNortriptyline

Why are tricyclics used less often Side effects and potential for lethalthan SSRIs? overdose. They are still very efficacious

medicines.

What other syndrome are TCAs Chronic and neuropathic painindicated for treating?

What are common side effects Tricyclic antidepressants haveof tricyclic antidepressants? anticholinergic effects including dry

mouth, blurry vision, constipation,urinary retention, confusion, andmemory deficits.

What are the effects of TCA overdose? Cardiac effects are the mostdangerous—notably widened QRS,prolonged PR and QT intervals.Sedation, delirium, and anticholinergiceffects may also be present.

What is the most important clinical ECGtest in evaluating suspected tricyclicoverdose?

What is the treatment of acute tricyclic IV Sodium Bicarbonateoverdose?

Which tricyclic antidepressants Tertiary tricyclics (eg, amitriptyline)are known to have more anticholinergiceffects?

What are examples of other tertiary Imipraminetricyclics antidepressants? Doxepin

Which tricyclic agent is the least Desipraminesedating?

Which tricyclic is prescribed for Imipramineenuresis (bed-wetting)?

Which tricyclic is classically prescribed Clomipraminefor obsessive-compulsive disorder? Note: This is the most serotonin-specific

tricyclic.

Psychopharmacology 169

Page 185: Deja review   behavioral science

Which tricyclic is the least likely Nortriptylineto cause orthostatic hypertension?

What is the most common indication Treatment of atypical depressionfor MAOIs (monoamine oxidase (depression with increased sleep,inhibitors)? appetite, and leaden paralysis)

What is the mechanism of action They work by inhibiting theof MAOIs? mitochondrial enzyme monoamine

oxidase which metabolizesnorepinephrine, serotonin, anddopamine, resulting in a buildupof these biogenic amines and theirsubsequent leakage into thesynapse.

What are the names of commonly Selegilineprescribed MAOIs? Phenelzine

Tranylcypromine

What are the common side effects Sedationof MAOIs Anticholinergic effects

Orthostatic hypotensionCardiac conduction disturbances

What other medication must be avoided Stimulants and pseudoephedrine mustin patients taking MAOIs? be avoided because they can cause

hypertensive crisis.

What other substance must be avoided Tyramine, which is found in agedby patients taking MAOIs? cheeses, beer, certain meats and fish,

fava beans, red wine (particularlyChianti), avocados, chocolate,and dairy products. Signs oftyramine ingestions includeheadache, arrhythmias, andhypertensive crisis.

What MAO inhibitor is now available Selegiline, which is also useful forin a transdermal patch which does not patients who cannot swallow oralrequire a change in diet? medicines

What are some pharmacologic agents Amphetaminesthat can cause psychosis? Cocaine

L-Dopa

170 Deja Review: Behavioral Science

Page 186: Deja review   behavioral science

What is the mechanism of action for They are dopamine receptormost typical antipsychotics? antagonists.

What is the most popular of the The dopamine hypothesis which citesbiochemical theories that explain an increase in dopamine transmissionthe development of psychosis? as the cause. This is supported by the

clinical potentcy of typicalantipsychotics being directly relatedto their affinity to the D2 receptor.

What is the most common problematic Extrapyramidal effects/Parkinson-likeside effect of typical antipsychotics? side effects:

BradykinesiaRigidityTremorAcute dystoniaAkathisiaNote: Chronic tardive dyskinesias mayalso occur.

Which of the extrapyramidal side Acute dystonic reactioneffects is a flag for caution for futureuse of typical antipsychotics?

Which typical antipsychotics have the Fluphenazine = haloperidolmost extrapyramidal effects? > chlorpromazine > thioridazine

Note: This is correlated withpotency/D2 blockade.

Which is the most potent of the typical Fluphenazineantipsychotics?

What pharmacologic evidence does not Atypical antipsychotics are verysupport the dopamine hypothesis? efficacious, but many work on other

neurotransmitters and have little effecton dopamine.

What is the mechanism of action of the It acts on dopamine, serotonin,atypical antipsychotic clozapine? and acetylcholine receptors.

What is the greatest benefit of atypical They usually have fewer extrapyramidalantipsychotics? side effects than typical antipsychotics.

What is the major side effect to beware Agranulocytosis → necessitates weeklyof with clozapine? or biweekly white blood cells (WBC)

monitoring in patients on clozapinetherapy.

Psychopharmacology 171

Page 187: Deja review   behavioral science

What is the biggest benefit of clozapine? It is efficacious for treatment. It isefficacious for resistant schizophrenia,though because of the side effects andmonitoring it is never first line.

What is the mechanism of action of the It works by blocking both dopamineatypical antipsychotic risperidone? and serotonin receptors.

What are other side effects of Weight gain and metabolic syndromeantipsychotic therapy? Anticholinergic effects

Antihistaminergic effects/sedationAmenorrhea Temperature dysregulationNeuroleptic malignant syndromeBlood dyscrasiasSexual dysfunction

What is the cause of amenorrhea Dopamine receptor antagonism leadingin women taking antipsychotics? to elevated prolactin levels

Which pharmacologic agents are used Lithiumto treat acute mania associated with Atypical antipsychoticsbipolar disorder?

Valproic Acid (aka Depakote, valproate)CarbamazepineBenzodiazepines

What agents are used for maintenance Lithiumtherapy in bipolar disorder? Valproic acid

CarbamazepineAtypical antipsychoticsLamotrigine (Lamictal)

What are some of the side effects Polyuria → acts as an antidiureticof lithium? hormone (ADH) antagonist to cause

nephrogenic diabetes insipidus

TremorHypothyroidismWeight gainGastrointestinal effects (nausea,vomiting, diarrhea)Leukocytosis (high white bloodcell count)AcneNote: Use of lithium requires closemonitoring of serum levels.

172 Deja Review: Behavioral Science

Page 188: Deja review   behavioral science

Which pharmacologic treatment is safe None. Risk of relapse must be weighedin pregnant bipolar women? against the risks of birth defects.

Electroconvulsive therapy is likely thesafest treatment overall in pregnancy.

What is the main drawback of using Side effects, especially weight gainantipsychotics such as olanzapinefor bipolar maintenance?

Which anticonvulsants are used to Carbamazepinetreat bipolar disorder? Valproic acid

Lamotrigine

What type of therapy is used for major Electroconvulsive therapy (ECT)depressive order that is refractoryto treatment?

In what other psychiatric conditions Acute maniacan ECT be used as treatment therapy? Schizophrenia with catatonic symptoms

Depression, particularly with psychoticsymptoms or in pregnancyCatatonia

When does the maximum response After a period of several weeksto ECT usually occur? over which 5 to 12 treatments are

administered.

What is the biggest side effect of ECT? Memory loss (Retrograde amnesia),which may or may not be permanent.

How can the amnesia and memory Unilateral electrode placementloss associated with ECT beminimized?

Is modern ECT different than Absolutely. Use of appropriate20 years ago? anesthesia and modern ECT techniques

has significantly reduced side effectsand complications.

All possibilities considered, ECTwhat is the most efficacioustreatment for depression?

Psychopharmacology 173

Page 189: Deja review   behavioral science

CLINICAL VIGNETTES

An unresponsive young woman is brought to the ER by her college roommate.Her roommate states that she had confided in her that she was being treated fordepression for quite some time with little success. She found an empty pill bottleby the patient as well, but didn’t bring it in. On examination the patient haswidely dilated pupils, a dry mouth, and tachycardia.

What was likely in that bottle of pills taken by the patient?

She likely is suffering from tricyclic antidepressant overdose. Note the anticholinergicsymptoms and the limited success of treatment (perhaps necessitating a few differentmedication trials).

What diagnostic test would you order? What result do you expect on that test?

You should order an ECG, on which you would likely see a prolonged PR interval,a widened QRS, and a long QT interval.

What is the appropriate management of the patient?

Of course you would start with ABCs, but intravenous bicarbonate is a specifictreatment for TCA overdose.

A 62-year-old patient with schizophrenia comes in to inquire about new treatmentoptions. He has been on haloperidol which has controlled his symptoms well. Hispast medical history includes pre-diabetes, obesity (though he’s trying to loseweight), hypertension, and a family history of coronary artery disease. He says thathe has heard that some of the new atypical antipsychotics are more effective thanhis old agent.

What should you advise him?

Atypical medications are no more effective than typical agents, with the notableexception of clozapine. The main advantage of atypical agents is a more favorableside-effect profile with less EPS and tardive dyskinesia than typical agents.

What side-effects might you worry about in this patient?

Prominent metabolic effects and weight gain are side effects of atypical agents,which would be especially detrimental to this patient.

If he was treatment resistant, would this change your thinking?

Patients who have failed multiple other antipsychotic medications may benefitfrom clozapine—though the risk of agranulocytosis and subsequent FDA-mandated laboratory monitoring limit its widespread use.

174 Deja Review: Behavioral Science

Page 190: Deja review   behavioral science

S E C T I O N I I I

Ethics, Health Care,and Statistics

Page 191: Deja review   behavioral science

This page intentionally left blank

Page 192: Deja review   behavioral science

C H A P T E R 2 3

Clinical Practice andDifficult Situations

177

The USMLE sometimes gives “quote” questions asking you to pick something tosay in a provider-patient interaction. In general, it’s usually a good idea to get moreinformation, respect the patient’s autonomy, keep information confidential, andnever lie.

What is the importance of rapport? The relationship and trust built betweenthe doctor and patient greatly enhancethe effectiveness of care.

What are the most important Admit the mistake and apologize for it.things to do if a medical error Never try to cover up an error.is committed?

What should you do if a patient tries You should thank the patient for theirto give you a gift? thoughtfulness, but not accept it. Gift

giving may be misinterpreted as“buying better care”—equal care shouldbe given to all.

When is it appropriate to refer a patient Only when the problem cannot beto another physician? handled by yourself. This is rarely the

answer on the USMLE—it isinappropriate to refer a patient to apsychiatrist just because they have adifficult situation to be dealt with!

What would you say to a patient with Get more information about why theya terminal illness who wants to die? feel this way. Commonly, they are

afraid of dying in pain; they mayhave seen a loved one die painfully.Reassure them that you will stick withthem and that their symptoms willbe well controlled. Also assess foruntreated depression.

Page 193: Deja review   behavioral science

Your fellow resident gets called in to Yes. It’s unethical to endanger patientwork at night and you smell alcohol care with a possibly impaired physician.on his breath. He says he only had twobeers and asks you not to say anything.Do you tell someone?

A patient’s sibling asks you for No. You must have formal permissioninformation about their condition— from the patient to share their healthdo you give it to them? information with anyone other than

them.

Can you tell information about a No. This is a violation of the Healthpatient to someone uninvolved in Insurance Portability and Accountabilitytheir care? Act (HIPPA). You should be careful

where and with whom you discussany patient information.

You see a former patient at a bar, No. It is never ethical to have awhom you treated briefly 10 years ago. romantic relationship with a patient,Can you ask them out on a date now? former or current.

When is it OK to withhold information If the patient tells you they don’t wantabout an illness from a patient? to know. It must be the patient that

indicates this—not a family member.

A patient asks you to do a procedure No, you do not need to do anything northat is legal, but is against your belief treat anyone that would compromisesystem. Must you do it? your beliefs. However, you should refer

the patient to someone who will treatthem, as well as provide support in themeantime.

What is a good general approach to an Label and validate their emotions, thenemotional patient—be it angry, sad, offer support. Eg: “You sound like youor scared? are scared—I don’t blame you, it’s OK

to be scared about this procedure.”

What if the patient starts to cry? Let them cry. Try to be comfortable withsilence when appropriate. Offering atissue is always a good gesture.

What are the steps to giving bad news? 1. Set the stage—find a private placeand ensure you have a properamount of time free.

2. Find out what the patient knowsabout his/her illness.

3. Find out how much the patientwants to know.

4. Tell the information.5. Respond to feelings.6. Make a plan for next steps.

178 Deja Review: Behavioral Science

Page 194: Deja review   behavioral science

Which of the steps above is most likely Finding out how much the patientto be asked on the USMLE? knows about their illness (no 2). It’s

always good to get information first soyou can frame the discussion.

How do you use an interpreter? Speak to the patient, not the interpreter.Speak clearly, being sure to pauseintermittently to allow the interpreterto convey information to the patient.

Is it OK to allow a child or family No, not if it can be helped. Even if themember to act as an interpreter? family member is fluent, it is good to at

least offer to get an interpreter for thepatient. Family members may also havetheir own agenda that may undulyinfluence the patient.

What is patient adherence (formerly The degree to which the patient followscalled “compliance”)? the advice of the treating physician

What are barriers to patient adherence? Complicated medical regimensPoor physician-patient rapportLack of patient involvement in thetreatment planInfrequent appointmentsPoor support structure/inadequate helpat home

How might a physician make a The treatment plan can be simplified bytreatment plan less complicated? limiting:

• The number of medications thepatient is taking

• The number of times the medicationsis taken per day

• The number of changes made at eachvisit

What is an open-ended question? A question that is intentionally leftbroad, such that the patient can saywhat is really on their mind—“Whatbrings you into the clinic today?” or“Tell me about your stomach pain.”

What is a close-ended question? A very specific question with a discreteanswer. Used to get more detail—“Did you have nausea?” “Howfrequently do you have pain?”This is also useful with seductiveor disorganized patients.

Clinical Practice and Difficult Situations 179

Page 195: Deja review   behavioral science

What is the cone method of Starting with an open-ended question,interviewing, or “coning”? then progressively narrowing down to

more specific close-ended questions.

CLINICAL VIGNETTES

A 79-year-old woman came into the hospital complaining of shortness of breath.After an extensive workup, you find that she has metastatic lung cancer. Her sonfinds you in the hallway and states “In our culture our elders do not like to knowabout serious illnesses. It is my duty as a son to take care of my mother.” You areunclear about what cultural practices are appropriate to the patient. How do youapproach this situation?

It may very well be that there is a cultural aspect to discussions of terminal illnessthat is appropriate to consider in this case. If available in your institution, a “culturalconsult” may be appropriate with someone more versed than you in these issues. Ifnot, it would be inappropriate to withhold information based solely on what the sonof the patient says. The best case would be to sit down with the patient and ask herabout her cultural beliefs around illness—“Some members of your culture prefernot to talk about their illnesses, but instead allow their children to make medicaldecisions for them. How do you feel about the discussion of illness?”

A patient with known sickle cell disease comes into the ER. He states he is in 10/10pain and requests 15 mg of morphine. The nurse says he is a “frequent flier” andworries that he is an addict. He doesn’t look like he’s is in any pain. What shouldyou do?

Treat his pain. With known disease it is more important that he does not sufferthan to distinguish his symptoms from malingering. Remember, sickle cell crisiscan be extremely painful even without outward signs of pain.

You call a family meeting for a chronically ill patient whom you just found out hasleukemia. Everyone is comfortably sitting in your office. What do you say next?

“What do you know about your illness?” It is important to be open-ended and seewhat the patient’s thoughts are. Most likely he has been thinking about this muchmore than you and probably is very well informed; or he may have completelyunrealistic views that would be good to learn about before the conversation. Probea bit to understand this, and then to ascertain how much he wants to know beforedelivering the news.

180 Deja Review: Behavioral Science

Page 196: Deja review   behavioral science

C H A P T E R 2 4

Medical Ethics andLegal Issues

181

What are the four principles 1. Patient Autonomyof medical ethics? 2. Beneficence

3. Nonmaleficence4. Justice

What is Autonomy? The patient has the right to make theirown decisions regarding their care.

What is Beneficence? The principle that physicians should beof benefit to their patients.

What is Nonmaleficence? Do no harm.

What is Justice? The allocation of resources in a fair andjust manner, including equal treatment.

Of these principles, which one tends Patient Autonomyto take precedence in ethical questions,especially on the USMLE?

What is the principle of double-effect? When an action that has a primarilygood effect may also cause anunintended bad effect, but is stillpermissible.

What is the classic example of Giving morphine to relieve the pain of adouble-effect? dying patient, even if it causes respiratory

depression hastening the patient’sdeath. As long as the primary intentof the treatment is to reduce pain andnot to kill the patient, it is ethicallypermissible so long as there is painto be relieved.

Is malpractice a crime? No—malpractice is a civil court matter.

Page 197: Deja review   behavioral science

What four elements in a malpractice Duty: There must be an establishedcase must exist in order to find liability doctor-patient relationship.with the treating physician (otherwise Deviation or dereliction: Treatment referred to as the 4 D’s)? strayed from established standard of care.

Damages: Physical, psychological, orsocial damage was done.Direct cause: Damages were causeddirectly by negligence or dereliction.

How is a deviation from standard A jury decides if there has been aof care decided upon? deviation from the standard of care.

The jury makes this determination basedupon testimony of one or more expertwitnesses who testify as to the standardof care as generally recognized by themedical community and how thedefendant deviated from that standardof care.

What is the best way to avoid If an error is made, admit when you area malpractice suit? wrong and apologize to the patient.

Maintain a healthy physician-patientrelationship.Keep current with standards of care.

When can a physician become impaired? When clinical judgment is affected bythe following:• Physical illness• Mental illness• Substance abuse

Is it acceptable to work with an No. It is the ethical duty of a physicianimpaired health-care provider? to report an impaired health-care

provider to the proper authorities.

How does a physician’s human Under the American Medical Associationimmunodeficiency virus (HIV) status (AMA) ethical guidelines, an HIVaffect his or her ability to practice physician should not engage in any medicine? activity that would put a patient at risk

of contracting HIV (eg, an HIV-positivesurgeon may be precluded frompracticing surgery. Some states requirethat an HIV-positive physician disclose,as part of the informed consent process,his or her HIV status before engaging inan invasive procedure that would putthe patient at risk).

182 Deja Review: Behavioral Science

Page 198: Deja review   behavioral science

Is it acceptable for a physician to No—it’s not acceptable under anyestablish a romantic relationship circumstances.with a patient or formerpatient?

What is competence? Competence is the ability to evaluatesituations and make sound judgmentsthat are:• Consistent throughout time• Consistent with the patient’s belief

system (unless the belief system isdelusional)

Who is considered legally competent? All adults >18 years of age andemancipated minors—includingadults with mental illness or mentalretardation—unless declaredincompetent by a court of law

Who is an emancipated minor? Someone <18 years of age who is:• Financially independent of his or her

parents• Married• Serves in the armed forces

Can a person be competent in some Yes. Legal competence is situationareas and not others? specific (eg, competence to stand trial)

and can change with time. A patientcan be legally competent, but notmedically competent if he or she lacksthe capacity to make, understand, orcommunicate his or her health-caredecisions.

Can a physician deem a patient No. Legal incompetence is determinedincompetent? in a court of law. Any physician (not

just a psychiatrist) can determine if apatient has the capacity to makemedical decisions.

If a patient is ruled incompetent, how The case must be brought before the does the patient become competent judge again, and after reviewing the again? medical evidence, the judge determines

whether the patient has regained legalcompetence.

Medical Ethics and Legal Issues 183

Page 199: Deja review   behavioral science

Who may give informed consent The patient only unless:for a patient? • It is an emergency, delay puts the

patient’s life at risk, and there is noone available to give consent on thepatient’s behalf.

• The patient is legally incompetent(depending upon state law, thelegally appointed guardian, or theagent named in a durable power ofattorney for health care givesconsent).

• The patient lacks the capacity tomake, understand, or communicatehis or her health-care decisions (asurrogate decision-maker givesconsent as provided by state law,usually in the following order ofpriority: health-care agent, spouse, anadult child, parent, an adult sibling,or a grandparent).

• The patient is a nonemancipatedminor (parent or legal guardian givesconsent, and in some states, someonestanding in the place of a parent mayalso give consent).

What happens if a patient lacks A surrogate decision-maker may give the capacity to make, understand, or consent on the patient’s behalf and communicate his or her health-care should make a good-faith effort to make decisions and has not named a health- decisions based upon what the patient care agent under a durable power would have chosen. State law designatesof attorney for health care or been a list of surrogate decision-makers, appointed a legal guardian by usually in the following order of prioritythe court? (in the absence of a health-care agent or

legal guardian): spouse, adult child,parent, adult sibling, or a grandparent.State laws may differ.

What is an advance directive? A decision made by a patient aboutwhat type of medical care he or shewishes to receive in case that he orshe is not able to make decisions in thefuture.

What is a durable power of attorney A durable power of attorney for healthfor health care? care (or health-care proxy) is a legal

document that allows the patient todesignate a health-care agent to makehealth-care decisions on behalf of thepatient when and if the patient is unableto do so. It is an advance directive.

184 Deja Review: Behavioral Science

Page 200: Deja review   behavioral science

What is a living will? A living will is a legal document thatallows a patient to decide, in advance,whether he or she wants to be keptalive by artificial means if two doctorsdiagnose that the patient is (1) terminallyand incurably ill, (2) in a persistentvegetative state, or (3) in an irreversiblecoma. State laws may differ as toconditions under which a living willmay be honored.

What is a DNR (do not resuscitate) order? A DNR order is an order writtenby a physician, after determiningwhether the patient is a candidatefor nonresuscitation and obtaining theappropriate consent that directs medicalpersonnel not to resuscitate a patient inthe event of cardiopulmonary arrest.

Do DNR orders mean no interventions No. Patients may choose to still haveare done on a patient? antibiotics, fluids and nutrition, or other

“non-heroic” means depending on theirwishes.

When does a judge consent for medical When a parent or legal guardian refusestreatment of a minor? to consent to medical treatment on the

minor’s behalf, and the physicianbelieves that the treatment is medicallynecessary and justifies court intervention.A court is more likely to intervene whenthe proposed treatment carries a lowrisk and high benefit or when theminor’s life is threatened.

Does this apply to a fetus? No. The competent pregnant mother, inmost circumstances, has a right to refuseany intervention on the part of the fetuseven if it compromises her own or thefetus’ life. In some states, courts haveintervened on behalf of a viable fetus.

Under what circumstances can a Although the answer is state dependent,nonemancipated minor receive most states allow minors to consent fortreatment without the consent treatment involving sexually transmittedof his or her parent or legal guardian? diseases (STDs), contraception and

pregnancy, and alcohol or illegalsubstance use. Some states also allowminors to consent for an abortion.Other states allow a minor to consentto an abortion, but require parentalnotification or a court order waivingparental notification.

Medical Ethics and Legal Issues 185

Page 201: Deja review   behavioral science

When can a patient refuse treatment Anytime, as long as they have theeven though refusal is life threatening? capacity to refuse the treatment, and

the refusal is an informed refusal.

Does this include artificial Yeslife support?

Is removing artificial life support the No. Removing artificial life support is asame as physician-assisted suicide? decision that the competent, informed

patient is allowed to make. It does notaccelerate the natural course of thepatient’s disease process. Physician-assisted suicide is illegal in most statesbecause it purposely acceleratesdeath.

When can a physician decide to remove A physician may remove life support life support without the consent of the without consent if the patient is legallypatient or patient’s decision-maker? dead. The patient must be “brain dead”

in order to be declared legally dead inthe United States, which includes globaldysfunction of the brain (coma) andabsent brainstem reflexes.

Is palliative care of the terminal patient No. Palliative care is not done with the the same as euthanasia? intention of accelerating death. It is

done with the intention of making thepatient comfortable through the naturalend point of a terminal illness. However,palliative care may unintentionallyaccelerate death as a side effect.

What is an informed consent? Informed consent is required if theproposed treatment or procedureinvolves a material risk to the patient.An informed consent includes thevoluntary agreement by a patient toproceed with treatment after thephysician has discussed the procedure,risks, benefits, and outcome of theprocedure or treatment, alternativetreatments (including no treatment),and the risk, benefits, and outcomesof those alternatives.

Does consent have to be written? No. Basic consent (eg, consent totouching contact during a physicalexamination) need not be in writing andis often implied. An informed consentshould be in writing and documentedin the chart.

186 Deja Review: Behavioral Science

Page 202: Deja review   behavioral science

Can a physician refuse to treat patients Generally, a physician may refuse tobased on race, financial status, and treat a patient as long as the reasons forpresence of mental illness or refusal are not illegal. Illegal reasons toHIV status? refuse treatment include race, national

origin, gender, religion, and disability(which include mental illness andHIV status). Except in emergencysituations, a physician may refuse totreat a patient based upon inabilityto pay.

If a patient is homicidal, is a physician Yes. In almost all states, the Tarasoff allowed to break patient confidentiality? decision applies, which requires a

physician to warn the person in severedanger and notify law enforcement. Instates such as Georgia, Tarasoff has notbeen adopted. Instead, the physicianhas a duty to prevent harm by thepatient if there is a right to control. Inother words, if a physician has thelegal right to initiate involuntarilycommitment proceedings and fails toexercise this right, the physician may beheld liable for harm done by the patientto third parties if the harm wasforeseeable. Georgia has not specificallyrecognized a duty to warn, andthus, a physician may be breachingconfidentiality in warning anintended victim.

In what other circumstances is a • A patient is considered suicidalphysician required to break patient • Reporting child abuse, elder abuse, confidentiality by law? or domestic violence (call Child

Protective Services, Adult ProtectiveServices, or the police)

• Court order (except to the extent theinformation is privileged)

• Patient driving without cognitiveabilities to do so (some stateshave specific procedures andforms)

• Reporting certain infectiousdiseases, including HIV/acquiredimmunodeficiency syndrome(AIDS)

Medical Ethics and Legal Issues 187

Page 203: Deja review   behavioral science

In what other circumstances If the patient signs a written authorization,can a physician breach patient the physician may disclose privateconfidentiality? health information consistent with the

authorization. State law may alsoauthorize a physician to share privatehealth information with specified peoplewho are at risk for contracting HIV froma patient. Disclosure of this informationis controlled by state law, and physiciansmust proceed carefully because if statelaw does not authorize disclosingHIV/AIDS information, the physicianmay be guilty of violating HealthInsurance Portability and AccountabilityAct of 1996 (HIPAA) which may resultin fines and a prison sentence.

When can a patient be “committed” The process varies by state, but a patientor involuntarily hospitalized? who presents a substantial risk of

imminent harm to himself or herself orothers, or if a patient is so unable to carefor his or her own physical safety as tocreate an imminently life-endangeringcrisis, then the patient needs involuntaryinpatient treatment.

If a patient is involuntarily hospitalized, No. Even an involuntarily committedcan a physician administer any patient has the right to refuse medicaltreatment they want? treatment.

How do mentally ill patients receive A judge can order the administrationtreatment even if they refuse treatment? of treatment if the patient is found

incompetent to refuse treatment. Also,if the patient is violent and posing animmediate, severe danger to himself orherself or others, this is considered amedical emergency and the physicianmay administer treatment without thepatient’s consent.

CLINICAL VIGNETTES

Your patient has metastatic lung cancer and he is nearing the last weeks of his life.He is in considerable pain and is having trouble breathing without beingintubated. His wishes were to not be intubated or having any life-prolongingmeasures. His son asks if you can do anything for his pain. You ask the nurseabout giving him more morphine and she says that the respiratory depression willkill him. Is it ethical to do this?

Yes—it is ethically permissible under the principle of double-effect even if itcauses his death, as long as your primary goal is to treat his pain.

188 Deja Review: Behavioral Science

Page 204: Deja review   behavioral science

A 72-year-old woman comes in with a gangrenous foot. The orthopedic servicewould like to amputate the foot to save her life. You are worried about the risksinvolved in the surgery. The man in the bed next door also needs a surgicalprocedure, but he is unable to afford it. The woman with the gangrenous footdoesn’t want the surgery because she wants to keep herself “whole.” Of the fourparties involved in this story (Patient, Orthopedic service, the man next door, andyou), which ethical principles is each representing?

Patient: Patient AutonomyOrthopedic service: BeneficenceMan next door: JusticeYou: Nonmaleficence

Medical Ethics and Legal Issues 189

Page 205: Deja review   behavioral science

This page intentionally left blank

Page 206: Deja review   behavioral science

C H A P T E R 2 5

Health Care in theUnited States

191

HEALTH-CARE INSURANCE

What is the primary difference in The United States is the only industrializedhealth-care insurance coverage in the country without government-fundedUnited States as compared to other health care for all citizens.industrialized countries?

What options for insurance coverage Employer-Sponsored Programsare there and how do most people (most common)get their coverage? Individual Policy (self-purchased)

MedicareMedicaid

What is Medicaid? Government-funded program forlow-income people below a certainincome

What is Medicare? Government-funded program for those>65 years or those who are disabled orthose on dialysis

What are the “Parts” of Medicare? Part A: Hospital coveragePart B: Outpatient coveragePart C: Private supplement to A and BPart D: Prescription drug coverage

Does Medicare have a premium, Yes—they vary, but there are out-co-pay, and/or deductible? of-pocket expenses for Medicare.

What is the group of people most likely Those who are employed, make too muchto be uninsured or have no affordable money to qualify for Medicaid, and doavailable coverage? not receive benefits through their

employer and/or cannot affordpremiums.

Page 207: Deja review   behavioral science

How many people do not have health The number continues to rise, butinsurance in the United States? approximately 45 million plus people.

What are some of the disadvantages It may be very difficult to qualify forof self-purchased individual these if you have any illnesses. Theyhealth-care plans? can also be prohibitively expensive.

What is “pooled risk”? Insurance companies make money byhaving the average premium of theirinsured group be less than the totalamount they pay out for that samegroup. Each person has a certainamount of “risk” for developingdisease. Combining the risk to eachperson in a group is “pooling risk.”

What is fee-for-service? Each service that is done is billedseparately (eg, a doctor visit, x-ray,lab test, and surgical procedure areeach charged for separately).

Who pays more for the same The uninsured patient. Insuranceservice—an uninsured patient companies negotiate discounted rate paying out of pocket or an insurance with physicians and hospitals—if a company? patient pays out of pocket, they have

to pay full price.

What is a Diagnosis-related Group With DRGs, a hospital is paid a set(DRG)? amount for an individual diagnosis, no

matter how long it takes to get better.For example, Patient A gets pneumoniaand stays in the hospital 2 days, Patient Bgets pneumonia also and takes a 10-daystay to recover. Medicare pays the hospitalthe same $1500 for Patient A as forPatient B.

What two health-care costs are not 1. Long-term nursing carecovered by Medicare? 2. Outpatient prescription drugs

Which government-funded insurance Medicare. Medicaid receives fundingplan is funded completely by the from the federal and state governments federal government? both.

What are the primary sources of funding Employee benefit.for health-care insurance for US citizens? People obtain health-care insurance on

their own.

Which not-for-profit insurance carrier Blue Cross/Blue Shieldprovides insurance for 30% to 50% Note: Blue Cross covers hospital costs.of working citizens in all 50 states? Blue Shield covers diagnostic tests and

physicians’ fees.

192 Deja Review: Behavioral Science

Page 208: Deja review   behavioral science

What are the two primary health plans 1. Fee-for-service plansoffered by private health-care insurers? 2. Managed-care plan

Which type of health-care insurance Fee-for-service plansplan has high premiums and does notimpose restrictions on provider’schoice?

What are the primary characteristics Low premiumsof a health-care managed-care plan? Restrictions on provider’s choice

What are the types of health-care Health maintenance organizationmanaged-care plans? (HMO)

Preferred provider organizations (PPOs)Point of service (POS)

What is a health maintenance An HMO has arrangements withorganization (HMO)? health-care providers to form a

“network.” The providers give theHMO a discounted rate in exchange forreceiving health plan referrals. Membersmay only see physicians in this network,otherwise the costs are not be coveredby the plan. Members select a primarycare physician (PCP), often called a“gatekeeper,” who provides, arranges,coordinates, and authorizes all aspectsof the member’s health care.

What is meant by a physician being a Often the primary care physician (PCP)“gatekeeper”? in an HMO plan must approve specialist

and ancillary service referrals as well ascoordinate the member’s care.

What are preferred provider PPOs are similar to HMOs in that theyorganizations (PPOs)? have arrangements with a “provider

network.” Unlike an HMO, membersmay choose a doctor outside of thisnetwork, but their coverage will be at areduced rate (ie, out-of-pocket expensesare higher).

HEALTH-CARE COSTS

How does the United States compare The United States spends significantlywith other countries in terms of more per capita and as a percentagehealth-care spending? of GDP on health care than any other

nation.

Health Care in the United States 193

Page 209: Deja review   behavioral science

How is the US life expectancy The 2009 CIA (Central Intelligencecompared with other countries? Agency) World Factbook ranks

United States at no. 50 out of 224—despite the increased spending.

Which factors have contributed to the Larger percentage of elderly individualsincrease in health-care expenditures? Overuse of medical technology

advancesMedicaid and Medicare expenditures

During what time in a person’s life The last year of their lifespan is the most money spent?

What are the most expensive 1. Hospitalizationcomponents of health care in the 2. Physician costsUnited States (rank items from 3. Nursing home costsmost expensive to least expensive)? 4. Prescription drugs

5. Medical supplies6. Mental health services7. Dental and other care

HEALTH-CARE DELIVERY SYSTEMS

What is the role of nursing homes? To provide long-term care, especially forindividuals aged 65 years and older

What percentage of the elderly population 5%uses nursing home services?

What is the range of costs spent on an $35,000 to $75,000—depending uponindividual that resides in a nursing nursing home level of carehome?

What is hospice care? A unique type of service provided topeople with terminal illness and a lifeexpectancy thought to be <6 months.

What are the goals of hospice and Palliation refers to the treatment of palliative care? symptoms rather than a focus on curing

underlying disease. Patients on hospicegenerally are given more palliation thancurative interventions.

What is “comfort care” and do hospice Comfort care refers to administering nopatients have to be on it? life-prolonging interventions. Hospice

patients generally will not receivecardiopulmonary resuscitation (CPR)or other “heroics,” but may receivenutrition, fluids, antibiotics, andother appropriate interventions.

194 Deja Review: Behavioral Science

Page 210: Deja review   behavioral science

Is hospice expensive to the health-care No—patients generally receive moresystem? services with less expenditure than if

they were to be repeatedly hospitalized.

HEALTH STATUS AND DETERMINANTS

What is the percentage of physical Approximately 70%illness that is due to individualpatterns of living (eg, lack of exercise,poor dietary choices, and smoking)?

What are the primary determinants Education levelof socioeconomic status? Income

OccupationResidence

Which socioeconomic group tends People of low socioeconomic status →to delay seeking health care and due to lack of funds for health carepresent with more progressiveillnesses?

Which gender is most likely to seek Femalemedical care?

Which gender has the lowest life Maleexpectancy?

Which group of people has the lowest Black maleslife expectancy with regards to raceand gender?

Which group of people has the highest Asian womenlife expectancy with regards to raceand gender?

Which group of people represents Elderlythe fastest growing segment of theUS population?

What is the percentage of health- 30%care costs the elderly population isresponsible for?

Which ethnic minority represents the Hispaniclargest percent of the US population?

What are the leading causes of death Heart diseasein the United States, irrespective of Cancerage group?

Stroke

Health Care in the United States 195

Page 211: Deja review   behavioral science

What are the three leading causes 1. Congenital anomaliesof death in infants <1 year of age 2. Sudden infant death syndrome (SIDS)(rank in order from most common 3. Respiratory distress syndrometo least common)?

What are the three leading causes 1. Accidentsof death in children between 1 and 2. Congenital anomalies4 years of age (rank in order from 3. Cancermost common to least common)?

What are the three leading causes 1. Accidentsof death in children between 5 and 2. Cancer14 years of age (rank in order 3. Homicidefrom most common to leastcommon)?

What are the three leading causes 1. Accidents → mostly motor vehicleof death in young adults between 2. Homicide15 and 24 years of age (rank in order 3. Suicidefrom most common to leastcommon)?

What are the three leading causes 1. Accidentsof death in adults between 25 and 2. HIV infections44 years of age (rank in order from 3. Cancermost common to least common)?

What are the three leading causes 1. Cancerof death in adults between 45 and 2. Heart disease64 years of age (rank in order from 3. Strokemost common to least common)?

What are the three leading causes 1. Heart diseaseof death in elderly over 65 years 2. Cancerof age (rank in order from most 3. Strokecommon to least common)?

CLINICAL VIGNETTES

A law is passed saying that private insurance companies can charge no more than$250 per month per person it insures. How can this company continue to makemoney?

By enrolling only those people who are in excellent health and have low risk ofdisease. This is referred to as “cherry picking” or “cream skimming.” Of course, bytaking these healthy people out of the risk pool, those that are left over are sicker(and riskier) overall—creating a group of difficult-to-insure patients who need care.

196 Deja Review: Behavioral Science

Page 212: Deja review   behavioral science

A 50-year-old woman is shopping for insurance. She has rheumatoid arthritis andtakes some very expensive monoclonal-antibody agents which keep her veryfunctional, and would like to keep her own doctor. She is employed making$60,000 a year and has the option of insurance through her employer. Whichoptions are best for her: employer-sponsored insurance, individually purchasedplan, Medicare, or Medicaid? HMO or PPO?

She is neither 65 years of age or older nor disabled, so would not qualify forMedicare. Similarly, she is not low income and would not be eligible for Medicaid.Since she has a preexisting condition of rheumatoid arthritis for which she takesexpensive medication, an individual plan may be difficult for her to obtain. Assuch, employer-sponsored health insurance would be best. The best way to keepher own doctor would likely be through a PPO, where she could go out of theprovider network if needed to stay with her doctor.

Health Care in the United States 197

Page 213: Deja review   behavioral science

This page intentionally left blank

Page 214: Deja review   behavioral science

C H A P T E R 2 6

Epidemiology andResearch Design

199

EPIDEMIOLOGY

What is prevalence? The total number of individuals whohave a disease in a population at aspecific time or period of time, dividedby the total number of people in thatpopulation.

What is incidence? The total number of individuals whoare newly diagnosed with a diseasedivided by the number of individualswho are originally at risk (usuallymeasured over a time period).

What equation represents Prevalence = incidence × disease duration the relationship between P = I × Dprevalence and incidence?

What happens to the prevalence It would increase—use the equation.of a disease as the duration of it islengthened, assuming the incidenceis held constant?

Which is usually greater in chronic Prevalence is usually greater with longdiseases—prevalence or incidence? duration diseases, eg, HIV.

Which is more useful for describing Incidence. With acute illnesses, durationinfluenza over the course of a year— is short and prevalence at any givenincidence or prevalence? time is not of as much value as incidence.

The shorter the illness, the closerprevalence becomes to being equalto incidence.

Page 215: Deja review   behavioral science

What is Sensitivity? The ability of a test to detect disease if itis present

What is Specificity? The ability of a test to discern onedisease from another

What is best for a screening test A test with a high sensitivityor to “rule out” disease (low falsenegative)?

What is best for a confirmatory True negatives divided by all of thetest or to “rule in” disease people without a disease or illness(low false positive)?

What is a good mnemonic to SPin (SPecificity rules in) and SNoutremember this? (SeNsitivity rules out)

What is positive predictive value (PPV)? The likelihood that a positive result on atest represents actual disease

What is negative predictive value (NPV)? The likelihood that a negative result ona test actually represents the absence ofdisease

What is the relation between sensitivity, Sensitivity and specificity are directlyspecificity, NPV, and PPV? related to NPV and PPV (respectively),

but NPV and PPV occur when a test isapplied to a specific population.

What is the difference between Sensitivity and specificity are inherentsensitivity, specificity, NPV, properties of a test, regardless of the and PPV? population’s disease prevalence. PPV

and NPV are directly dependent onthe population being tested.

How do sensitivity and specificity A high sensitivity will increase NPV ofaffect NPV and PPV? a test, a high specificity will increase the

PPV of a test.

How does prevalence of disease in a High prevalence of disease will increasepopulation affect PPV and NPV? the PPV and decrease the NPV of a test.

Low prevalence will decrease the PPVand increase the NPV. (This shouldmake sense!)

What is the best way to solve problems Use the box method—draw it out andrequiring the calculation of sensitivity, plug in the numbers for each problem.specificity, PPV, and NPV?

200 Deja Review: Behavioral Science

Page 216: Deja review   behavioral science

How is sensitivity calculated? True positives divided by all of theindividuals with a disease

Sensitivity = a/(a + c)

How is specificity calculated? True negatives divided by all of thepeople without a disease

Specificity = d/(b + d)

How is positive predictive value True positives divided by all people calculated? with a positive test

PPV = a/(a + b)

How is negative predictive value True negatives divided by all people calculated? with a negative test

NPV = d/(c + d)

RESEARCH STUDY DESIGNS

What is a prospective study? A research study that starts beforethe studied disease or outcome hashappened, and usually monitors forits development.

Epidemiology and Research Design 201

a b

c d

Actual disease state

Test result

a = True positivesb = False positivesc = False negativesd = True negatives

All patients with disease = (a + c)All patients without disease = (b + d )All positive tests = (a + b)All negative tests = (c + d )

Sensitivity = a/(a + c)Specificity = d/(b + d )PPV = a/(a + b)NPV = d/(c + d )

–+

+

Figure 26.1

Page 217: Deja review   behavioral science

What is a retrospective study? A research study that starts after thestudied disease or outcome hashappened, and usually looks backfor factors that led to it.

What is meant by a study being The researcher separates subjects into“observational”? groups and merely waits for the

outcome in question to happen.

What is meant by a study being The researcher separates subjects into“experimental”? groups and applies an intervention to

one or more of those groups, hopingto modify the outcome.

What is a case-control study? An observational study that comparessubjects who have a disease or outcome(cases) with subjects who do not havean illness or outcome (controls).Note: Groups are divided based ondisease presence (outcome).

What is a cohort study? An observational study that comparessubjects with and without a certain riskfactors or exposure. This study thenfollows subjects for the developmentof disease.Note: Groups are divided based on riskfactor (exposure).

What are the two types of cohort studies? 1. Prospective 2. Retrospective

What is a prospective cohort study? A prospective cohort study evaluates acohort of individuals after they haveexperienced the risk factor/exposure,but before development of disease/outcome. (Eg, a study is constructed toevaluate whether children exposed tosecondhand smoke at birth will be moresusceptible to lung cancer than thosechildren not exposed, started whenchildren are 5 years old).

What is a retrospective cohort study? A retrospective cohort study evaluates acohort of individuals after both the riskfactor/exposure and the disease/outcomehave happened. (Eg, a study of if childrenexposed to secondhand smoke at birthwill be more susceptible to lung cancerthan those children not exposed, startedat age 80).

202 Deja Review: Behavioral Science

Page 218: Deja review   behavioral science

What is a cross-sectional study? A descriptive study (neither observationalnor experimental) that compares subjectsin respect to both risk factor/exposureand disease/outcome at one specific pointin time.

What is a clinical trial? An experimental study in which groupsof subjects are given different treatmentsor interventions to determine if there isan effect.Note: Often there is an experimentalgroup that receives the studiedintervention and a control group thatreceives either placebo or a different“standard” treatment.

What are two primary characteristics They are double-blinded and patientsof high-quality clinical trials? are randomized to be in either the

experimental or control group.

TESTING

What are the attributes of useful testing Lacks biasinstruments? Reliably Valid

Name the types of bias that may be Admission rate biasprevalent in a research study. Lead time bias

Nonresponse biasSampling biasSelection bias

Which bias can occur when hospital A Admission rate biasadmits sicker patients than hospital B?

Which bias can occur when a disease Lead time biasor illness is detected earlier, leadingto seemingly increased survival time?

Which bias can occur when people fail Nonresponse biasto return surveys or respond to a phoneor email survey?

Which bias can occur when a study Sampling biasfavors selecting subjects that havea particular characteristic or set ofcharacteristics?

Epidemiology and Research Design 203

Page 219: Deja review   behavioral science

Which bias can occur if the subjects Selection biasstudied are not representative of thetarget population about whichconclusions are drawn?

What percentage of patients generally Usually at least 33% of patientsrespond to placebos?

What happens to the response rate to It increases in psychiatric illnesses.placebos in psychiatric conditions?

What is the definition of a double- A study in which neither the research blind study? scientist nor the subject knows which

participants are in the experimentalgroup and which are in the controlgroup.

If initially group A receives a treatment Crossover studiesand group B receives a placebo andlater the protocol is switched so thatgroup A receives the placebo andgroup B receives a treatment,what type of research study is being utilized?

What is reliability? It is the reproducibility of a given test.

What type of reliability is demonstrated Interrater reliabilitywhen different examiners are able toachieve test results that are similar?

What type of reliability is demonstrated Test-retest reliabilitywhen subsequent tests yield similarresults to initial tests?

What is the definition of validity? It determines whether a test measureswhat it is supposed to measure.

What is the definition of precision? It is the consistency and reproducibilityof a test. On a dartboard, it would behitting the same place over and over(though not necessarily the correctplace).

What is the definition of accuracy? It determines how true test measurementsare. On a dartboard, it would be hittingthe correct place (though perhaps notconsistently).

204 Deja Review: Behavioral Science

Page 220: Deja review   behavioral science

MEASURES OF ASSOCIATION

Which measures are used to quantify Relative riskrisks in population studies? Attributable risk

Odds ratio

Which measure(s) of association is used Relative risk to evaluate cohort studies? Attributable risk

Which measure(s) of association is used Odds ratioto evaluate case-control studies?

How does our four-box method The left heading of “test result” is for calculating specificity and sensitivity changed to “exposure/risk factor” change when calculating risk? (the top heading stays the same).

Epidemiology and Research Design 205

a b

c d

Disease state

Exposure/riskfactor

a = Exposed subjects with diseaseb = Exposed subjects without diseasec = Unexposed subjects with diseased = Unexposed subjects without disease

All with disease = (a + c)All without disease = (b + d )All exposed = (a + b)All unexposed = (c + d )

RR = ([a/(a + b)] / [c/(c + d )] )AR = ([a/(a + b)] – [c/(c + d )] )OR = ad/bc

+

–+

What is relative risk (RR)? RR is a measure of the likelihood ofdisease in exposed subjects comparedto unexposed. It is expressed as a ratio,so an RR = 1.0 is the risk of theunexposed subjects.

How is RR calculated? It is the incidence of disease in theexposed group divided by the incidenceof disease in the unexposed group.

RR = [a/(a + b)]/[c/(c + d)]

At what value is RR significant? When RR is greater than or less than 1.0

Figure 26.2

Page 221: Deja review   behavioral science

If the RR is >1, what can be said about There is an increased risk of the diseasethe risk? or illness.

If the RR is <1, there is a decreased riskof illness or disease.

What is the definition of Attributable AR is used to determine how much risk Risk (AR)? is caused by a single risk factor (eg, how

likely is a high cholesterol diet to leadto atherosclerosis?).

How is AR calculated? AR is the disease rate in exposedsubjects minus that in unexposedsubjects.

AR = [a/(a + b)] − [c/(c + d)]

If the incidence rate of atherosclerosis 40/100 → This answer assumes thatin the general population in Atlanta, there is a properly matched control.GA is 10/100 and in individuals with ahigh cholesterol diet is 50/100, what isthe attributable risk?

What is an odds ratio (OR)? This determines the relative risk if theprevalence of disease is low.

OR = (a/c)/(b/d) = ad/bc

How can you interpret odds ratio in In most situations, odds ratio is a comparison to relative risk? reliable way to estimate relative risk

(interpret them the same).

What would be concluded about a study There would not be a statisticallyif the 95% confidence interval for RR significant result of the study (p = >0.5).or OR crosses 1?

CLINICAL VIGNETTES

How have advances in antiretroviral therapies influenced the prevalence of HIV,assuming incidence of HIV has remained constant?

HIV has an increased prevalence due to the lengthening of life expectancy ofinfected patients. Using P = I × D, and assuming the medications have little effecton incidence, as duration is increased, so is prevalence. (Note that HAART doessignificantly decrease the transmission of HIV.)

If you are told that almost all children with chickenpox have a fever, but that fevercan be a feature of many illnesses. How would you describe fever in terms ofsensitivity and specificity in regards to chickenpox?

Sensitivity would be high (someone without fever would be unlikely to havechickenpox), specificity would be low (fever could be from many diseases).

206 Deja Review: Behavioral Science

Page 222: Deja review   behavioral science

Two groups of male patients are looked at over a 10-year period, ones that like togo to the doctor regularly for checkups and ones that don’t get much care at all. Itis found that the group which goes to the doctor regularly has a longer survivaltime after diagnosis of prostate cancer than the ones who don’t get much care.What type of study design is this? What type of measure of risk would you use tocompare the groups? What is a likely potential type of bias in this study?

This is a cohort study—it separates groups based on the risk factor/exposure ofhealth-care utilization and then monitors for the development of disease anddeath. Cohort studies use relative risk measures (as opposed to case-controlstudies which use odds ratios). This study likely suffers from lead time bias as thesurvival time after diagnosis is directly dependent on how early a patient isscreened for disease—likely more often for those going to the doctor regularly.

A new laboratory test for depression is developed. Nearly 2500 patients are tested,800 with depression and 1700 without depression. There are 720 true positives,1500 true negatives, 80 false negatives, and 200 false positives. What is thesensitivity of test? What is the specificity of the test? If a patient from this samepopulation tests positive for depression, how certain are you that the patientactually has depression? How about if the same patient tested negative—how sureare you that he/she doesn’t have depression?

First off, make a four-box diagram, as done below. You should start with this everytime. The bold numbers are row and column totals. Remember that a is truepositives, b is false positives, c is false negatives, and d is true negatives.

Epidemiology and Research Design 207

Disease

+ −

Exposure + 720 200 920

_ 80 1500 1580

800 1700 2500

For the first two questions, plug in numbers. Sensitivity = 720/(80 + 720) = 90%.Specificity = 1500/(1500 + 200) = 88%. The second two questions are really askingyou to calculate the positive predictive value and negative predictive value,respectively. PPV = 720/(720 + 200) = 78%. NPV = 1500/(1500 + 1580) = 95%.

Page 223: Deja review   behavioral science

This page intentionally left blank

Page 224: Deja review   behavioral science

C H A P T E R 2 7

Biostatistics

209

STATISTICAL DISTRIBUTION

What are the three measures of central 1. Meantendency? 2. Median

3. Mode

What is the definition of mean? The average of a set of numbers

What is the definition of median? The middle number in a set of numberswhen they are put in sequential orderNote: If there is an even amount ofdata in a set, the median is theaverage of the two middle valuesin the data set.

What is the definition of mode? The number that appears mostfrequently in a set of numbers.

Using the following set of numbers, Mean = [(1 + 2 + 3 + 4 + 5 + 6 + 7 + 8 + what are the mean, median, and mode? 9 + 10 + 11 + 12 + 12)/13] = 90/13 = 6.9231, 2, 3, 4, 5, 6, 7, 8, 9, 10, 11, 12, 12 Median = 7

Mode = 12

What is the range of a data set? The difference between the highest andlowest values in a data setNote: The range in the data set above is12 − 1 = 11.

Define normal distribution. A set of numbers in which the mean,median, and mode are equal.Mean = median = mode

What type of curve demonstrates Gaussian or bell-shaped curve a normal distribution?

Page 225: Deja review   behavioral science

When a data set shows a large number Negatively skewedof high values and a small number Mean < median < modeof low values, what is the distribution

Note: The tail of the curve is on the leftof the data set?or on the negative end of the number line.

When a data set shows a large number Positively skewedof low values and a small number of Mean > median > modehigh values, what is the distribution

Note: The tail of the curve is on the of the data set?right or on the positive end of the number line.

When a data set shows two humps, Bimodalwhat is the distribution of the data set?

210 Deja Review: Behavioral Science

Normal distribution

Bimodal distribution

Positively skewed

Negatively skewed

Figure 27.1 Statistical frequency distributions.

What is a variable? It is a quantity that changes throughouttime.

What is an independent variable? It is an attribute that the researchscientist may adjust in an experiment.

Page 226: Deja review   behavioral science

What is a dependent variable? It is the outcome that is measured andaffected by the experiment.

How do I remember this? The dependent variable is dependent onthe independent variable.

What is the correlation coefficient (r)? It expresses the strength of a relationshipbetween two variables. Its value mustbe between −1.0 and +1.0.Note: The minus sign (−) implies anegative correlation and the plus sign(+) implies a positive correlation.

Why is it important to take the absolute The absolute value of r will determinevalue of the r? the strength of the correlation.

What is the definition of standard It is the root mean square deviation from deviation (σ)? the average. The standard deviation is

defined as the square root of the variance.

How is standard deviation used? It is a measure of how dispersed a dataset is around the mean. With a large σ,data will be more spread out than with asmall one—even if the mean is the same.

Biostatistics 211

0.15% 2.35% 13.5% 34.0% 34.0% 13.5% 2.35% 0.15%

Mean +1 +2 +3−3 −2 −1

68%

95%

99.7%

Figure 27.2 Normal (Gaussian) distribution represented by a Bell-Shaped Curve. The (+) and (−)numbers under the curve correspond to the standard deviations from the mean.

Page 227: Deja review   behavioral science

What percentage of data falls in 68%1 standard deviation from the mean?

What percentage of data falls in 95%2 standard deviations from the mean?

What percentage of data falls in 99.7%3 standard deviations from the mean?

STATISTICAL HYPOTHESIS AND ERROR TYPES

What is meant by “null hypothesis The idea that, by default, there are notesting”? relationships between variables. Research

must then be done to disprove this—called “rejecting the null.”

What is the null hypothesis (H0)? A hypothesis stating that there is nodifference between an experimentaland control group. (ie, variables areunrelated)

What is the alternative hypothesis (H1)? A hypothesis stating a relationshipbetween variables, or an effect on anexperimental group different than thecontrol group. If this was true, youwould “reject the null.”

Which would be true if there was no You would accept the null hypothesis.effect in your research study?

Which would be true if there were an You would reject the null hypothesis.effect in your research study?

What is a type I (α) error? A type I error rejects the null when thenull is actually correct—this type oferror says there is an effect when thereactually is not. A “False-Positive.”Related to p-value.

What is a type II (β) error? A type II error does not reject the nullwhen the alternative hypothesis isactually correct—this type of error doesnot detect an effect when there actuallyis one. A “False-Negative.”

What is power? It is the probability of rejecting the nullhypothesis when it is indeed incorrect.This is the study’s ability to detect aneffect, if present.

212 Deja Review: Behavioral Science

Page 228: Deja review   behavioral science

If the sample size increases, what It increases.happens to power?

What is statistical significance? That the probability of an observedeffect being due solely to chance is lowenough that the effect is likely true.

What is the probability (p) value? It is the percent chance that an effectseen between variables or in anexperimental group was due to chancealone. For example, if p = 0.25, there is a25% chance that the effect seen was justby chance (a type I error).

What p value is most commonly p <0.05considered a cutoff to determinestatistical significance?

What is the confidence interval (CI)? It is an estimated range of valueswhich is likely to include an unknownpopulation parameter, the estimatedrange being calculated from a givenset of sample data.

What is the point of p values and In research, we are looking for the trueconfidence intervals? value of the particular effect of one

variable on another variable. P valuesand CI express the probability that thedata we get from research is the truevalue of that effect.

What is a way to state in words a If the study was repeated an infinite 95% confidence interval? number of times, 95% of the time this

interval would contain the true value.(Therefore there is only a 5% chancethat the true value sought is outsideof this interval.)

How is the CI related to the p value? Closely! They are two ways ofdescribing the probability of anobserved effect being due to chance.

Can I guess the approximate p value Yes—a 95% confidence interval that from the CI? does not contain the reference value

(eg, 0 or 1.00 for relative risk/oddsratio) will have a p <0.05.

What’s the big deal about a 95% CI This would mean p >0.05, and the “crossing 1” or “crossing 0”? effect would not be “statistically

significant.”

Biostatistics 213

Page 229: Deja review   behavioral science

STATISTICAL TESTS

What types of data are evaluated Nominalin statistical tests? Ordinal

IntervalRatio

What is nominal data? It is categorical data where the order ofthe categories is arbitrary (eg, gender,religious beliefs).

What is ordinal data? It is categorical data where there is alogical ordering to the categories(eg, first place winner, second placewinner, third place winner, etc).

What is interval data? It is continuous data where there is aset interval between values, but wherethere is no “natural” zero (eg, Celsius,Fahrenheit).

What is ratio data? It is continuous data where there is botha set interval between values and anatural zero (eg, weight, height).

What type of common statistical Chi-squaretests is used to analyze nominal orordinal data?

What types of statistical tests are used t-testto analyze interval or ratio data? Analysis of variance (ANOVA)

Linear correlation

What is the difference between Parametric tests analyze data with a parametric and nonparametric normal distribution. Nonparametric statistical tests? analyze data sets without a normal

distribution.

What type of parametric statistical test t-testdetermines the difference betweenthe means of two groups?

Which type of t-test evaluates the means Independent (nonpaired) t-testof two groups at one period of time? (eg, comparing the mean income of one

set of orthopaedic surgeons, GroupBone, to the mean income of another setof orthopaedic surgeons, Group Tendon,at the beginning of the calendar year)

214 Deja Review: Behavioral Science

Page 230: Deja review   behavioral science

Which type of t-test evaluates Dependent (paired) t-test (eg, comparingthe means of two groups at two the mean income of one set of different time periods? orthopaedic surgeons, Group Bone,

to the mean income of another set oforthopaedic surgeons, Group Tendon, atthe beginning and in the middle of thecalendar year)

Which type of parametric ANOVAstatistical test determines thedifference of means of morethan two groups?

Which type of ANOVA test One-way ANOVA (eg, comparing thedetermines the difference of mean income of orthopaedic surgeons means of more than two groups in Group Bone, Group Tendon, andusing only one variable? Group Ligament at the beginning of

the calendar year)

Which type of ANOVA test Two-way ANOVA (eg, comparing the determines the difference of mean income and malpractice insurancemeans of more than two groups rate of orthopaedic surgeons in Groupusing two variables? Bone, Group Tendon, and Group

Ligament at the beginning of thecalendar year)

Which type of parametric Linear correlationstatistical test determines therelation between two continuousvariables?

What are some examples of Mann-Whitney Unonparametric tests? Wilcoxon’s

Biostatistics 215

Page 231: Deja review   behavioral science

CLINICAL VIGNETTES

A cohort study is done to relate amount of vegetables eaten per day with thedevelopment of diabetes. It is found that the group who is fed five servings perday of vegetables has a Relative Risk of 0.65 (95% CI = 0.40 − 0.95) for developingdiabetes (control group having RR = 1.00).

What is the independent variable?

The independent variable is amount of vegetables eaten (the manipulated variable).

What is the dependent variable?

The dependent variable is development of diabetes (dependent on vegetables eaten).

Are the results statistically significant?

The results are statistically significant as the CI does not cross 1.00 (the control value).

Is the p-value greater than or less than 0.05?

The p-value would be <0.05 (less than a 5% probability of the result due to chancealone).

Should the researchers accept or reject the null?

Researchers may reject the null hypothesis.

A research team at a tobacco company takes three subjects who smoke 1 pack perday and follows them over 25 years to monitor for the development of lung cancer.None of the subjects develop cancer. The researchers conclude that tobacco doesnot cause lung cancer (which is not true).

Did they reject or accept the null?

The researchers have accepted the null hypothesis, when in fact there was an effectand the null should have been rejected.

What type of error has their research committed?

They committed a type II (β) error.

How can they improve the power of their study?

Power can be most easily improved by increasing the number of subjects in the study.

Taste testers are asked to rate different hot sauces as poor, fair, good, or excellent.What type of data is being is collected? What would be a good statistical test toanalyze this data?

The data is ordinal—it has a logical order to it, but the intervals between thecategories are not well defined. A Chi-square would be appropriate.

216 Deja Review: Behavioral Science

Page 232: Deja review   behavioral science

AAbnormal thought formation, 88Abuse. See also Substance abuse

child, 57–60domestic partner, 60elder, 57–60emotional, 59–60sexual, 58–59

Acceptance, as stage of dying, 21Accuracy, 204Accutane, 94Acetylcholine, 80–81Acting out, as defense mechanism, 30Active suicidal ideation, 67Acute stress disorder (ASD), 103, 108ADHD (attention-deficit hyperactivity

disorder), 162Adjustment disorder, 108–110Adolescence, 12Adoption, children’s knowledge of, 12Adoption study, 69Adulthood, 15–17Advance directive, 184Aggression, 62–63Aging, 19–20. See also Elderly

memory and, dementia v., 113physiologic changes, in males, 17

Agnosia, 112Agoraphobia, 103, 105Agranulocytosis, 171–172Alcohol abuse, 148–150. See also Substance

abusechronic effects, 149ethanol

liver metabolism limits, 149toxicity, 143

genetic factors for, 72HVA and, 79relapse rate, 150screening for, 148sexual response and, 55–56thiamine deficiency, 115tolerance types, 149toxic effects, 149in twins, 72withdrawal, 141, 143

with benzodiazepines, 150benzodiazepines for, 166with disulfiram, 141DTs, 150

Alcohol abuse, withdrawal (Cont.):hallucinations, 150seizures, 150symptoms, 150

α (type I) error, 212Alternative hypothesis (H1), 212Altruism, as defense mechanism, 27, 30Alzheimer disease

brain changes, postmortem, 114cholinergic neuron decrease, 81clinical course, 113dementia and, 113–114genetic factors, 71medications, 114neuropeptides, 82risk factors, 114sleep patterns, 48vascular dementia v., 113

Amenorrhea, 153from antipsychotics, 172

Amines, 78–81. See also Acetylcholine;Dopamines; Serotonin

γ -Aminobutyric acid (GABA), 63, 81Amnestic syndromes, 115

brain structures affected, 115dementia v., 115

Amphetamines, 55, 144. See also Substanceabuse

Amygdala lesion, 76Anaclitic depression, 11Anal phase, 4Analysis of variance (ANOVA), 214–215Androgen insensitivity, 50Anemia, 153–154Anger, as stage of dying, 21Anhedonia, 93, 141Anorexia nervosa, 121, 153–154

bulimia nervosa v., 153medical effects of, 153–154treatment, 154

ANOVA (analysis of variance), 214–215Antipsychotics, 171–172

amenorrhea from, 172antitypical

mechanism of action, 171side effects, 171

for delirium, 112histamine blocking, 80for NMS, 91for schizophrenia, 90

217

Index

Page 233: Deja review   behavioral science

Antipsychotics (Cont.):therapy, side effects of, 172typical

extrapyramidal effects, 171mechanism of action, 171side effects, 171

Antisocial personality disorder, 29,127–128, 161

genetic factors, 71treatment, 128

Anxiety disorders, 103–110adjustment disorder, 108–110agoraphobia, 103ASD, 103GAD, 103–104genetic factors, 71OCD, 103panic disorder, 103–105PTSD, 103social phobia, 103specific phobia, 103suicide, 66

Anxiolytics, 165Aphasia, 112Apolipoprotein E4, 71Apomorphine, 53Apraxia, 112AR (attributable risk), 206Artificial life support, 186ASD (acute stress disorder), 103, 108Asperger disorder, 159–160Association, 205–206Associative learning, 35–39

classical conditioning, 35–37, 36fimprinting, 35–36nonassociative v., 35operant conditioning, 35, 37

Asymmetric tonic neck, 6Atomoxetine, 162Attachment, 11–12Attention-deficit hyperactivity disorder

(ADHD), 162Attributable risk (AR), 206Autistic disorder, 159–160Avoidance, as defense mechanism, 30Avoidant personality disorder, 71, 130–131

BBarbiturates, 81, 167Bargaining, as stage of dying, 21Basal ganglia lesion, 76Beneficence, 181Benzodiazepines

for alcohol withdrawal, 150, 166for anxiety, barbiturates v., 167anxiolytics, 165delirium from, 111

Benzodiazepines (Cont.):GABA channel opening, 81for GAD, 104high potency, 166hypnotics, 165lack of tolerance development, 165low potency, 166mechanism of action, 165metabolism phases, 166during pregnancy, 165reversal of effects, 165side effects, 166for sleep disorders, 47for social phobia, 107withdrawal symptoms, 166

Bereavement, 21–22, 22tβ (type II) error, 212Beta-blockers, 94Bias, 203–204Biostatistics, 209–216Bipolar disorder, 66, 91, 99, 101

mood stabilizers for, 100tpharmacologic agents, 172–173

during pregnancy, 173prevalence, 101types, 99, 101

Body dysmorphic disorder, 106, 121Borderline personality disorder, 29, 128–129

diagnostic criteria, 128genetic factors, 71splitting, 128treatment, 129

Brain, 75–77amnestic syndromes’ effects on, 115lesions, 76–77

consequences, by location, 76–77neuroanatomy, 75–76schizophrenia and, physical changes

from, 89Brief psychotic disorder, 86, 88Bulimia nervosa, 153–155

anorexia nervosa v., 153medical effects, 155treatment, 155

Buprenorphine, 146Bupropion, 96t, 168Buspirone, 104, 167

CCaffeine, 143Carbamazepine, 100t, 173Carbatrol. See CarbamazepineCatalepsy, 90Catatonic schizophrenia, 89Catecholamine, 78–79CBT (cognitive behavioral therapy), 106CCK (cholecystokinin), 82

218 Index

Page 234: Deja review   behavioral science

Celexa. See CitalopramCentral nervous system (CNS), 75–76Child abuse, 57–60

burns from, typical, 58fractures from, typical, 57leading causes of death, 196sexual, 58–59

age range, 59by gender, 59incidence rates, 58physical signs, 59psychological signs, 59

shaken baby syndrome, 58traits of abusers, 57

Child neglect, 57Childbirth. See Pregnancy and childbirthChildhood disintegrative disorder, 159, 161Children

adoption, knowledge of, 12developmental milestones, 6–8, 9–11tlegal consent for, 185neuropsychiatric disorders, 159–164

ADHD, 162disruptive behavior disorders, 161pervasive development disorders,

159–161selective mutism, 163separation anxiety disorder, 163Tourette disorder, 72, 162–163

understanding of death, 12Cholecystokinin (CCK), 82CI (confidence interval), 213Circadian rhythm sleep disorder, 45, 47Citalopram, 95t, 167Classical conditioning, 35–37, 36f, 39Climacterium, 17Clinical trial, 203Clomipramine, 106Close-ended questions, 179Clozapine, 171–172CNS (central nervous system), 75–76Cocaine, 55, 144. See also Substance abuseCodeine, 146Cognitive behavioral therapy (CBT), 106Cognitive disorders, 86, 111–116. See also

Delirium; Dementiaamnestic syndromes, 115delirium, 111–112dementia, 20, 112–114

Cohort study, 202Coining, 58Commitment, of patients, 188Competence, 183Compliance. See Patient adherenceCompulsions, 106Concordance, 69Conduct disorder, 127, 161

Confidence interval (CI), 213Confidentiality, patient, 178, 187–188Congenital adrenal hyperplasia, 50Coning, 180Conscious, as part of mind, 25Consent, 184, 186Continuous reinforcement, 39Conversion disorder, 117t, 119–120Correlation coefficient (r), 211Countertransference, 30Cravings, 142Cross-sectional study, 203Crystallized intelligence, 20Cupping, 58Cyclothymic disorder, 101Cymbalta. See DuloxetineCYP2D6, 146–147

DData sets, 210Death and dying

bereavement, 21–22, 22tchildren’s understanding of, 12five stages, 21–22

Defense mechanisms, 26–30, 31–32tDelirium, 111–112

causes, 111definition, 111dementia v., 114ttreatment, 112

Delirium tremens (DTs), 150Delusional disorder, 86, 91Dementia, 20, 112–114. See also Alzheimer

diseaseamnestic syndromes v., 115core symptoms, 112delirium v., 114tdiagnostic criteria, 112memory and aging v., 113prevalence, 113types, 113–114vascular, 113

Denialas defense mechanism, 29–30as stage of dying, 21

Depakote. See Valproic acidDependent personality disorder, 131–132Dependent variables, 211Depersonalization disorder, 136Depression. See also Major depressive

disorderanaclitic, 11bereavement, 22, 22tbody dysmorphic disorder and, 121double, 94ECT for, 173in elderly, 20

Index 219

Page 235: Deja review   behavioral science

Depression (Cont.):HVA and, 79learned helplessness and, 37mnemonic for, 93monoamine theory, 78postpartum, 16, 94serotonin levels, 47sleep disorders, 47–48as stage of dying, 21suicide and, 67treatment, 94–95, 95t, 96t

for postpartum, 16with SSRIs, 94, 95t, 167

Desyrel. See TrazodoneDetoxification, 142Development

through adolescence, 12milestones, 6–8, 9–11tTanner stages, 12, 13ttheories, 3–5

Developmental retardation, 11Dextromethorphan, 146Diabetes, sexual dysfunction and, in males,

55Diagnosis-related group (DRG), 192Diazepam, 167Disorganized speech, 87Displacement, as defense mechanism, 27–29Disruptive behavior disorders, 161Dissociation, as defense mechanism, 27Dissociative amnesia, 135Dissociative anesthetics, 144. See also

Substance abuseDissociative fugue, 135–136Dissociative identity disorder, 136Disulfiram, 141Dizygotic twins, 69, 72Do not resuscitate (DNR) order, 185Domestic partner abuse, 60Donepezil, 81, 114Dopamine hypothesis, 89“dopamine reward pathway,” 140Dopamines, 78–79

aggression and, 63HVA and, 79for psychiatric conditions, 78sexual response and, 55

Double-blind study, 204Down syndrome, 71, 81Downward drift, 89DRG (diagnosis-related group), 192Drug(s). See also specific drugs

first-order elimination, 149fmanic episodes from use of, 101sexual function and, 55–56withdrawal from, 141–142zero-order elimination, 148, 148f

DTs (delirium tremens), 150Duloxetine, 97tDyspareunia, 52Dyssomnias, 45–46Dysthymic disorder, 94

EEarly adulthood, 15–16Eating disorder(s), 153–157Eating Disorder Not Otherwise Specified

(NOS), 156Echolalia, 88ECT (electroconvulsive therapy),

98–99, 173Effexor. See VenlafaxineEgo, 25Elder abuse, 57–60Elder neglect, 58Elderly, 19–23

demographics, 19life expectancy, 19longevity factors, 21physiological changes, 19–20psychological characteristics, 20psychopathology, 20–21

Electroconvulsive therapy (ECT),98–99, 173

Emancipated minors, 183Emotional abuse, 59–60Endogenous opioids, 82Endorphins, 81Enkephalins, 81Epidemiology, 199–201Epilepsy, 81EPS (Extrapyramidal symptoms), of

schizophrenia, 90–91Equetro. See CarbamazepineErikson, Erik, 3–4Error types, 212–214Errors. See Medical errorsEscitalopram, 95tEskalith. See LithiumEstrogens, 51Ethanol

glutamate receptors, 147mechanisms of action, 147toxicity, 143

Ethics, medical, 181–189Ethylamine, 78, 80Etorphine, 146Euthanasia, 186Excitatory neurotransmitters, 77Exhibitionism, 54Experimental study, 202Extinction, 37–38Extrapyramidal symptoms (EPS),

of schizophrenia, 90–91

220 Index

Page 236: Deja review   behavioral science

FFactitious disorder, 117, 117t, 122–123Factitious disorder by proxy, 122–123Failure to thrive, 11Family risk study, 69Fatigue, 93Fentanyl, 146Fetal alcohol syndrome, 143Fetishism, 54First-order elimination, of drugs, 149fFive stages of death and dying, 21–225-HIAA (5-Hydroxyindoleacetic acid), 805-HT (5-hydroxytryptamine), 79Fixation, as defense mechanism, 27Flooding, 40, 109Flumazenil, 165Fluoxetine, 95t, 167–168Fluvoxamine, 167Fragile X syndrome, 72Free association, 27Freud, Sigmund. See also Psychoanalytic

theorydevelopmental theory, 3–4structural mind theory, 25topographic mind theory, 25

Frontal lobe lesion, 76Frotteurism, 54

GGABA (γ -Aminobutyric acid), 63, 81GAD (generalized anxiety disorder),

103–104Ganser syndrome, 123Gender identity, 49Gender identity disorder, 49, 121Gender roles, 49Generalized anxiety disorder (GAD),

103–104Genetic studies, 69Genetics, 69–73

alcoholism and, 72neuropsychiatric disorders and, 71–72psychiatric disorders and, 69–71

Genital phase, 4Glutamate, 81, 147Glycine, 81

HH0 (null hypothesis), 212H1 (alternative hypothesis), 212Habituation, 40Hallucinations, 85–86

alcohol withdrawal, 150illusions v., 86

Haloperidol, 112Health maintenance organization (HMO),

193

Health status, determinants for, 195–196Health-care costs, 193–194Health-care delivery systems, 194–195Health-care insurance, 191–193

coverage, 192government programs, 191–192private, 193types, 191–192

Health-care managed-care plans, 193Heroin, 56, 146. See also Substance abuseHimmelsbach hypothesis, for substance

abuse, 140Hippocampus lesion, 76Histamines, 80Histrionic personality disorder, 29, 71, 130HIV (human immunodeficiency virus),

113, 182HMO (health maintenance

organization), 193Homicide, 62Homosexuality, chromosomal factors, 50Homovanillic acid (HVA), 79Hormones, behavior influenced by, 51Hospice care, 194–195Human immunodeficiency virus (HIV),

113, 182Humor, as defense mechanism, 27–28Huntington’s Disease, 71, 113HVA (Homovanillic acid), 79Hydromorphone, 146Hypertensive crisis, 98Hypnotics, 165Hypoactive sexual desire, 52Hypochondriasis, 120Hypomania, 99Hypothalamus lesion, 76–77Hypotheses, 212–214

IId, 25Identification, as defense mechanism, 27, 29Illusions, 86Immature defense mechanisms, 27Impotence, 52Imprinting, 35–36Incidence, 199Incompetence, in patients, 183Independent variables, 210Indolamines, 78Infants. See also Neonates

developmental milestones, 6–8, 9–11tmorbidity and mortality, 5

leading causes, 196Informed consent, 186Inhibitory neurotransmitters, 77Insomnia, 45–46, 167–168Insurance. See Health-care insurance

Index 221

Page 237: Deja review   behavioral science

Intellectualization, as defense mechanism,27–28

Intelligence quotient (IQ), 41Intelligence tests, 41Interferon, 94Interpreters, 179Interrater reliability, 204IQ (intelligence quotient), 41Isolation, as defense mechanism,

27–28

JJustice, 181

KKappa receptor agonists, 146Kappa receptor antagonists, 146Ketamine, 144. See also Substance abuseKleine-Levin syndrome, 46Kubler Ross, Elizabeth, 21–22

LLamictal. See LamotrigineLamotrigine, 100t, 173Lanugo, 153–154Latency phase, 4Learned helplessness, 37Learning theory, 35–42

associative, 35–39classical conditioning, 35–37, 36foperant conditioning, 35, 37

nonassociative, 35, 39–41habituation, 40observational learning, 39

Leukopenia, 153–154Levodopa, 111Lexapro. See EscitalopramLife expectancy, 19Linear correlation, 214–215Lithium, 100t, 172Living wills, 185Longevity, 21Loperamide, 147Lorazepam, 167

MMajor depressive disorder, 66, 91, 93–98

dysthymic disorder v., 94genetic factors, 70MAOIs, 98from medications, 94prevalence, 93psychotherapy treatments, 98–99suicidal ideation, 93symptoms, 93TCAs, 97–98treatment for, 94–95, 95t, 96t

Malingering, 117, 117t, 122, 123Malpractice, 181–182Mania, 99, 101Mann-Whitney test, 215MAOIs. See Monoamine oxidase inhibitorsMarijuana, 55–56, 144–145. See also

Substance abuseMarriage, 15Mature defense mechanisms, 27Mean, 209Median, 209Medicaid, 191Medical errors, 177Medical ethics, 181–189Medicare, 191–192Melanosis coli, 154Memantine, 114Menopause, 17Mental age, 41Mental retardation, 41Meperidine, 146Methadone, 56, 147. See also Substance abuse3-Methoxy-4-hydroxyphenyglycol (MHPG),

79Methyldopa, 94N-methyl-D-aspartate (NMDA), 114Methylphenidate, 162MHPG (3-Methoxy-4-hydroxyphenyglycol),

79Middle adulthood, 17Mirtazapine, 96t, 104Mode, 209Modeling, in nonassociative learning, 39–40Monoamine oxidase inhibitors (MAOIs),

67, 170drug interactions, 170mechanism of action, 170side effects, 170

Monoamine theory of depression, 78Monozygotic twins, 69–72Mood disorders, 86Mood stabilizers, 100tMoro reflex, 5–6Morphine, 146Mu receptor agonists, 146Mu receptor antagonists, 146

NNalorphine, 146Naltrexone, 146Namenda. See MemantineNarcissistic personality disorder,

121, 129–130Narcolepsy, 45–46, 86Narcotic overdose. See Overdose, narcoticNecrophilia, 54Nefazodone, 96t

222 Index

Page 238: Deja review   behavioral science

Negative predictive value (NPV), 200, 201fNegative reinforcement, 37–38Neologisms, 88Neonates. See also Infants

reflexes, 5–6Neuroanatomy, 75–76Neuroleptic malignant syndrome (NMS), 91Neuropeptides, 81–82Neurotensin, 82Neurotransmitters, 77–78

alterations in psychiatric conditions, 78trelease steps, 77

NMDA (N-methyl-D-aspartate), 114NMS (neuroleptic malignant syndrome), 91Nominal data, 214Nonassociative learning, 35, 39–41Nonmaleficence, 181Nonrapid eye movement (NREM), 43–45Norepinephrine

aggression and, 63as biogenic amine, 79MHPG, 79sexual response and, 55synthesis, 79

Normal distribution, 209NOS (Eating Disorder Not Otherwise

Specified), 156NPV (negative predictive value), 200, 201fNREM (nonrapid eye movement), 43–45Null hypothesis (H0), 212Nursing homes, 194

OObservational learning, 39Observational study, 202Obsessive-compulsive disorder (OCD),

28, 103, 105–106, 132Obsessive-compulsive personality disorder

(OCPD), 28, 103, 106, 132Obstructive sleep apnea (OSA), 46–47OCD. See Obsessive-compulsive disorderOCPD. See Obsessive-compulsive

personality disorderOdds ratio (OR), 206Open-ended questions, 179Operant conditioning, 35, 37–39Opiates, 146Opioid receptors

location, 145mechanisms of action, 145types, 145

Opioidsagonists, 145–146antagonists, 145endogenous, 82mechanisms of action, 145opiates v., 146

Oppositional defiant disorder, 161OR (odds ratio), 206Oral phase, 4Ordinal data, 214Orgasmic disorder, 52OSA (obstructive sleep apnea), 46–47Osteoporosis, 17Overdose, narcotic, 147Oxycodone, 146Oxytocin, 82

Pp (probability) value, 213Palliative care, 186, 194Palmar grasp, 6Panic disorder, 103–105Papaverine, 53Parachute reflex, 6Paranoid personality disorder, 29, 126–127Paranoid schizophrenia, 89Paraphilias, 54Parasomnias, 45Parietal lobe lesion, 76Parkinson’s disease, 78–79Parotiditis, 155Paroxetine, 95t, 167–168Passive suicidal ideation, 67Patient adherence, 179Patient autonomy, 181Patient confidentiality, 178, 187–188Patient referrals. See Referrals, patientPavlov’s classical conditioning, 36, 36fPaxil. See ParoxetinePedigree study, 69Pedophilia, 54Pentazocine, 146Peripheral nervous system (PNS), 75–76Personality disorders, 66, 125–133

classification, 125–126cluster A, 126–127cluster B, 127–128cluster C, 130–131definition, 125diagnostic criteria, 125psychosis and, 86

Pervasive development disorders, 159–161Asperger disorder, 159–160autistic disorder, 159–160childhood disintegrative disorder, 159, 161Rett disorder, 159–160

Phallic phase, 4Phencyclidine, 144. See also Substance abusePhentolamine, 53Phobias, 37Physician impairment, 182Piaget, Jean, 3–5PNS (peripheral nervous system), 75–76

Index 223

Page 239: Deja review   behavioral science

“Pooled-risk,” 192Positive predictive value (PPV),

200, 201fPositive reinforcement, 37–38Postpartum “blues,” 15–16Postpartum major depression, 16, 94Postpartum psychosis, 16Posttraumatic stress disorder (PTSD),

62, 103, 107–108adjustment disorder v., 108ASD v., 108suicide and, 66

Power, 212–213Power of attorney, 184PPOs (preferred provider organizations),

193PPV (positive predictive value), 200, 201fPrazosin, 108Precision, 204Preferred provider organizations (PPOs),

193Pregnancy and childbirth

benzodiazepines during, 165bipolar disorder and, pharmacologic

treatment, 173postpartum period, 15–16

“blues,” 15–16major depression, 16psychosis, 16

Premature birth, 5Premature ejaculation, 52Prevalence, 199Priapism, 168Primary process thinking, 26Principle of double-effect, 181Probability (p) value, 213Progesterone, 51Projection, as defense mechanism,

27, 29Prospective study, 201Prozac. See FluoxetinePsychiatric disorders, 69–71. See also Major

depressive disorder; Psychosis;Schizophrenia

Psychoanalytic theory, 25–33defense mechanisms, 26–30, 31–32tpsychotherapy, 26–27structural theory of the mind, 25topographic theory of the mind, 25

Psychosis, 85–86clinical hallmarks, 85hallucinations, 85–86from medical conditions, 86narcolepsy and, 86personality disorders and, 86from pharmacological agents, 171postpartum, 16

Psychotherapy, 26–27for major depressive disorder, 98–99psychodynamic, 26–27for specific phobia, 109

Psychotic disorders, 85–92PTSD. See Posttraumatic stress disorderPunishment, 37–38Purging, 155

QQuaternary amines, 78

Rr (correlation coefficient), 211Rape, 61–62Rapid eye movement (REM), 20, 43–45Rapport, 177Ratio data, 214Rationalization, as defense mechanism,

27–28Reaction formation, as defense mechanism,

27–28Referrals, patient, 177Refusal of treatment

by patients, 186by physicians, 187

Regression, as defense mechanism, 27, 29Reinforcement, in operant conditioning,

37–39Relapse, 142, 150Relative risk (RR), 205–206, 205fReliability, 204REM (rapid eye movement), 20, 43–45Remeron. See MirtazapineRepression, as defense mechanism, 27, 29Research study designs, 201–203Reserpine, 94Reticular system lesion, 76Retrospective study, 202Rett disorder, 159–160Risperidone, 172Ritalin, 46Rivastigmine, 114Romazicon. See FlumazenilRooting reflex, 6RR (relative risk), 205–206, 205f

SSchizoaffective disorder, 86, 91Schizoid personality disorder, 126, 130–131Schizophrenia, 69–70, 87–91

abnormal thought formation, 88antipsychotics for, 90catalepsy, 90catatonic, 89characteristic symptoms, 87–88disorganized speech, 87

224 Index

Page 240: Deja review   behavioral science

Schizophrenia (Cont.):dopamine hypothesis, 89dopamines for, 78downward drift, 89EPS, 90–91gender and, 69, 89genetic factors, 69–70among monozygotic twins, 70paranoid, 89prevalence, 69prodromal phase, 88prognosis, 90residual phase, 88schizotypal personality disorder and, 127subtypes, 89suicide and, 66, 90TD, 90–91waxy flexibility, 90

Schizophreniform disorder, 86Schizotypal personality disorder, 29, 71, 127Selective mutism, 163Selective serotonin reuptake inhibitors (SSRIs)

for children, 167commonly used, 167for depression, 94, 95t, 167for GAD, 104MAOIs and, 98for panic disorder, 105side effects, 167for suicide, 67

Sensitivity, 200Sensitization, 41Separation anxiety disorder, 163Serotonin, 79–80

altered behavioral factors, 79–80as biogenic amine, 79–80decreased aggression and, 63depression, 47sexual response and, 80in sleep, 44synthesis, 80

Serotonin syndrome, 98Sertraline, 95t, 167Serzone. See NefazodoneSexual abuse, 58–59Sexual assault, 61–62Sexual aversion disorder, 53Sexual consent, 61Sexual development, 49–51Sexual masochism, 54Sexual response, 51–56

drugs as influence on, 55–56dysfunction, 52–53medical conditions and, 54–55normal cycle, 51–52paraphilias, 54serotonin and, 80

Sexual sadism, 54Sexual violence, 61–62Sexuality, Tanner stages of development for,

12, 13tShaken baby syndrome, 58Shaping, 38Shared psychotic disorder, 91SIG E CAPS, 93Sildenafil citrate, 53Sleep, 43–48

abnormal, 45–47Alzheimer’s disease, 48depression, 47–48disorders, 45–47normal, 43–45patterns among elderly, 20

Sleep terrors, 47Sleepwalking, 47Social phobia, 103, 106–107, 109Somatization disorder, 71, 118–119Somatoform disorders, 117–124

body dysmorphic disorder, 106, 121conversion disorder, 117t, 119–120factitious disorder, 117, 117t, 122–123hypochondriasis, 120malingering, 117, 117t, 123primary types, 117, 117tsomatization disorder, 71, 118–119

Somatostatin, 82Specific phobia, 103, 108–109Specificity, 200Splitting, as defense mechanism, 27–29, 128Spontaneous recovery, 37Spooning, 58SSRIs. See Selective serotonin reuptake

inhibitorsStandard deviation, 211–212, 211fStanford-Binet scale, 41“Startle reflex.” See Moro reflexStatistics

distribution, 209–212, 210fhypothesis and error types, 212–214tests, 214–215

Statutory rape, 62Stepping reflex, 6Steroid use, 63, 94Stimulus generalization, 37Structural mind theory, 25Sublimation, as defense mechanism, 27–28Substance abuse, 139–151

aggression and, 63cravings, 142delirium and, 111diagnostic criteria, 139dissociative disorders, 135among elderly, 21Himmelsbach hypothesis, 140

Index 225

Page 241: Deja review   behavioral science

Substance abuse (Cont.):medical effects, 143mood disorders and, 86nonmedical effects, 144physiological pathways, 140relapse, 142sexual assault and, 61sexual response and, 55–56suicide, 66tolerance and, 139treatment, 139–142

detoxification, 142negative reinforcement, 142positive reinforcement reduction, 141principles, 140substitute drugs, 141symptom prevention, 140withdrawal, 139–142

Substance P, 82Suicide, 65–68

antidepressants, 67depression and, 67gender and, 65ideation, 67major depressive disorder and, 93MAOIs in treatment therapy, 67PTSD, 66race and, 66risk factors, 65–66schizophrenia, 66substance abuse, 66

Superego, 26Suppression, as defense mechanism, 29Systematic desensitization, 40

TTacrine, 81Tanner stages of development, 12, 13tTardive dyskinesia (TD), 90–91TCAs. See Tricyclic antidepressantsTD (tardive dyskinesia), 90–91Tegretol. See CarbamazepineTemporal lobe lesion, 76Testing, 203–204Testosterone, 51Test-retest reliability, 204Thiamine deficiency, 115Thought blocking, 88Tolerance, 139, 149. See also Substance abuseTopographic mind theory, 25

Tourette disorder, 72, 162–163Transference, 30Transvestic fetishism, 54Trazodone, 96t, 168Trichotillomania, 106Tricyclic antidepressants (TCAs), 80, 97–98,

169–170commonly used, 98, 169mechanism of action, 168–169overdose, 169side effects, 97, 169tertiary, 169

t-test, 214–215Turner syndrome, 51Twins. See Dizygotic twins; Monozygotic

twinsType I (α) error, 212Type II (β) error, 212

UUprima. See Apomorphine

VValproic acid, 100t, 173Vanillylmandelic acid (VMA), 79Variable ratio reinforcement, 39Variables, 210Vascular dementia, 113Vasoactive intestinal peptide (VIP), 82Vasopressin, 82Venlafaxine, 97t, 104Viagra. See Sildenafil citrateVIP (vasoactive intestinal peptide), 82VMA (vanillylmandelic acid), 79Voyeurism, 54

WWaxy flexibility, 90Wellbutrin. See BupropionWilcoxon test, 215Withdrawal. See also Substance abuse

from alcohol, 141, 143from benzodiazepines, 166from drugs, 141–142

Word salad, 88

ZZero-order elimination, of drugs,

148, 148fZoloft. See Sertraline

226 Index